Download as pdf or txt
Download as pdf or txt
You are on page 1of 210

 

 
Copyrighhts © 2016 by
b the Deparrtment of Clin
nical Pharma
acology at Fa
aculty of Meedicine, Mnas
soura
Universitty, Egypt.

Previouss editions copyright © 20


015, 2014, 20
013, 2012, 2011, 2010, 2009,
2 2008, 22000 by the
Departm
ment of Cliniccal Pharmacoology at Facu cine, Mnasou
ulty of Medic ura Universitty, Egypt.

No part of this book may be reprroduced or d distributed in


n any form orr by any meaans, or stored
d in a
databasee or retrievall system, witthout the prio
or written pe
ermission of the
t copyrighhts owner,
Departm
ment of Cliniccal Pharmaco ology at Facuulty of Mediccine, Mansouura Universitty.

This is a copyrightedd work and is s protected b


by the Egypttian Intellectu
ual Property Law 82 of 2002. Use
of this w
work is subjec
ct to this law w. The Departrtment of Clin
nical Pharmaacology at MMansoura Fac culty of
Medicine e reserves alll rights in an
nd to the worrk.

2000 ‫سنة‬
‫ لس‬1456 :‫رقم اإليداع بدار الككتب‬
‫م‬
2000/9/6 ‫ بتاريخ‬ 
Preface

C
linical training for undergraduate students often focuses on diagnostic rather than
therapeutic skills. Sometimes students are only expected to copy the prescribing
behavior of their clinical teachers, or existing standard treatment guidelines, without
explanation as to why certain treatment is chosen. Books may not be much help either.
Pharmacology reference works and formularies are drug-centered, and although clinical
textbooks and treatment guidelines are disease-centered and provide treatment
recommendations, they rarely discuss why these therapies are chosen. Different sources
may give contradictory advice.
This book in primarily intended for under graduate medical students who are about to
enter the clinical phase of their studies. It will provide step by step guidance to the process
of rational prescribing together with many illustrative examples. It teaches also skills that are
necessary throughout a clinical career. Postgraduate students and practicing doctors may
also find it a source of straightforward information.
I wish to acknowledge the ongoing efforts of my contributing authors, and we are deeply
grateful to all those who have with such good grace given us their time and energy to supply
valuable facts and opinions, they principally include:

 Prof. Hussein El-Beltagi who took over the preparation of all books since the 1st edition
in 1995 including the revision process, printing control, distribution and selling control.
 Assist. Prof. Mohamed-Hesham Daba who took over the revision process and
amendments of the last two editions.
 Assist. Prof. Abdel-Motaal Fouda who prepared the last two editions in a readable up-
to-date text to provide essential information necessary throughout the clinical career.
 Dr. Sameh Abdel-Ghany who assisted in the revision process.

Much of any merit this book may have is due to the


generosity of those named above.

Gamal M. Dahab (MD, PhD)

Professor Emeritus in Clinical Pharmacology


Mansoura Faculty of Medicine
 

iii
Miss
sion and
a Vis
sion

Our mission
The Clinnical Pharm macology Department
D is seeking excellence
e and leadeership in fou
ur major
core acctivities: edu
ucation, ressearch, com
mmunity serrvice, and faculty
f and staff development.
We are e connectin ng basic medical
m sc iences with clinical care
c througgh innovattive and
disciplin
ned teachin ng of clinica
al pharmaco
ology in an integrative
i manner
m

Our v
vision
The dep partment off Clinical Ph
harmacolog gy is aiming
g to be a premier acad demic model in the
field of pharmacoloogy and the erapeutics in Egypt annd Middle East
E throug h promoting use of
the bestt therapeutics and dev veloping new
wer experimmental and clinical reseearch proje
ects.

Value
es

The gu
uiding prin
nciples and beliefs ffor the dep
partment

 Excellence, creativity, innovation, fairness, honesty, transparenncy, collab


boration,
teammwork and lifelong learning
 Reccognition tha
at our stude
ent comes ffirst
 All m
members ofo our department musst see them mselves as integral to the successs of our
misssion and ouur departmeent as integ ral to their personal
p su
uccess.
 As we subscrribe to the ese values,, we shall be profes ssionals in the profes
ssion of
education.

iv
Contributers

Effat A. Haroun MD, PhD Somaya A. Mokbel MD, PhD


Prof. of Clin Pharmacology Prof. of Clin Pharmacology
Mansoura Faculty of Medicine Mansoura Faculty of Medicine

Elhamy M. El-Kholy MD, PhD Amany A. Shalaby MD, PhD


Prof. of Clin Pharmacology Prof. of Clin Pharmacology
Mansoura Faculty of Medicine Mansoura Faculty of Medicine

Gamal M. Dahab MD, PhD, MSc (Int.Med) Amal Abdel-Hamid MD, PhD
Prof. of Clin Pharmacology Prof. of Clin Pharmacology
Mansoura Faculty of Medicine Mansoura Faculty of Medicine

Farida M. El-Banna MD, PhD Essam A. Ghyati MD, PhD


Prof. of Clin Pharmacology Assist. Prof. of Clin Pharmacology
Mansoura Faculty of Medicine Mansoura Faculty of Medicine

Aly M. Gaballah MD, PhD, MSc (int.Med) Mohamed-Hesham Y. Daba MD, PhD
Prof. of Clin Pharmacology Assist. Prof. of Clin Pharmacology
Mansoura Faculty of Medicine Mansoura Faculty of Medicine

Layla T. Hanna MD, PhD Abdel-Motaal M. Fouda MD, PhD


Prof. of Clin Pharmacology Assist. Prof. of Clin Pharmacology
Mansoura Faculty of Medicine Mansoura Faculty of Medicine

Mohamed Kheriza MD, PhD, MSc (Int.Med) Vivian Boshra MD, PhD
Prof. of Clin Pharmacology Assist. Prof. of Clin Pharmacology
Mansoura Faculty of Medicine Mansoura Faculty of Medicine

Abdel-Rahman A. Yassin MD, PhD Hala A. Al-Ashri MD, PhD


Prof. of Clin Pharmacology Assist. Prof. of Clin Pharmacology
Mansoura Faculty of Medicine Mansoura Faculty of Medicine

Mohmmad A. Attia MD, PhD Nageh Rizk MD, PhD


Prof. of Clin Pharmacology Lecturer in pharmacology
Mansoura Faculty of Medicine Mansoura Faculty of Medicine

Mohamed Abdel-Ghani MD, PhD Elsayed A. Hassan MD, PhD


Prof. of Clin Pharmacology Lecturer in Clin Pharmacology
Mansoura Faculty of Medicine Mansoura Faculty of Medicine

Hussien M. El-Beltagi MD, PhD Mohamed Abdel-Monem MD, PhD


Prof. of Clin Pharmacology Lecturer in Clin Pharmacology
Mansoura Faculty of Medicine Mansoura Faculty of Medicine

Karawan M. Abdel-Rahman MD, PhD Mahmoud A. Naga MD, PhD


Prof. of Clin Pharmacology Lecturer in Clin Pharmacology
Mansoura Faculty of Medicine Mansoura Faculty of Medicine

 
  v
Ahmad Hassan MD, PhD Mohamed Abou El-khair MD, PhD
Lecturer in Clin Pharmacology Lecturer in Clin Pharmacology
Mansoura Faculty of Medicine Mansoura Faculty of Medicine

Ahlam El-masry MD, PhD Sameh A. Abdel-Ghani MSc.


Lecturer in Clin Pharmacology Assist. Lecturer in Clin Pharmacology
Mansoura Faculty of Medicine Mansoura Faculty of Medicine

Rehab Hamdy MD, PhD


Lecturer in Clin Pharmacology
Mansoura Faculty of Medicine

vi iii
Table of Contents
 

CHAPTER 8: GASTROINTESTINAL PHARMACOLOGY

Part 1: Peptic ulcer and reflux esophagitis 251


Peptic ulcer 251
Gastroesophageal reflux disease 259
Part 2: Management of liver disease complications 262
Hepatic encephalopathy 262
Bleeding esophageal varices 263
Part 3: Antiemetic drugs 264
Part 4: Antispasmodic drugs 266
Part 5: Therapy of constipation 266
Part 6: Therapy of diarrhea 269
Part 7: Drug therapy of gall stones 274
Review questions 276

CHAPTER 9: ENDOCRINE PHARMACOLOGY

Part 1: Diabetes mellitus and antidiabetic drugs 281


Insulin 283
Oral antidiabetic drugs 288
Treatment of diabetic complications 292
Part 2: Medical treatment of obesity 295
Part 3: Thyroid gland and antithyroid gland 297
Part 4: Adrenocortical steroids 303
Part 5: Regulation of calcium metabolism 307
Part 6: Sex hormones 311
Part 7: Hypothalamic and pituitary hormones 316
Review questions 320

CHAPTER 10: CNS PHARMACOLOGY

Part 1: CNS stimulants 327


Part 2: Analgesics 329
Opioid analgesics 329
Non-opioid analgesics 337
Part 3: Sedative hypnotic drugs 339
Part 4: Skeletal muscle relaxants 343
Part 5: Antiepileptic drugs 345
Part 6: Antidepressant drugs 350

vii
Part 7: Mood-stabilizing drugs 354
Part 8: Antipsychotic drugs 355
Part 9: Antiparkinsonian drugs 358
Part 10: Drugs used for Alzheimer disease 362
Part 11: General anesthetic drugs 363
Part 12: Local anesthetic drugs 369
Review questions 373

CHAPTER 11: ANTIMICROBIAL DRUGS

Part 1: Basic principles of antimicrobial drugs 379


Part 2: Individual classes of antibiotics 385
Cell wall inhibitors 385
Inhibitors of bacterial protein synthesis 394
Inhibitors of bacterial nucleic acid synthesis 402
Inhibitors of bacterial metabolism 404
Miscellaneous antibacterial drugs 406
Part 3: Urinary tract infection 408
Part 4: Chemotherapy of TB and leprosy 412
Part 5: Antiviral drugs 418
Part 6: Chemotherapy of fungal infections 421
Part 7: Antiamoebic drugs 425
Part 8: Antimalarial drugs 428
Part 9: Anthelmintic drugs 433
Review questions 437

viii
   
 
Part 1
1: Pep
ptic Ulce
er Disea
ase and Reflux Esophag
E gitis

█ PEP
PTIC ULCE
ER

Definittion: ulcera
ation of the
e duodenuum or stom
mach due tot imbalannce betwee
en local
invasive force (e.g
g. HCl andd pepsin) a
and protecttive mecha
anisms.

Invasiv
ve factors:
 ess: ↑ HCl and pepsin secretion
Stre n by pariettal cells.
 Diett: coffee, alcohol
a and
d spices.
 Druugs: NSAID Ds, corticosteroids, m
morphine, methylxan nthines, etcc.
 ection with
Infe h Helicoba acter pylorri.
H. ppylori is sppiral gram –ve flageellates foun
nd in the antrum
a of human sttomach.
Cerrtain enzym mes and toxins
t prooduced by y the bacteria causee tissue damage.
d
Infe
ection with h H. pylori can be d diagnosed by endosc copic biop psy or serological
marrkers.

Defens
sive mech
hanisms:
 Muccus prod duction by gasttric
muccosa.
 Pan
ncreatic bic
carbonate secretion.
 Goo
od mucosa al blood flo
ow.
 Loc
cal PGE2 annd PGI2 production.

Regula
ation of HC
Cl secretio
on
 Ach
h: ↑ HCl seecretion thrrough M1 rreceptors → ↑ intrace
ellular Ca2+.
 Gasstrin: ↑ HC n through G receptorrs → ↑ intra
Cl secretion acellular Caa2+.
 Histamine: ↑ HCl secretion througgh H2 rece eptors → ↑ intracellulaar cAMP.

 
  251
Both C Ca2+ and cAMP activate H+ /K+
ATPase e at the membrane ofo the pari etal
+
cell to secrete H into the gastric lum
men
“proton pump”.
 PGE
E2 and PGI2: act on PG
P recepto
ors
→ ↓ cAMP → ↓ HCl secre
etion.

Clinica
al picture
 Epig
gastric paiin: charactterized by:
– DDiffuse annd worsen ns by food d in
GGU.
– L Localized (point ten nderness) a and
rrelieved byy food in DU.
D
 Signns of comp plications e.g. bleediing,
ane
emia, etc.

Diagno
osis
– End
doscopy: visualizatio
v cer.
n of the ulc
– Rad
diologic: byy barium meal.
m

█ The
erapy of peptic
p ulcer

▌Non-d
drug thera
apy = life style
s mod
dification

 Resst and Sedation: theyy improve hhealing and relief


pain of DU.
 Stopp Smoking g, Spices, alcohol, coffee, an nd tea:
beccause theyy ↑ HCl.
 Avo
oid Stress: because stress
s ↑ HC
Cl.
 Avo
oid ulcerogenic drugs s: e.g. NSAAIDs.
 Diett:
– Frequeent small meals
m in DU
U in order to buffer high acidityy.
– Encourage milk anda fats.
– Avoid Spices
S and
d fried foodd

▌Pharm
macologic
cal therapy
y

■ Dru
ugs that neutralize HCl:
H antac
cids

■ ugs that ↓ HCl secre


Dru etion:

 
252  
– Selective M1 blockers: pirenzepine, telenzepine.
– H2 blockers: cimetidine, ranitidine, famotidine.
– Proton pump inhibitors: omeprazole, lanzoprazole, etc.

■ Drugs that ↑ mucosal defense mechanisms:


– Sucralfate
– Colloid bismuth compounds: e.g. bismuth subcitrate.
– Carbenoxolone
– PGE1 analogues: misoprostol.

■ Antimicrobial drugs for H. pylori: see later.

■ Adjuvant therapy: Sedatives and multivitamins to ↓ stress to enhance healing.

█ ANTACIDS

 Antacids are weak bases that are taken orally and partially neutralize gastric acid
and reduce pepsin activity.
 They are used as symptomatic relief of hyperacidity and should not be used as
long-term treatment.

Sodium bicarbonate Calcium carbonate Magnesium and


aluminum salts
(Mg hydroxide and
Aluminium hydroxide)
 It can be absorbed  Partially absorbed ant-  They are poorly ab-
systemically leading to acid. sorbed from GIT and
salt & water retention,  Ca2+ may act directly to have no systemic
and metabolic stimulate gastrin secre- effects.
alkalosis. tion leading to acid  The unabsorbed Mg
 It is contraindicated in rebound. salts cause osmotic di-
hypertension and heart  It is contraindicated in arrhea; the unabsorbed
failure. hypercalcemia and Al salts cause
 It has rapid onset and renal stones. constipation.
short duration.  They have slow onset.

Adverse effects
■ Change in bowel habits: Al3+ hydroxide causes constipation, while Mg2+
hydroxide cause diarrhea. For this reason, both salts are combined together to
manage this problem.

 
  253
■ Reb
bound hyp
peracidity a2+ and
y: with Ca
NaH
HCO3 containing anta
acids.

■ Cattion overlo
oad:
– Na+ saltss → hyp pertension
n and
ssystemic alkalosis.
a
– C 2+
Ca saltss → hyperrcalcemia, renal
sstones and
d milk-alk kali syndro
ome.

■ Dec
crease ab bsorption of other d drugs: the e metal ion n in some preparatio
ons can
che
elate other drugs esp
pecially tetrracycline, digitalis
d an
nd iron.

N.B. wwhen iron or


o tetracyccline is pre
escribed with
w antacid ds, we sho
ould make 30 min
interval between both drugs to avoid chelationn with the antacid.
a

█ DEC
CREASE HCL
H SECR
RETION

1. Sellective M1 blocke
ers:
(Pirenz
zepine - Te
elenzepine)

Mecha anism of action: selectively


s astric M1 receptors → ↓ bas
block ga sal HCl
secretio
on.

Uses: TThe selecttive M1 blo


ockers are hibitors of HCl secreetion and are
e weak inh a now
replace
ed by morre effectivee drugs. T
They are sometimes
s s used as adjuvant therapy
with H2 blockers.

Advers s: high do
se effects oses prod pine-like effects: dryy mouth, blurred
duce atrop
vision, tachycardia, urine re
etention.

N.B. Attropine itse


elf is not used
u in the
e treatment of PU be
ecause it iss non-sele
ective M
blockerr and may y aggravatte esophag geal reflux
x.

2. H2 blockerrs:
(Cimettidine – Ra
anitidine – Famotidi ne - Nizattidine)

Mecha
anism and pharmac
cological e
effects

 Theey act as competitive


c e inhibitorss of histam
mine H2-receptors onn the parie
etal cell.
Thiss results in d ↓ in histam
n a marked mine-stimu
ulated HCl secretion .
 Alth
hough othe er agents such as gastrin an nd ACh maym inducee acid seecretion,
histtamine is thhe predom
minant final mediator that stimulates HCl ssecretion.

 
254  
Therapeutic uses
 Duodenal and gastric ulcers.
 Prophylaxis & treatment of stress ulcers (e.g. after burn or major trauma).
 Prophylaxis against bleeding of esophageal varices.
 Reflux esophagitis.
 Zollinger-Ellison syndrome (gastrin-secreting tumor of the pancreas which ↑ HCl
secretion): usually larger doses are required.
 With ulcerogenic drugs (e.g. NSAIDs) to protect the gastric mucosa from injury.

Adverse effects (mostly with cimetidine):


– Cimetidine has anti-androgenic effects (due to block of androgen receptors)
leading to ↓ sperm count, impotence and gynecomastia.
– Cimetidine inhibits hepatic microsomal enzymes (P450) leading to ↓
metabolism of other drugs e.g. theophylline, warfarin, sulphonylureas, etc.
– Reversible hepatotoxicity and Reversible anemia.
– CNS symptoms: headache, slurred speech, delirium, coma, etc. occurs mainly in
elderly people with i.v. administration.

Precautions of H2 blockers
– Avoid sudden withdrawal to prevent rebound ulceration.
– Avoid their use in pregnancy and lactation (they cross the placental barrier and
secreted in breast milk).
– Avoid combination of cimetidine with drugs having narrow therapeutic index
(because cimetidine inhibits microsomal P450 and ↑ their toxicity).

Cimetidine Ranitidine Famotidine


H2 Blocking effect: Weak Potent More potent
Anti-anderogenic effect: Strong Minimal No
Liver enzyme inhibition Strong Minimal No
Adverse CNS effects: Frequent Less frequent Less frequent
Duration of action: 8h 12 h 24 h
Dose: 800 mg/day for 300 mg/day for 20 mg/day for
6-8 weeks then 6-8 weeks then 6-8 weeks then
400 mg/day for 150 mg/day for 10 mg/day for
6-8 months 6-8 months 6-8 months

 
  255
3. Pro
oton pum
mp inhibiitors (PP
PIs):
(Omep
prazole – Lansopraz
L zole – Panttoprazole)

Chemistry: all arre imidazole derivativves.

Mecha
anism of action
a
 Theey are pro
odrugs. Th
hey are co
onverted
into
o the activee form in thhe gastric mucosa
andd produce e irreversiible inhib bition of
+ +
gasstric H /K ATPase
A ennzyme leadding to ↓
both basal an nd stimula ated HCl se ecretion to
o around th
he zero levvel for 1-2 days.
 Full restoratio
on of acid secretion n after stopping the PPI takess about 3--5 days
+ +
(tim
me of re-synnthesis of H /K ATP Pase).
 Theeir bioavaillability is decreased d significan
ntly by food and, id deally, shoould be
admministered 1 hour beffore a mea al.

Therap peutic usees: The saame as H2 blockers (dose of omeprazo


o ole: 20-40 mg/day
orally fo
or 4-6 wee
eks then 10
0-20 mg/d
day for 4-6 months to
o prevent re
recurrence)).

Advers
se effects
– Low
w incidenc
ce of diarrrhea, abdo
ominal colic, dizzine
ess, skin rrash, leuco
openia,
andd transient increase of o liver enzzymes.
– Deccrease vitt B12 abso orption aftter > 12 weeks
w of therapy
t duue to interrference
with
h intrinsic factor
f secrretion by th
he stomac ch.
– Inhiibition of gastric
g acid
dity leads to alteratiion of bioavialability ty of somee drugs
e.g.. ketocona azole, digox xin, and iro
on.
– Ommeprazole inhibits microsomall P450 enzymes and decreasses metabo olism of
pheenytoin, wa arfarin, and cyclosporrin. Newer PPIs
P do noot affect liveer enzymess.
– Ommeprazole in n high dos se induced d gastric ca
arcinoid tuumor in ratts.

█ ENH
HANCING MUCOSA
AL DEFENS
SE MECHANISMS

1. Suc
cralfate

 It iss an alumin
num salt off sulfated ssucrose.
 Slig m metal maay accumulate in
ghtly (3%) absorbed from the GIT. The aluminum
casses of renal failure, so
o it should be avoide
ed in renal failure.

Mecha
anism of action
a
 It n dic medium to be a
needs acid esence of aacidic med
activated. In the pre dium, it

 
256  
forms a complex with protein debris at the ulcer base and forms a physical
barrier (so not taken with antacids, H2 blockers, or PPIs).
 It ↓ pepsin secretion and ↑ secretion of endogenous PGs.

Adverse effects
N.B. Both sucralfate and
– Constipation (due to presence of aluminum).
bismuth compound are not
– ↓↓ absorption of tetracycline, digoxin and given simultaneously with
phenytoin. antacids or H2 blockers (at
least 30 min must be elapsed
in-between). Why?
2. Bismuth compounds:
Bismuth subsalicylate and subcitrate

Mechanism of action
 In acidic pH, it forms a complex with protein debris at the ulcer base and forms a
physical barrier.
 It ↓ pepsin secretion and ↑ secretion of endogenous PGs.
 It has additional antimicrobial activity against H. pylori.

Adverse effects
– Stool and teeth discoloration.
– Encephalopathy in presence of renal failure

Contraindications: Chronic renal failure and CNS diseases.

3. Carbenoxolone

It is a liquorice derivative having steroid structure.

Mechanism of action
 It ↑ production and viscosity of gastric mucus and ↑ mucosal resistance.
 It ↓ pepsin secretion and ↑ secretion of endogenous PGs.

Adverse effects
Salt & water retention (aldosterone-like effects) → edema and hypertension
especially in cardiac and renal patients. This edema can be treated by thiazide
diuretics (not by spironolactone) because both spironolactone and carbenoxolone
have steroid structure and can compete with each other.

Contraindications: Hypertension and/or renal failure

 
  257
4. Syn
nthetic PG
GE1 analo
ogue: Mis
soprostol

Mecha
anism of action
a
 It aacts on specific rece
eptors on gastric pa s to ↓ histtamine-stim
arietal cells mulated
HCll secretion
n.
 ↑m
mucus and bicarbonate secretio on (cytopro
otective acction).
 ↑m
mucosal blo ood flow an
nd stimulat
ates mucossal cellular regeneratiion.

Therap
peutic use
es
Preven
ntion of peeptic ulcerr in high risk patients e.g. th
hose on l ong term use of
NSAIDss for chronic inflamm
matory disseases. [mmisoprostool 200 g is combined with
xen or dicllofenac in single tab
naprox blet].

Advers
se effects
– Diarrhea and cramping pain: due tto ↑ GIT motility
m and water seccretion.
– Uteerine contraactions during pregnnancy → ab
bortion.

Contra ns: pregna


aindication ancy.

█ ERA
ADICATION THERAP
PY FOR H
H. pylori

 Infe
ection with H. pylori is
s a main c
cause of re e of PU.
ecurrence
 Thee following 10 days
“sequential proto-
p
col”” is highly effective
for e
eradicationn of H.
pylo
ori:

█ THE
ERAPY OF
F BLEEDIN
NG PEPTIC
C ULCER

 Hosspitalizatio
on and Fre
esh blood
d transfusion.
 Acid suppre ession witth high d
dose PPIs s by continuous i.vv. infusion is the
stan
ndard of ca
are e.g. om
meprazole 80 mg i.v. bolus follo
owed by 8 mg/h for 72h.
7
 Vita
amin K1: 10 mg i.m or
o s.c.
 Enddoscopic therapy:
t several
s type
es of endo
oscopic tre
eatments aare available.

 
258  
█ TREATMENT OF GASTROESOPHAGEAL REFLUX DISEASE (GERD)

Definition: reflux of gastric contents into the esophagus due to incompetent lower
esophageal sphincter (LES). Heartburn and chest pain are the major complain
which may be misdiagnosed of angina pectoris.

▌Non-drug therapy = life style modification


Drugs ↑ LES pressure:
 Head of bed elevation: because most damage to  Metoclopramide
the esophagus occurs at night when HCl can  Domperidone
 Bethanechol
remain in contact with the mucosa for long period.
 Erythromycin
 Weight reduction.
 Avoid: Drugs ↓ LES pressure:
– Stress, Smoking, Spices, alcohol.  Anticholinergic
– Ulcerogenic drugs e.g. NSAIDs. drugs
 Nitrates

▌Drug therapy

 Decreasing HCl secretion: H2 blockers and proton pump inhibitors.


 Prokinetic drugs.
 Antacids and antacid combinations: (Gaviscon).

▌Surgical treatment: if medical therapy failed.

█ PROKINETIC DRUGS

Definition: they are drugs that increase upper GIT motility and enhance gastric
emptying. They include:
■ Dopamine antagonists: e.g. Metoclopramide and Domperidone.
■ Serotonin (5-HT4) agonists: e.g. Mosapride
■ Cholinomimetic agents: e.g. Bethanechol.
■ Macrolide antibiotics: e.g. Erythromycin

1. Metoclopramide

Mechanism and pharmacological effects


 Metoclopramide ↑ LES tone and enhances gastric emptying and upper GIT
motility through:
 Blocking of dopamine (D) receptors (central & peripheral) leading to decrease
the inhibitory action of dopamine on the GIT motility.
 Enhances cholinergic transmission in the upper GIT.

 
  259
 N
N.B. Metoclopramide has no e
effect on small
s intesttinal or collonic motility.
 Anttiemetic action:
a du
ue to bloc
ckade of D2 recepttors in thee chemoreceptor
trigg
ger zone of
o the medu
ulla (CTZ)..

Therap
peutic use
es
 Gasstroesoph
hageal reflux: to enh
hance gastric emptyinng and ↑ LLES pressu
ure.
 Disorders off gastric emptying:
e e.g. diabetic gastro
oparesis aand postop
perative
gasstric retentiion.
 Beffore small bowel en ndoscopy (20 mg given by slo ow i.v.i.): to
o enhance
e gastric
eva
acuation an nd peristalttic movem
ment. Also to
t prevent vomiting.
 Beffore emerrgency su urgery and d labor to evacuate the stom mach and prevent
asp
piration of gastric
g con
ntents duri ng anestheesia.
 Tre
eatment off nausea anda vomitiing of vario ous causes.

Advers
se effects

– Sed
dation (the most commmon adve erse effect)).
– Extrapyramid s (e.g. dys
dal effects ystonia andd dyskines
sia): (especcially in old age)
due
e to blocka
ade of D2 in
n the basall ganglia.
– Hyp
perprolacttinemia du ue to blockkade of D2 in the pitu
uitary gland
d.

Drug in
nteraction
ns

– Antticholinerg pine) antag


gic drugs (e.g. atrop gonize its prokinetic
p action.
– Oth
her dopamine blo ockers (e e.g. antips
sychotic drugs) ad dministere
ed with
mettoclopramiide may prrecipitate a
acute extra
apyramidal effects.

2. Dom
mperidon
ne

Mecha
anism and pharmac
cological e
effects
 It blocks periipheral D2 receptorss leading to ↓ the inhibitory acction of do
opamine
on G
GIT motilityy. It does NOT
N crosss BBB so itt has no CNS
C side efffects.
 Anttiemetic efffect less than
t meto clopramide.

Therap
peutic use
es
 Thee same usees as meto
oclopramid
de.
 To counteracct nausea a and vo omiting caused by
y
levoodopa an nd bromocriptine d during tre
eatment off
Parrkinson’s disease because
b it blocks D2
2 receptors s
in th
he CTZ ressponsible for
f vomiting g but does
s not blockk
D2 receptors in
i the basaal ganglia re
esponsible
e for parkins
sonism.

 
260  
Adversse effectss: there is growing
g at domperidone may ↑ QT interrval and
evvidence tha
predisp
pose to seriious arrhyth
hmia and sudden dea ath.

M
Metoclopra
amide D
Domperido
one
Dopam
mine recepttor blockad
de C
Central and
d periphera
al P
Peripheral only
o
Choline
ergic transsmission In
ncrease N
No effect
Antiem
metic effect S
Strong W
Weaker
Extra-p
pyramidal side
s effects P
Present N
No
Hyperp
prolactinem
mia S
Significant M
Minimal

3. Betthanechol

Bethannechol stim mulates mu uscarinic MM3 in the smooth


s ms of the GGIT and my yenteric
plexus.. It was ussed in the past for tthe treatmment of GE ERD and g gastroparesis, but
now, it is rarely used for this indicatio
on due to multiple
m cholinergic sside effects
s.

4. Mac
crolide an
ntibiotics
s: erythro
omycin

Macrolide antibiootics such as erythro omycin dirrectly stimulate mottilin receptors on


GIT sm mooth mu uscle and promote e the onset of a migrating motor co omplex.
Intravenous erythhromycin (33 mg/kg) iss beneficia
al in some patients w
with gastrop
paresis;
howeve er, tolerance rapidly
y developss. It may beb used in n patients with acutee upper
GIT hemmorrhage to promote e gastric e
emptying of
o blood be efore endosscopy.

█ ANT
TACIDS AN
ND ANTAC
CID-ALGIN
NIC ACID PRODUC
CTS

Gavisc
con: (algin
nic acid + Mg-trisilica
M ate + Al-hy
ydroxide +
NaHCOO3):

Alginic acid in prresence off saliva an


nd NaHCO O3 forms a
highly vviscous fo
oamy soluttion of Na--alginate that
t floats
on the gastric co ontents as
s a raft an nd prevents gastric
reflux.

█ H2 B
BLOCKER
RS AND PP
PIS

 PPIs are moree effectivee than H2 blockers foor the treattment of G


GERD.
 Oncce-daily do
osing of PPPIs provide
es effective
e symptom m relief and
d tissue he
ealing in
85––90% of paatients; up to 15% off patients require
r twice-daily doosing.

 
  261
Part 2
2: Ma
anageme
ent of L iver Dis
sease Co
omplica
ations

█ MAN
NAGEMEN
NT OF HEP
PATIC EN
NCEPHALO
OPATHY

Definittion and pathogenesis


Hepatic c encephalopathy is the
syndrome of disordered
d con-
sciousn ness and neuromu uscular
activityy seen in n patients s with
acute o or chronic liver failure
e.
The failing liver cannot metabo-
m
mmonia and
lize am a benzoodiaze-
pines-llike med diators (G GABA-
ansmitters) generatted by
like tra
the inntestinal bacteria. These
toxins a
are shunte S causing encephalo
ed directly to the CNS opathy.

gement: Treatment
Manag T is aimed a
at reductio
on of hype
erammone
emia:

■ Diet:
– Protein reestriction to
t decreasse formatio on of ammo onia by inttestinal bac
cteria.
– VVegetablee protein is
s better tole
erated tha
an animal protein.
p
– TThe rationa
ale and beenefit of die
etary prote
ein restriction is conttroversial.

■ Ene
emas: clea
ansing of the colon n is a rap
pid and efffective meethod to remove
moniagenic substrates. It can b
amm be done with
w lactulosse or tape water.

■ Lac
ctulose:
– It is syntheetic non-ab bsorbable disaccharride. In the
e colon, it iis transformed by
bacteria intto lactic an acids → ↓ pH
nd acetic a p of the co
olonic med dium leadin ng to:
– Inhibitio
on of intes eria → ↓ pro
stinal bacte oduction of
o ammoniaa.
– ↑ transsport of ammonia from blood to intestinal lum men wherre it is
converrted to the poorly abssorbed am
mmonium io
on.
– Osmotic laxation → ↑ excrettion of ammonium io
on.

– It is admin
nistered ora
ally or as eenema (for patients in
n coma).
– A
Adverse effects:
e rellatively saffe drug.

■ Ora
al antibiotiics:
Neo
omycin:
– It is non-absorbable aminoglyccoside antibiotic.

 
262  
– It ↓ blood ammonia by killing intestinal bacteria that generate ammonia.
– It is used in a dose of 1-2 g 4 times daily orally or as retention enema.
– Small amounts of neomycin may be absorbed (~1%) and result in ototoxicity
and nephrotoxicity especially in patients with renal impairment.

Other antibiotics:
– Metronidazole acts on anaerobic bacteria. It is the preferred option if there is
fear from adverse effects of neomycin (but given for short term).
– Rifaximin: is non-absorbable and better tolerated antibiotic.

█ MANAGEMENT OF VARICEAL BLEEDING DUE TO PORTAL HYPERTENSION

▌Management of acute bleeding

■ Fresh blood transfusion.


■ Acid suppression with omeprazole (80 mg) to minimize HCl irritation.
■ i.v. vasopressin or its analogues:

 Vasopressin:
– It produces mesenteric VC leading to  portal venous flow and pressure.
– It can produce systemic VC (coronary, cerebral, limb, etc), so it is better
combined with i.v. nitroglycerine to reduce systemic and coronary VC.
– The vasopressin/nitroglycerine combination is rarely used now.
 Terlipressin:
– It is synthetic analog of vasopressin that is released in a slow and sustained
manner allowing more sustained hemodynamic effects with fewer systemic
side effects than vasopressin.

■ Prophylactic antibiotics: to prevent infectious complications after GI


hemorrhage. The preferred antibiotic is i.v. ceftriaxone 1 gm/day for 7 days.

■ Endoscopic sclerotherapy: injection of the varices with a sclerosing agent to


induce fibrosis and obliteration.

▌Prevention of re-bleeding (prophylaxis):

 Beta-blockers (propranolol 40 mg twice daily). It ↓ portal BP through:


– They ↓ COP → ↓ portal blood flow.
– They cause unopposed α- action → VC of the splanchnic vascular bed.
 H2 blockers or PPIs: to prevent gastroduodenal erosions.
 Metoclopramide: to enhance gastric evacuation and  LES pressure.

 
  263
264
Part 3: Antiemetic Drugs

Several classes of antiemetic drugs are available that antagonize the neurotransmitter receptors known to be involved in the
physiology of nausea and vomiting. The antiemetic drugs are classified according to their primary action; some agents affect
multiple receptors.

Drug Antiemetic mechanism Uses as antiemetic Adverse effects

1. Muscarinic blockers: They block M1 receptors in the Prevention and treatment of  Blurred vision
 Atropine vestibulocerebellar pathway, vomiting due to motion sickness  Dry mouth
 Hyoscine solitary tract nucleus, and (0.3-0.6 mg/8 hrs orally).  Urine retention
chemoreceptor trigger zone  Glaucoma
(CTZ).  Tachycardia

2. H1-blockers:  They block H1 (also M1)  Vomiting due to motion  Sedation (excitation may
 Diphenhydramine receptors in the vestibulo- sickness (diphenhydramine) occur in children).
 Cyproheptadine cerebellar pathway and CTZ.  Vomiting of pregnancy  Atropine-like actions (dry
 Cyclizine  They have sedative action. (cyclizine and meclizine) mouth, blurred vision, urine
 Meclizine  True vertigo: combined with retention).
VDs to improve labyrinthine  Hypotension
blood flow.

3. 5-HT3 blockers: They block 5HT3 receptors in  Vomiting due to cancer  Dizziness, headache, and
 Ondansetron the GIT, solitary tract nucleus chemotherapy or radio- constipation.
 Granisetron and CTZ. therapy.
 Tropisetron  Postoperative nausea and
vomiting.

4. Dopamine blockers  They block D2 receptors in  Vomiting due to drugs or  Sedation


 Metoclopramide the CTZ. fevers.  Extrapyramidal effects e.g.
 Domperidone  They inhibit peripheral trans-  Vomiting due to cancer dystonia and dyskinesia.
 Phenothiazines e.g. mission to VC. chemotherapy.  Hyperprolactinemia
chlorpromazine  Postoperative nausea and  Postural hypotension
vomiting.

5. Cannabinoid  It is a partial agonist at  Vomiting due to cancer  Sedation


derivatives: central and peripheral chemotherapy  Hallucinations
 Nabilone and cannabinoid receptors (CB1).  Patients refractory to other  Psychotropic effects
Dronabinol The exact mechanism is antiemetics.  Postural hypotension
unclear.  Drug abuse.

6. Vitamin B6 (pyridoxin) May be related to the balance  Vomiting in pregnancy (50 mg -


between GABA (CNS inhibitory at bedtime).
transmitter) and glutamate  Vomiting in children
(CNS excitatory transmitter).

7. Corticosteroids The exact mechanism is  Combined with Vit B6 to treat  See endocrine chapter
 Dexamethasone unclear. vomiting in pregnancy.
 Prednisolone  Vomiting due to cancer
chemotherapy.

8. Benzodiazepines Allosteric facilitation of central  Stress-related vomiting  See CNS chapter


 Lorazepam GABA inhibitory transmission  To controls symptoms in
 Diazepam Ménière disease

9. Neurokinin-1 receptor Substance-P induces vomiting  In combination with 5-HT3  Diarrhea and fatigue
blockers: through stimulation of NK-1 blockers to treat vomiting due
 Aprepitant receptors. Aprepitant blocks to cancer chemotherapy
this receptor.

265
Part 4: Antispasmodic Drugs (smooth ms relaxants)

Classification

■ Anticholinergic drugs: atropine, hyoscine, propantheline, oxyphenonium.


■ Direct smooth muscle relaxants: papaverine, mebeverine, alverine,
drotaverine.
■ Mixtures: Librax (clidinium + chlordiazepoxide), Donnatal (hyoscine + phenobarbital).

Mebeverine, Alverine,
Papaverine Librax
Drotaverine
It is opium alkaloid but They are synthetic It is a combination of:
Chemistry & mech-

 Chlordiazepoxide:
anism of action

chemically different drugs


from morphine benzodiazepine that
has antianxiety effect.
The exact mechanism is unclear but may be due
 Clidinium: anticho-
to inhibition of PDE enzyme  cAMP 
linergic drug which ↓
smooth muscle relaxation.
GIT motility and spasm

 Spasms of the GIT, bile duct and genitourinary tract.


Use

 Librax is used for treatment of irritable bowel syndrome (IBS).

– Cardiac arrhythmia. – Atropine-like actions


Side effects

– Abnormal liver functions in the form of elevated e.g, dry mouth, urine
serum transaminases and alkaline retention, etc.
phosphatase. – Sedation, drowsiness,
– Headache and dizziness confusion, etc.

– Paralytic ileus.
C/I

– Constipation for more than one week

Part 5: Therapy of Constipation

▌Non-drug therapy: It is the first line in all cases of constipation


 Diet rich in fibers e.g. fruits, vegetables, whole meal bread, etc. to be increased to 30
g/day.
 Increase fluid intake
 Minimize tea and coffee.
 Physical exercise to activate abdominal muscles and intestinal peristalsis. This
help food move more efficiently through the gut.

 
266  
▌Drug therapy: LAXATIVES:
Drug causes of constipation:
1. Bulk-forming agents:
 Atropine and related drugs.
[Dietary fibers – Methylcellulose – Bran]
 Aluminum containing antacids
Mechanism of action  Adsorbents (kaolin & pectin).
 CCBs: e.g. Verapamil
They are non-digestible fibers; they retain  Opioids: morphine &
water in the gut and distend the large loperamide
intestine → activation of stretch receptors →
stimulation of peristalsis.

Adverse effects: they are safe laxatives but may cause:


– Bloating and abdominal distension.
– ↓ absorption of some drugs e.g. digoxin.
– They may form masses in the gut leading to intestinal obstruction.

2. Osmotic laxatives:
[Mg sulfate & Na salts – Lactulose – Polyethylene glycol]

Mechanism of action
They are retained in the gut lumen and retain water by their osmotic effect →
activation of stretch receptors → stimulation of peristalsis.

Adverse effects
– Mg & Na salts (saline laxatives) may be absorbed systemically and produce
hypermagnesemia and hypernatremia especially in patients with renal failure.
– Lactulose may produce abdominal discomfort.
– Polyethylene glycol may produce electrolyte disturbance (hypokalemia).

3. Irritant (or stimulant) laxatives:


[Castor oil – Senna – Bisacodyl]

Mechanism of action
They produce inflammation (irritation) of the intestinal mucosa and inhibit Na+/K+
ATPase enzyme leading to:
– Accumulation of water and electrolytes in the gut lumen.
– Direct stimulation of peristalsis by their irritant effect.

Adverse effects

 
  267
Castorr oil – Bad taste
e.
– Stimulatio
on of uteri ne contrac
ction and abortion
a
Senna – It passess in urine annd cause urine
u discoloration
– It passess in breast milk and cause
c hartic effecct in the ba
cath aby.
– Prolonged d use → degeneration of gutt nervous plexus → atonic
(cathartic)) colon.
– Increase menstrual blood flow w and abo ortion in prregnancy.
– Laxative depend dence: Irrritant lax xatives ccause co omplete
evacuatio on of the c colon. The
e colon requires 2-55 days beffore the
normal fe ecal masss can be reestablish hed. The patient be ecomes
worry reg garding th he lack of bowel mo ovement d during thiss period
and may use the la axative agaain and a vicious cyycle is esta
ablished
leading too partial orr complete
e loss of no
ormal boweel functionn.
Bisaco
odyl – It is prepared as ennteric coated tablets to avoid ggastric irrittation. If
it is given
n with milkk or with otther drugs that chang ge gastric pH, the
enteric coating
c maay dissolve e in the stomach
s aand cause gastric
irritation and
a pain
– Prolonged d use → degeneratiion of gutt nervous plexus → atonic
(cathartic) colon (shoould not bee used morre than 10 ddays).

4. Sto
ool softeners: Do
ocusate so
odium

Mecha anism: theyy are anion


nic surfacttants that enable
e add
ditional waater and fatts to be
incorpo
orated in th
he stool, making
m it ea
asier to moove throug
gh the GIT.

5. Lub
bricant laxatives
s:
[Liquid
d paraffin – Glycerin
n supposittories – Ev
vacuant en
nema]

anism of action
Mecha a
– Parraffin oil it coats th
he fecal m
matter and
reta
ards water absorption by the co olon.
– Glyycerin has hygrosco opic effect . It draws
watter from rectal muc cosa and lubricates
the anal can nal. It also stimulattes reflex
recttal contra
actions an nd promo otes stool
evaacuation in 15-20 minn.

se effects:: paraffin oil


Advers o decreasses absorption of fat--soluble vittamins

 
268  
6. Chloride channel activators: Lubiprostone

Mechanism of action
It acts by activating chloride channels to increase fluid secretion in the intestinal lumen.
This eases the passage of stool and causes little change in electrolyte balance.

General indications of laxatives


 Constipation: laxatives should not be used for prolonged duration to avoid
laxative dependence.
 To fasten excretion of toxic substances from the GIT.
 To prepare the bowel before X-ray or colonoscopy.
 Hepatic encephalopathy (lactulose): to kill ammonia producing bacteria.
 Painful anal conditions e.g. anal fissure or piles.
 Postoperative: e.g. after hemorrhoids (piles) to avoid strain.

Contraindications of laxatives
 Laxatives are dangerous in cases of undiagnosed abdominal pain or
inflammatory bowel disease. They may lead to intestinal perforation.
 Organic obstruction of the GIT.

Part 6: Therapy of Diarrhea

▌Causes of diarrhea

■ Infectious diarrhea: bacterial, viral, fungal, protozoal, etc. Infectious diarrhea is


the most common type.

■ Hormonal (secretory) diarrhea: e.g. serotonin in carcinoid syndrome; calcitonin in


medullary thyroid carcinoma, histamine in mastocytosis, thyroxine in hyperthyroidism.
Diarrhea is caused by: (1) increased water secretions into the intestinal lumen
and (2) increased intestinal motility.

■ Malabsorption syndromes: e.g. bile acid malabsorption

■ Inflammatory (exudative) diarrhea: caused by inflammatory bowel diseases


e.g. Crohn’s disease (transmural lesion in the small intestine) and ulcerative
colitis (ulceration of the colon with bloody diarrhea).

■ Iatrogenic (drug induced) diarrhea:


– Overuse of laxatives.
– Mg-containing antacids.

 
  269
– Antibiotic--associated
d diarrhea (pseudome s colitis) seee chemoth
embranous herapy
– Cholinomimetic drug gs.

▌Patte
erns of dia
arrhea

■ Acu
ute self-limmited diarrrhea: acutte diarrhea
a disappeaars within 224 hrs.
■ Acu
ute diarrhea (<2 we eeks): passsage of watery
w stool more thhan 10 tim
mes/day
associated wiith dehydra ation and e
electrolyte imbalancee.
■ Chrronic diarrrhea: (>2 weeks): persistent diarrhea for 3 weeeks in adults or 4
weeeks in infan
nts. It caus
ses weightt loss and weakness.
w

▌Inves
stigations of diarrhe
ea
 Sto nation: – Macrosc
ool examin copic: con
nsistency, color,
c bloo
od, etc.
– Microsccopic: RBC Cs, WBCs, parasites, ova, etc.
– Stool cuulture and sensitivity tests.
 End
doscopy and
a biopsy y in chronic
c cases.
 Rad
diologic ex
xamination
n: by bariu m enema.

▌Trea
atment of diarrhe
ea

Lines o
of therapy
y

 Maiintenance e of fluid and


a electro olyte balannce: is the
e first priorrity.
 Non n-specific
c antidiarrheal agen nts: should
d be applied after eexclusion of
o other
rele
evant causees of diarrh
hea.
 Speecific antid
diarrheal agents:
a tre
eatment off the causee e.g.
 Antimic crobials for infectiouss diarrhea..
 Antiinflammatory y drugs for inflammattory bowel diseases.
 Anttispasmod dic drugs: if there is colic or abbdominal cramps.
c

I. MAIN
NTENANCEE OF FLUID
D AND
ELECTR ROLYTE BALANCE

■ Oraal Rehydra ation Therrapy (ORT)):


 Balaanced salts solu
ution con ntaining
elecctrolytes and glucose (gluc cose is
impportant for sodium and
a conse
equently
watter absorpttion).
 90%% of acute cases of childhood
c d
diarrhea
can be correctted using ORT
O only.
■ Intrravenous solutions:: if dehydrration is
sevvere.

 
270  
N.B. in
n infants with
w dehydrration, blo ood pH, se erum Na+ and K+ m must be meeasured
before giving anyy i.v. solutiion to avoiid electroly
yte and acid/base im
mbalance.

II. NON
N-SPECIFIC
C ANTIDIAR
RRHEAL TH
HERAPY

1. Ads
sorbents:: Kaolin, pectin, act ivated cha
arcoal

Mecha
anism
 The
ey adsorb water,
w microoganism
ms and toxiins.
 The
ey coat the
e mucosa and
a protec ct it.

Advers y ↓ absorpttion of othe


se effects:: they may er drugs.

2. Bism
muth sub
bsalicylatte

Mecha
anism
 Bismmuth: provvides a pro oat for the
otective co e mucosa anda bindss toxins prroduced
by ppathogenic c bacteria.
 Sub bsalicylate:: hydrolyze
ed by inte
estinal bactteria into salicylic
s acid →↓ in
a ntestinal
infla
ammation, hypermottility and se ecretions.

3. Antti-choline
ergic drug
gs: atropin
ne, hyosciine and prropantheliine

Mecha
anism
 Antidiarrheal action: ↓ colonic
c peeristalsis by
b blockingg the respponse of in
ntestinal
smo
ooth musc cle to cholin
nergic stim
mulation.
 Antispasmodiic action: relieve
r cram
mps assoc ciated with diarrhea.

4. Syn
nthetic op
pioid prep
parations
s: dipheno
oxylate and
d loperam
mide

Mecha
anism
 The
ey act on opioid
o mu) and δ (delta) rec
μ (m ceptors in the entericc nervous system
(both pre- and
d postsyna
aptic) lead ing to:
 ↑ segm menting (no
on-propuls ive) contra
actions of the
t small inntestine.
 ↑ waterr absorptio
on and ↓ wwater secreetion by intestinal muucosal cells
s.
 ↓ Ach release
r by cholinergi c neurons in the ENSS.

 Lop
peramide cannot
c crross BBBB while dipphenoxylatte can cro
oss BBB in very
small amoun nt (no CN NS effects in usual therapeutiic doses) but it can
n cause
add
diction if ussed in large doses an
nd for prollonged durration.
 The
ey are com
mmonly combined w omotil® is a combination of
with atropiine (e.g. Lo

 
  271
diphenoxylate 2.5 mg + atropine 0.25 mg) to produce more ↓↓ in intestinal motility
and decrease liability for abuse.

Adverse effects
 Anti-cholinergic side effects e.g. dry mouth.
 Addiction: if used for prolonged duration.
 Precipitation of toxic megacolon if used in ulcerative colitis.

5. Cholestyramine

Mechanism: it binds bile acids in the intestine preventing their absorption and
decreasing their irritation.

Therapeutic uses: diarrhea due to bile salt malabsorption.

III. SPECIFIC ANTI-INFECTIVE AGENTS

 It is not necessary in simple gastroenteritis as most cases are viral in origin.


 Chemotherapy is necessary in specific types of enteritis e.g.

C. difficile colitis Metronidazole 400 mg/8h orally for 10 days (1st choice).
Clostridium difficile If no response to metronidazole: vancomycin 250
mg/6h.
Campylobacter jejuni Azithromycin 500 mg/day orally for 3 days.
E. coli (enterotoxigenic Ciprofloxacin 500 mg/12h for 1-3 days
and enteropathogenic)
Non-typhoid Salmonella Ciprofloxacin 500 mg/12h for 5-7 days or ceftriaxone
spp. 1g IV/24h.
Shigella spp. Ciprofloxacin 500 mg/12h for 3-5 days or co-
trimoxazole 2 tab (80/400)/12h.
V. cholerae Doxycycline 300 mg orally single dose or ciprofloxacin
500 mg/12h orally for 3 days.
V. parahemolyticus Ciprofloxacin 500 mg/12h for 1-3 days

IV. ANTI-INFLAMMATORY DRUGS

1. Sulfasalazine (5-aminosalicylic acid "5-ASA" + sulfapyridine)

Mechanism: In the colon, the “azo” bond is broken by intestinal bacteria to release 5-
ASA and sulfapyridine: 5-ASA has anti-inflammatory and immunosuppressive, while
sulfapyridine is poorly absorbed sulfonamide with antibacterial action.

 
272  
Therapeutic uses:
 Active ulcerative colitis (3-4 gm/day) and to maintain remission (2 gm/day).
 Rheumatoid arthritis.

Adverse effects: mainly due to sulfapyridine (sulfonamide) → drug allergy, bone


marrow depression and megaloblastic anemia (avoided by giving folic acid).

Other aminosalicylates:
 Mesalazine: modified-release preparation.
 Olsalazine is a dimmer of 5-ASA. It is cleaved in the lower bowel to release 5-ASA.

2. Corticosteroids
Travellers’ diarrhea:
Mechanism
Diarrhea that occurs to
 Stimulation of Na+ absorption from the travellers and tourists in high
intestine. endemic areas. Transmission of
 Anti-inflammatory action (see endocrine). infection is done through
contaminated food or water.
Therapeutic uses: they are given orally or as Drug therapy:
enema in severe cases.  Prophylaxis: doxycycline
(100 mg /day orally)
 To control acute episodes of Inflammatory
 Treatment: doxycycline +
bowel diseases
bismuth subsalicylate +
 Refractory diarrhea unresponsive to other fluids
agents.

3. Immunosuppressive agents

 Cytotoxic drugs (azathioprine and 6-mercaptopurine): can be used in


ulcerative colitis and Crohn’s disease in small dose to avoid bone marrow
depression.
 Cyclosporine-A use is limited for severe cases due to potential toxicity.

N.B.
 Metronidazol may be used in Crohn’s disease to eradicate anaerobic bacteria.
 Infliximab (monoclonal antibodies): can be used in Crohn’s disease to ↓ TNFα.
 Aspirin and indomethacin may of value in acute diarrhea because they ↓ PGs
synthesis → ↑ absorption and ↓ secretions of intestinal fluids.
 Clonidine (α2 stimulant) can be used in diabetic diarrhea to ↑ intestinal water
absorption and ↓ electrolyte secretion.

 
  273
Part 7: Drug Therapy of Gallstones (cholelithiasis)

Choleretics: are agents that increase bile production e.g. bile acids and bile salts.
Hydrocholeretics: are agents that increase volume of bile e.g. dehydrocholic acid.
Cholagogues: are agents that stimulate the evacuation of gall bladder e.g. olive oil.

The following drugs are used to dissolve non-calcified cholesterol stones:

Chenodeoxycholic acid (CDCA): It ↓ hepatic cholesterol synthesis and ↑ bile acid


and phospholipid synthesis.

Ursodeoxycholic acid (UDCA)


 It is a metabolite of chemodeoxycholic acid.
 It is more potent in reduction of hepatic cholesterol synthesis without affecting
the endogenous bile acid synthesis.
 Both CDCA and UDCA are given orally in a dose of 10-15 mg/kg/day for 6-12 month.
 Diarrhea is common side effect with CDCA but is unusual with UDCA.

 
274  
 

275
Review Questions

1. Classify laxatives. Mention the mechanism of action of each class.


2. Mention the mechanism of action and adverse effects of each of the following:
– Metoclopramide.
– Cimetidine
– Omeprazole
– Senna
– Lomotil
3. Mention the mechanism of action and therapeutic uses of:
– Papaverine
– Sulfasalazine
– Terlipressin
4. Mention the drug-drug interactions of metoclopramide.
5. Mention 2 drugs used for the treatment of nausea and vomiting due to the
following conditions. For each drug, mention the mechanism of action and the
main adverse effects:
– Vomiting due to cancer chemotherapy
– Motion sickness
6. Give the reason for:
– Atropine is combined with diphenoxylate in Lomotil preparation.
– Domperidone, and not metoclopramide, is used for treatment of vomiting due
to levodopa.
– Irritant laxatives should not be given for long period.
– Folic acid should be supplemented with sulfasalazine during treatment of
ulcerative colitis and rheumatoid arthritis.
7. Mention 3 differences between:
– Metoclopramide and domperidone.
– Cimetidine and omeprazole.

8. Mention the mechanism and management of portosystemic encephalopathy.


9. Mention the essential lines of the treatment of bleeding esophageal varices.

 
276  
Of each of the following questions, B. Sulfasalazine.
select ONE BEST answer: C. Sulfapyridine.
D. Sulfamethoxazole.
1. Toxicities of cimetidine include E. Salicylate sodium
which one of the following?
A. Blurred vision 7. An important drug in the therapy of
B. Diarrhea portal systemic encephalopathy is:
C. Orthostatic hypotension A. Lactulose.
D. P450 inhibition B. Lactate.
E. Sleepiness C. Loperamide.
D. Lorazepam.
2. Which of the following will result E. Doxapine.
from blockade of H2 receptors?
A. Decreased cAMP in cardiac muscle 8. Bismuth salts are thought to be
B. Increased cAMP in cardiac muscle effective in peptic ulcer disease
C. Decreased IP3 in gastric mucosa because they have bactericidal
D. Increased IP3 in gastric mucosa properties against:
E. Increased IP3 in smooth muscle A. Escherichia coli.
B. Bacteroides fragilis.
3. Which of the following is most C. Clostridium difficile.
effective in the treatment of peptic D. Helicobacter pylori.
ulcer disease? E. Staphylococcus aureus.
A. Cimetidine
B. Lanzoprazole 9. Misoprostol has a cytoprotective
C. Pirenzepine action on the gastrointestinal mucosa
D. Ondansetron because it:
E. LSD A. Enhances secretion of mucus and
bicarbonate ion.
4. A 55-year-old woman with insulin B. Neutralizes acid secretion.
dependent diabetes of 40 years’ C. Antagonizes nonsteroidal anti-
duration inflammatory drugs (NSAIDs)
complains of severe bloating and D. Relieves ulcer symptoms.
abdominal distress, especially after E. Coats the mucosa.
meals. Evaluation is consistent with
diabetic gastroparesis. The drug you 10. The primary pharmacologic action
would be most likely to recommend is: of omeprazole is the reduction of:
A. Docusate A. Volume of gastric juice.
B. Dopamine B. Gastric motility.
C. Loperamide C. Secretion of pepsin.
D. Metoclopramide D. Secretion of gastric acid.
E. Sucralfate E. Secretion of intrinsic factor.

5. The steatorrhea of pancreatic 11. Cimetidine slows the metabolism of


insufficiency can best be treated by: many drugs because it inhibits the
A. Cimetidine activity of:
B. Misoprostol. A. Monoamine oxidase (MAO)
C. Bile salts. B. Tyrosine kinase.
D. Pancreatic lipase. C. Hydrogen - potassium - adenosine
E. Secretin triphosphatase (H+, K+, ATPase).
D. Phase II glucuronidation reactions
6. A drug of choice in the therapy of E. Cytochrome P450.
inflammatory bowel disease is:
A. Sulfadiazine.

277
12. The absorption of iron is reduced D. A potent anti-anderogenic action.
when large and prolonged doses of E. None of the above.
which of the following drugs are given?
A. Vitamin C 18. Radiation-induced vomiting can be
B. Alum hydroxide. treated by drugs that act on:
C. Aspirin. A. 5-HT3 receptors.
D. Cimetidine. B. M1 receptors.
E. Lactuolse. C. H1 receptors.
D. Alpha receptors
13. Omeprazole, an agent for the E. Beta receptors.
promotion of healing of peptic ulcers,
has a mechanism of action that is 19. Cholesterol gallstones may be
based on: dissolved by oral treatment with:
A. Prostaglandins. A. Lovastatin.
B. Gastric secretion. B. Dehydrocholic acid.
C. Pepsin secretion. C. Methyl teriary butyl ether.
D. H+, K+, ATPase. D. Chenodeoxycholic acid.
E. Anticholinergic. E. Monoctanoin.

14. An effective antidiarrheal agent 20. A patient who must take verapamil
that inhibits peristaltic movement is: for hypertension and angina has
A. Clonidine. become constipated. Which of the
B. Bismuth subsalicylate. following drugs would be most suitable
C. Oral electrolyte Solution. as a long term laxative?
D. Pectin. A. Aluminum hydroxide
E. Diphenoxylate. B. Diphenoxylate
C. lactulose
15. The approved indication for D. Metoclopramide
misoprostol: E. Mineral oil
A. Reflux esophagitis.
B. Regional ileitis. 21. Your cousin is planning a three-
C. Ulcerative colitis. week trip overseas and asks your
D. Prevention of gastric ulceration in advice regarding medications for
patients using large doses of aspirin traveler's diarrhea. A drug suitable for
like drugs. non-infectious diarrhea is
E. Pathologic hypersecretory conditions A. Aluminum hydroxide
such as Zolinger- Ellison syndrome. B. Bismuth subcitrate
C. Magnesium hydroxide
16. Metoclopramide has antiemetic D. Metoclopramide
properties because it: E. Mineral oil
A. Lowers esophageal sphincter
pressure. 22. Which of the following drugs or
B. Is a central dopamine- receptor drug groups is not useful in the
antagonist. prevention of nausea and vomiting
C. Is a central opioid receptor agonist induced by cancer chemotherapy:
D. Has cholinomimetic properties. A. Dexamethasone
E. Has sedative properties. B. Dronabinol
C. Scopolamine
17. Fomatidine has the following D. Ondansetron
properties: E. Metoclopramide
A. A potent proton pump inhibitor.
B. A potent antiemetic agent.
C. A potent inhibitory effect on cyt P450.

278
23. A patient presents with Zollinger- 28. Bisacodyl frequently can cause:
Ellison syndrome due to a gastrinoma. A. Abdominal cramps
He has two bleeding ulcers and B. Constipation
diarrhea. A drug that irreversibly C. Skin rashes
inhibits the H+/K + ATPase in gastric D. Dizziness
parietal cells is E. Nauseas
A. Cimetidine
B. Cisapride 29. Patients with acute bleeding due to
C. Glycopyrolate ruptured esophageal varices could be
D. Omeprazole managed by:
E. Ondansetron A. i.v. terlipressin
B. i.v. sodium bicarbonate
24. A drug that is recently linked with C. Oral lanzoprazole
some cases of cardiac arrhythmias and D. i.v. hydrocortisone
sudden death is: E. Oral lactulose
A. Aluminum hydroxide
B. Lactulose 30. A patient being cared for by the
C. Loperamide gastroenterology service is being
D. Ranitidine treated with sulfasalazine. Which of the
E. Domperidone following is the most likely purpose for
which it is being given?
25. One recognized advantage of A. Antibiotic-associated
domperidone over metoclopramide as pseudomembranous colitis
a prokinetic agent is: B. E. coli–induced diarrhea
A. More prominent antiemetic action C. Gastric H. pylori infections
B. More powerful stimulant of GIT motility D. Inflammatory bowel disease
C. Less CNS adverse effects. E. NSAID-induced gastric ulcer
D. Less incidence of diarrhea prophylaxis
E. Less cardiac adverse effects

26. A cannabinoid receptors agonist


that is useful for prevention of nausea Answers
and vomiting due to cancer
chemotherapy is: 1D 7A 13 D 19 D 25 C
A. Dronabilone 2A 8D 14 E 20 C 26 A
B. Morphine 3B 9A 15 D 21 B 27 B
C. Diphenoxylate 4D 10 D 16 B 22 C 28 C
D. Naloxone 5D 11 E 17 E 23 D 29 A
E. Ondansetron 6B 12 B 18 A 24 E 30 D

27. A pregnant woman with 28 weeks


gestation complaining of distressing
constipation. Which of the following
drugs can be prescribed safely?
A. Ondansteron
B. Lactulose
C. Senna
D. Magnesium sulphate salt
E. Castor oil

279
 

280
   
 
 
 
Part 1
1: Diab
betes Me
ellitus a
and Antid
diabetic
c Drugs

Definittion of DM M: It is a clinical synd drome


charactterized by disturbance in carbo ohyd- Histolog
gy

rates, ffat, & pro oteins metabolism d due to There are


e 4 types off cells in the
e islets
either in ciency or ins
nsulin defic sulin resistaance. of Lange
erhans.

 α cellss (20%) : seecrete glucagon.


Manife
estations:  β cellss (75%) : seecrete insullin and
 Polyyuria, polyydipsia, polyphagia
p , and islet as
ssociated p polypeptide
losss of weightt. (amylinn).
 Com mplicationss of DM e.g. rec urrent  δ cellss (3-5%) : ssecrete
infe
ections, angiopathy
a y, retinoppathy, somattostatin.
nepphropathy, DKA, etc.  F cellss < 2 %: seecrete panc creatic
polype eptides (faccilitate dige
estive
proces ss).
Laboraatory invesstigationss:
 Urinne analysiss for glucosuria & kettonuria.
 Bloood glucose e: fasting and
a 2-hr affter oral 75
5 g glucose
e.
 Glyccated hemmoglobin (H HbA1C): in
ndicates avverage blo
ood sugar level for the past
2-3 months
Norm
mal IGT* DM
Fasting
g BG < 100
0 mg/dl 100-126 > 1226 mg/dl
2 hr aftter oral 75 g glucose < 140
0 mg/dl 140-200 > 2000 mg/dl
HbA1C < 6% 6-6.5% > 6.55%
Elevvated levels
s mean
unco ontrolled DM over
the llast 2-3 mo
onths.

* IGT = Impaired glucose tolerance


t es): it can be controlleed by diet regime
((prediabete
alone.

 
  281
Common types of DM:
 Type 1 DM: (old name: insulin dependent DM)
 Type 2 DM: (old name: non-insulin dependent DM)
 Gestational DM: appears during pregnancy and disappears after labor.

Type 1 (IDDM) Type 2 (NIDDM)


Pathophysiology: Autoimmune condition The pancreas usually
against the pancreatic beta produces some insulin. But
cells. The damaged either the amount is not
pancreas doesn't make sufficient, or the body's
insulin. cells are resistant to it.
Onset (age): < 30 years >30 years (adult-onset)
Body built: Usually undernourished Usually obese
Insulin therapy: Essentially required. Required in 20% of patients
Oral drugs: Not required. Essentially required.

Less common types of DM:


 Latent autoimmune diabetes of adults (LADA): is a condition in which type 1
DM develops in adults.
 Maturity onset diabetes of the young (MODY): is a condition in which type 2
DM develops in children or young adults who develop persistent, asympt-
omatic hyperglycemia without progression to diabetic ketoacidosis.
N.B. Adults with LADA or children with MODY are frequently misdiagnosed as
having type 2 or type 1 DM, based on age rather than etiology.
 Secondary DM: 2ry to drugs, infections, endocrine diseases, etc.

Lines of treatment:
 Diet control.
 Diet + insulin (type 1).
 Diet + oral antidiabetic drugs (type 2).
 Exercise: to enhance peripheral glucose utilization.

█ DIET REGIME

It is the first line of treatment in all types of DM.

 Glycemic Index (GI):


– It describes the capacity of a food to raise blood glucose in normal glucose

 
282  
tole
erant individ
duals.
– Fooods with a high GI may be e associated with ↑ risk of obesity and a the
posstprandial hyperglyca
h aemia, with so ↑ the rissk of T2DM
h such foods may als M.

Calo
oric requirrements of
o patients
s Distribution of foo d elementts
Averrage wt: 30--35 c/kg/d 15 % from
m proteinss
Child
dren or thin: 40--45 c/kg/d 55 % from
m carbohyydrates
Obe
ese: 20--25 c/kg/d 30 % from
m fats

█ INSU
ULIN

Chemistry
 2 peeptide chaains: A (21 aa) and B (30 aa)
linke
ed by 2 dissulfide bon
nds.
 Insuulin is sec
creted as proinsuliin then
cleaaved (within the sto orage gran
nules in
the β cells) in
nto insulinn and connnecting
pepptide “C-peeptide”.
B. Measu
N.B urable C-peptide
C levels
indicate the presence of endoge enously
prodduced insu
ulin and functioning β
β-cells.

Secrettion
 Glucose enterrs β cells → ↑ ATP → c
closure
+
of A
ATP-depen ndent K ch hannels →
opeening of vo
oltage-gateed Ca2+ cha
annels
→ ↑ Ca influx
2+
x → insulin release.

Insulin
n receptors
s
- A tyyrosine kin ptor consistts of 2 extra
nase recep acellular
ubunits and
α-su d 2 transme
embrane β
β-subunits.
- Bind
ding of inssulin to the
e α-subunitts causes
actiivation of the
t β-subu units → activvation of tyrosine
kinaase enzym me which triggers
t a sseries of
acellular efffects that ↑ number of glucos
intra se
trannsporters (especially y GLUT4) o on the celll
mem mbrane → ↑ transport of glucosse into the cell.

 
  283
Up-regulation of insulin receptors: Down-regulation of insulin receptors:
 Insulin deficiency (e.g. in fasting)  Insulin excess (Obesity).
 Drugs: INSULIN SENSITIZERS: Obesity causes continuous
a) Thiazolidinediones: e.g. stimulation of β-cells →
pioglitazone & rosiglitazone. hyperinsulinemia → down regulation
b) Trace elements: e.g. selenium & of insulin receptors.
chromium.  Drugs: e.g. corticosteroids

Pharmacokinetics of insulin
 Endogenous insulin:
- The pancreas releases insulin in the portal vein in small bouts at short
intervals according to blood glucose levels. The liver clears ~ 50% of the
secreted amount; the rest is distributed all over the body and cleared by
receptor-mediated uptake and intracellular degradation.
- The normal blood level of insulin is 5-15 U/ml (fasting) and 60-90 U/ml
(postprandial).
 Exogenous insulin:
- Insulin is not administered orally because it is rapidly destroyed by GIT
enzymes. It is given s.c. (common) or i.v. (in emergencies).
- The plasma half-life of insulin given intravenous is 10-12 min.

Sources of commercial insulin

 Traditional (animal) insulin:


- Prepared from animals (beef and pork).
- POrk insulin differs from human insulin in One aa (alanine in pork and threonine
in human) while bEEf insulin differs from human in thrEE aa.
- Animal insulin may contain traces of animal proteins and proinsulin, so it is more
antigenic and can cause insulin resistance.

 Human insulin:
- It is identical to human insulin and is prepared by recombinant DNA technology
(genetic engineering) from yeasts or bacteria.
- Advantages: Less antigenic and rare development of insulin resistance.

 Insulin analogs:
- Recombinant insulin analogs are now available in which some amino acids in the
"normal" insulin have been switched (e.g. insulin lispro) or replaced (e.g. insulin
glargine), making a “different molecule” so as to get different pharmacokinetic
properties.

 
284  
 
  285
Insulin
n administration
 All iinsulins aree given by s.c. injecttion.
 Reg gular insulin is the only
o type thhat can be
e given i.v. in diabeticc emergencies.
 Thee standard insulin co oncentratioon is 100 units/mL
u (U
U-100). It sshould be injected
i
with
h a standard U-100 syringe.
s

Insulin
n requirem
ment and lo
ong term rregimens
 Gennerally, the
e total daily insulin rrequiremen
nt in units is equal tto 0.55 tim
mes the
persson’s weight in kilo ograms. A conserv vative total daily doose (TDD) of 0.4
unitts/kg/d is given inittially to a newly diagnosed patient; tthe dose is then
adjuusted accoording to th
he blood gglucose levvel.

 Bas
sal-bolus regimen:
r
- GGive long g acting insulin at bed time; plus thre ee daily innjection of
o short
aacting ins sulin beforre each me eal. The lo
ong-acting insulin pro
ovides bas sal level
oof insulin that
t contro
ols blood gglucose du uring night and in-bettween mea als, and
tthe short acting
a insu
ulin control s postpran
ndial hyperrglycemia.
- It is comm monly used for patien nts with Typpe-1 DM and
a in preg gnant wom men with
ddiabetes.

 Twiice-daily biphasic
b in
nsulin reg imen:
- Use biphaasic insulin. The averrage daily requiremeent
is 0.5 unitss/kg. Use the
t two-thiirds rule:
- GGive the 2/3
2 of the TDDT in the
e morning and the 1/3
a
at eveningg.
- It is commmonly used for patien ts with Typpe-2 DM

 ulin pum
Insu mp (artifficial pa ancreas): insulin is
auto
omatically released d accord ding to continuous
meaasurementt of blood glucose
g byy electronic
c sensor.

Methods of adm
ministration
- S.c. injection (using insu
ulin syringe
es).
- Porrtable pen injector (Novo® pen)).

Follow
w up of insulin therapy
By esttimation ofo glucose in urin ne using urine
dipstick cose level using
ks or from capillary blood gluc
portablle glucomeeters.

 
286  
Indicattions of in
nsulin
 Typ
pe 1 DM.
 Typ
pe 2 DM in some con nditions:
- AAfter failurre of oral drugs.
d
- If the patieent of type 2 got “strress condiitions” e.g
g. infectionns, surgery,, or
p
pregnancyy.
 Diabetic ketoa
acidosis: regular
r inssulin is the e only type used i.v.
 Hyp a (insulin + glucose i.vv.): insulin ↑ shift of K+ from blo
perkalemia ood to tissues.

se effects of insulin
Advers
- Hyp
poglycemiia: the mos
st commo n and dangerous sid
de effect.
- Hyppokalemia a: insulin caauses shiftt of K+ from extra- to
o intracelluular fluid.
- Hyppersensitiv vity reactiions: urtica
aria, angio
oedema or anaphylacctic shock.
- Insu
ulin resisttance (see below).
- Loc
cal advers
se effects:

 A
Allergy: att the site of
o injection , especially mal insulin..
y with anim
T
Treatmentt:
- Change the type of insulin.
- Local corticoster
c roids.

 Lipodystrophy: (atrophy or hypertrophy)


of s.c. tisssue after re
epeated inj ections.
T
Treatmentt: rotating thet injectio
on site.

 Local infe
ection.

▌Insuliin resistan
nce (IR)

Definittion: failure
e of the boody cells tto respond d to either endogeno ous or exoogenous
insulin. As a result, larger doses of inssulin are re
equired to give the d esired resp
ponse.

Causes
s and mec
chanism of
o IR:
- Pre-receptorss defect (IImmunologgical): due ation of a ntibodies against
e to forma
insu
ulin.
- Recceptor defeect: due too down-re egulation ofo insulin receptors ee.g. in:
- The metabolic
m syndromee: it is a combination of central obesity,o
hyperc
cholesterollemia, hearrt disease, type2 DMM and IR.
- Pregnaancy.
- Severee infection or stress.
- Drugs: e.g. cortic costeroids .

 
  287
- Post-receptor defect (intracellular): abnormal signal transduction due to genetic
defect.
- Local insulin resistance: due to degradation of insulin in s.c. tissue by proteolytic
enzymes. It is diagnosed by changing the route of administration to i.v. injection.
If there is good response, then the resistance is local.

Treatment of IR: Hepatitis C makes


people three to four times
 Life-style modification: more likely to develop
- Strict diet control. type 2 DM and insulin
- Weight reduction. resistance.
- Physical exercise.

 Correction of any precipitating factor: e.g. infection or stress.


 Metformin: to improve receptor sensitivity and ↓ intestinal glucose
absorption (see later).
 Thiazolidindiones (Insulin sensitizers): e.g. pioglitazone to ↑ insulin
sensitivity (see later).
 Change the site of injection regularly In cases of local insulin resistance

█ ORAL ANTIDIABETIC DRUGS

1. Sulfonylureas

 Sulfonylureas are the most widely used medications for the treatment of T2DM.
 All of the sulfonylureas are well absorbed after oral administration and bind to
plasma proteins.

Classification
 First-generation compounds: chlorpropamide, tolbutamide, acetohexamide.
 Second-generation compounds: glibenclamide, glyclazide and glipizide; they
are up to 200 times more potent than first-generation agents.
 Third-generation compounds: e.g. glimepiride; these compounds may interact
with different cellular proteins than other sulfonylureas.

Mechanism of action
 Pancreatic action:
- Sulfonylureas ↑ insulin secretion by pancreatic β cells. They block K+
channels in β cells, leading to depolarization, increased Ca2+ entry via
voltage-dependent calcium channels, and increased insulin secretion.
- They ↓ serum glucagon, which opposes the action of insulin.

 
288  
 Extrapancreatic action:
- They ↑ insulin receptor sensitivity, may be by increasing the number of
insulin receptors. However, sulfonylureas do not decrease the insulin
requirements of patients with type I diabetes.
- They ↓ hepatic output of glucose (gluconeogenesis).

Therapeutic uses
 Type 2 DM (they are not effective in type 1 DM).
 Chloropropamide may be used in treatment of nephrogenic diabetes insipidus
(it ↑ sensitivity of renal tubules to ADH).

Adverse effects
- Hypoglycemia especially with long acting drugs (chlorpropamide) or in elderly
patients with hepatic or renal dysfunction .. (contraindicated in renal failure).
- Increased appetite and weight gain.
- Pharmacologic failure is common, initially affecting 15%–30% of patients (1ry
failure) and 90% after 6–7 years of therapy (2ry failure). It is due to progressive
decline in β cell function.
- Hepatotoxicity (cholestatic hepatitis) .. (contraindicated in liver disease).
- Allergic reactions.

2. Meglitinides: repaglinide and nateglinide

 They increase insulin secretion by the same mechanism like sulfonylureas


although the binding site on the β-cells is different.
 They have very fast onset and short duration so; they are taken orally just before
meals to control postprandial hyperglycemia.
 They don't contain sulfur, so they can be used in patients allergic to
sulfonylureas.
 Hypoglycemia is the major side effect.
 They should be used with caution in patients with hepatic or renal impairment.

3. Biguanides: metformin

Mechanism of action
 ↓ intestinal glucose absorption and ↓ hepatic gluconeogenesis. It does not
increase insulin secretion (so it doesn’t cause hypoglycemia).
 ↑ insulin receptors sensitivity.

 
  289
Therapeutic uses
 Type 2 DM either alone (in mild cases) or in combination with other drugs.
 To enhance weight loss in obese patients (↓ glucose absorption).
 Recent studies showed that metformin is useful in the treatment of polycystic
ovary syndrome (POS); it lowers serum androgen levels and restores normal
menstrual cycles and ovulation.

Adverse effects
- GIT upset (the most common): anorexia, vomiting, and diarrhea.
- ↓ absorption of vitamin B12: rarely a clinical problem.
- Lactic acidosis: due to increased anaerobic glycolysis especially in patients
with severe renal or hepatic diseases or if taken with alcohol.

4. Thiazolidinediones (insulin sensitizers): Pioglitazone – Rosiglitazone*

Mechanism of action
 They act on nuclear genes called peroxisome proliferator-activated receptor-
gamma (PPAR- γ) present in muscles, adipose tissue, and liver cells leading to:

- They ↑ insulin receptor sensitivity (by about 60%) and ↓ insulin resistance.
- They ↑ number of glucose transporters →↑ glucose uptake.
- They have beneficial effect on serum lipoprotein levels (↓TGs).
 They have slow onset and prolonged duration because their mechanism
involves gene regulation.

Therapeutic uses: To improve insulin resistance in type 2 DM.

Adverse effects
- Hepatotoxicity
- Fluid retention leading to peripheral edema & weight gain. (They should be
avoided in patients with CHF).

*N.B pioglitazone is now the only thiazolidinedione on the market.

5. α-Glucosidase inhibitors (starch blockers): Acarbose – Miglitol

 They act by competitive inhibition of intestinal α-glucosidase enzyme → ↓


digestion & absorption of carbohydrates.
 GIT side effects are common: flatulence, diarrhea, abdominal pain.
 They should be avoided in patients with inflammatory bowel disease.

 
290  
█ NEWER ANTIDIABETIC DRUGS

Incretins are group of metabolic peptides released by gastric and intestinal cells
after eating to stimulate insulin secretion and lower plasma glucose. They are rapidly
inactivated by the enzyme dipeptidyl peptidase 4 (DPP-4). The most important
incretin peptide is glucagon-like peptide-1 (GLP-1).

1. Incretin (GLP-1) mimetics: Exenatide

Exenatide is a synthetic 39-amino acid GLP-1 analog; liraglutide is longer acting


analog and more resistant to metabolism.

Mechanism of action
 They ↑ insulin secretion and ↓ glucagon secretion.
 They slow gastric emptying and ↓ appetite.

Therapeutic uses
 Type 2 DM either alone or in combination with oral drugs. (It should not be given
with insulin).
 They are given parenterally (s.c.) 60 min before breakfast and dinner. (It should
not be given after a meal).

Major adverse effects: nausea, vomiting, and pancreatitis (may be serious).

2. Dipeptidyl peptidase 4 inhibitors (Gliptins): Sitagliptin - Saxagliptin

Mechanism of action
 They inhibit dipeptidyl peptidase 4 (DPP-4), the enzyme responsible for the
proteolysis of the incretin GLP-1.
 DPP-4 inhibitors may also improve β-cell function.

Therapeutic uses
 Type 2 DM either alone or in combination with metformin.
 They are given orally; most common side effects are headache and nausea.

3. Amylin analogs: Pramlinitide

 Pramlintide is a synthetic amylin analog. Amylin is a polypeptide secreted by β-


cells of the pancreas, and it acts in concert with insulin to reduce blood sugar.
 It is injected s.c., typically with insulin.
 Common side effects are hypoglycemia and nausea.

 
  291
Drug interactions with oral hypoglycemic drugs

Drugs potentiate the hypoglycemic Drugs antagonize the hypoglycemic


effect: effect:
 Microsomal enzyme inhibitors.  Microsomal enzyme inducers.
 β-blockers: ↓ hepatic  β-agonists: ↑ hepatic glycogenolysis.
glycogenolysis  Thiazides and diazoxide: they open
 Salicylates: displacement of ATP sensitive K+ channels → ↓ insulin
sulfonylureas from plasma proteins. release and ↓ peripheral glucose
utilization.
 Anti-insulin hormones: e.g. steroids
and glucagon.

█ TREATMENT OF DIABETIC COMPLICATIONS

▌Hypoglycemic coma

Causes:
- Large dose of insulin or sulfonylurea.
- Missed meal while taking insulin or sulfonylureas.
- Vigorous muscular exercise without dietary adjustment.
Manifestations:
- Sympathetic overactivity: tremors, cold sweating, mydriasis, tachycardia, etc.
- Hunger pain, mental confusion, and coma.
Treatment:
 If the patient is conscious or semiconscious → give oral sugar solution.
 If the patient is in deep coma: (1) i.v. glucose 10%; (2) Glucagon 1 mg i.m.

▌Hyperglycemic coma (diabetic ketoacidosis; DKA):

Causes:
- Excess food intake.
- Inadequate insulin administration.
- Infection or stress condition.
Manifestations (DKA):
- Dehydration and dry shrunken tongue.
- Ketosis: smell of acetone in breath and ketones in urine.
- Acidosis (pH < 7.3) → acidotic breathing (rapid and deep).
- Deep coma in late cases.

 
292  
Treatment:
 Fluid replacement: fluid deficit = 4-5L.
– Use normal (0.9%) saline. Ringer’s lactate is also acceptable choice.
– Start with 1–2 L/h for the first 2 hours, followed by 500 mL/h for 6 hours.
– Potassium: is routinely added to the i.v. fluid as 20 mEq KCl/1L.

 Regular insulin:
– Start with a bolus of dose of 0.1 units/kg i.v. followed by an infusion of 0.1
units/kg/hour.
– Insulin infusion should continue until serum electrolytes, pH, and glucose
level are normalized.
– Glucose 5 % is given when blood glucose falls to ~250 mg/dl to prevent
cerebral edema.

 Bicarbonate (HCO3): is not routinely given unless there is severe acidosis


(pH < 7.1).

▌Hyperosmolar hyperglycemic state (HHS):

 HHS was previously termed hyperosmolar hyperglycemic nonketotic coma


(HHNC); however, the terminology was changed because coma is found in fewer
than 20% of patients with HHS.
 It is a life-threatening medical emergency that is more common in old age.
 Pathogenesis: severe hyperglycemia causes ↑ water loss (osmotic diuresis) with
little sodium loss → severe dehydration
with hyperosmolarity. Ketoacidosis is DKA HHS
absent. BG > 250 mg/l > 600 mg/l
pH < 7.3 > 7.3
Treatment: similar to DKA except in: Ketonuria ++ --

 Fluid replacement: fluid deficit


>5L.
– Rapid and aggressive i.v. isotonic saline is the principal treatment. The
patient may require 5-8 L of saline.
– Fluid replacement alone can reduce glucose levels.
 Insulin i.v.i similar to DKA. Insulin used without concomitant vigorous fluid
replacement increases the risk of shock.
 Hypokalemia is usually less marked but potassium may be required early in
the treatment.
 LMWH to prevent thrombotic complications.

 
  293
▌Diabetic neuropathy

Diabetic neuropathy is the most common complication of diabetes mellitus (DM),


affecting as many as 50% of patients with type 1 and type 2 DM. It may be cranial,
sensory, sensory-motor or autonomic. DN is an irreversible process.

Pathogenesis
 Advanced glycated end-products: elevated intracellular levels of glucose cause
a non-enzymatic covalent bonding with cellular proteins, causing microvascular
damage, ischemia, hypoxia, and nerve damage.

 Polyol pathway (sorbitol/aldose reductase pathway): elevated intracellular


glucose is converted into sorbitol via the enzyme aldose reductase. Sorbitol is
toxic to tissues.

Treatment
 Tight, stable glycemic control is the most important factor for slowing
progression of DN.
 For neuropathic pain, and tingling sensation: duloxetine (antidepressant drug)
30 mg/12h orally (first choice) or pregabalin (second choice).
 For diabetic gastroparesis: metoclopramide
 For erectile dysfunction: sildenafil

 
294  
Part 2
2: Med
dical Tre
eatmentt of Obe
esity

█ Bas
sic inform
mation

Leptin
 It is thought to
o play a ke
ey role in thhe regulation of body
y weight.
 It iss produced (synthes sized) by adipose tissue
t and
d acts on satiety centers in
the hypothala amus to ↓ appetite
a (i.e
e. Leptin induces saatiety).
 As such whe en patientss reach a certain peripheral
p fat mass, leptin actts as a
lipoostat to red
duce food d intake.

Ghrelin
n
 Ghrrelin stimu ulates hunnger.
 It is producedd mainly byy the fundu
us of the sttomach an
nd pancreaas.
 Ghrrelin levels ↑ before meals
m and ↓ after me
eals.

The bod
dy-mass in
ndex (BMI) – (Kg/m2)

 Norrmal: 18-25
5
 Ove
erweight: 25-30
0
 Obeese: >30
 Morrbid obesitty: >40

 
  295
▌Medical treatment

1. Orlistat (Xenical®)

 Orlistat is a pancreatic lipase inhibitor. It inhibits digestion and absorption of


dietary fats leading to weight loss.
 Orlistat is normally used for <1 year. Patients fail to lose at least 5% of their
bodyweight within 3 months, orlistat should be discontinued.
 Adverse effects include steatorrhea and flatulence.

2. Lorcaserin (Belviq®)

 It is recently approved selective 5-HT2C receptor agonist in the hypothalamus


leading to weight loss through satiety.
 It is used for long term control of obesity.
 Adverse effects are mild and include mainly headache.

3. Liraglutide (Saxenda®)

 It is an injectable long acting glucagon-like peptide-1 receptor agonist, binding


to the same receptors as does the endogenous hormone GLP-1 that stimulates
insulin secretion.
 It is developed principally for treatment of type 2 DM. In 2015, it was approved
for treatment of morbid obesity in adults.

4. Sibutramine (Meridia®)

 Centrally acting appetite suppressant related to amphetamine (inhibits uptake


of 5-HT and noradrenaline at hypothalamic sites that regulate food intake).
 Adverse effects: hypertension, arrhythmia, insomnia, and headache.
 It has been withdrawn in 2010 from the USA due to an increased risk of
cardiovascular events, but still available in many countries including Egypt and
Middle East.

 
296  
Part 3
3: Thyroid Gla
and and Antithy
yroid Dru
ugs

Genera
al principles
 Thee major hormone
h secreted
s b
by the
thyrroid is thyroxine
t (T4), whiich is
deioodinated in many tissues tto the
morre potent triiodothyrronine (T3)). Both
are bound reversibly y to thyrroxine-
bindding globuulin (TBG)). Only th
he free
(unbbound) frraction en nters cellss and
produces biollogical effe
ects.
 Thee most imp ep in the p rocess
portant ste
of thyroid hormone
h synthesis
s is oxidation (orga anification)) of iodid
des by
perroxidase enzyme
e to
t produc ce molecu ular iodine
e, which tthen attac ches to
tyro
osine to foorm monoo- and diiiodotyrosin ne. T4 is formed b by couplinng of 2
diio
odotyrosine
es, and T3 by couplinng of diiod
dothyrosine
e with monno-iodotyro osine.
 T4 secretion is stimula
ated by thyyroid-stimulating ho
ormone (TS
SH). In turrn, TSH
sec
cretion is in
nhibited by
y T4, formin
ng a negattive feedba
ack loop.
 Thee gland syn
nthesizes T4
T > T3 (20
0:1) but T3
3 is 4-times
s more pottent than T4.
T
 Mosst of the cirrculating T3 is derived
d from peripheral deio
odination o
of T4.
 B-b
blockers an
nd corticos
steroids inhibit periph
heral conve
ersion of T44 into T3.

Preparrations of thyroid ho
ormones
 L-th
hyroxine: a synthetic sodium
salt of T4 tha
at maintain ns normal
T4 aand T3 levels (t ½ is 7 days).
 Liotthyronine:: a synthetic
sod
dium salt off T3 (t ½ is
s 1 day).

anism of action
Mecha a
 T3 & T4 diffuse through
h the cell
memmbranes and bind
b to
cyto
oplasmic receptorrs then
tran
nsported too the nucleus and
mitochondria a, where it interacts
with
h many DNA receptors
(genes) and affect
a their function.
 Mosst of T3 & T4 recepto
ors are fouund in pituitary, liver, kidney, heeart, sk ms
s.
 T3 & T4 are re
esponsible
e for optima al growth and
a develo opment.

 
  297
▌Hypo
othyroidis
sm (myxedema)

 Hyp
pothyroidissm in infants lead
ds to
cretinism (m myxedema with phyysical
andd mental re etardation)..
 It iss treated by replacement witth L-
thyrroxin (T4). The thera apeutic go
oal is
'norrmalizationn' of the TS
SH level.
 Children requ uire more T4T than a adults
to maintain optimal physical and
men ntal develo
opment.
 T3 (Liothyronine) is nott used forr replacem
ment therappy becausse L-thyroxin (T4)
cann maintain normal T4 and T3 levvels, and also
a becau
use T3 has shorter t½
½.

▌Hype
erthyroidiism (thyro
otoxicosi s)

Definittion: It is a clinical sy
yndrome rresults from
m high leve
els of thyro
oid hormon
nes.

Clinica
al types

 Gra
aves’ diseaase:
- It is autoim
mmune dis
sease in wh
hich there are
a abnorm
mal
antibodiess (thyroid stimulating
s g immunog globulins)
activates TSH recep ptors in the
e thyroid gland.
g
- The glandd is diffuse
ely enlargedd and soft.
- The treatm
ment is usually medi cal.
 Toxxic multino
odular goiiter: treatm
ment is usu
ually surgic
cal.
 Lesss commo
on types: e.g.
e subacu
ute (de
The terrm “goiter”
Que
ervain’s) th
hyroiditis. It is due to viral infection. means any thyroid d
swellinng which is
Clinica
al picture neitherr inflammatoory
nor neo oplastic. Th
hyroid
 Mosst manifesstations off hyperthyyroidism re
esemble enlargeement may occur
symmpathetic overactivity be
ecause thyroxin
t with hyypo- or
hyperthhyroidism.
incrreases sen gic -recep
nsitivity off adrenerg ptors to
circ
culating cattecholamin nes.
 Theere are tach
hycardia, arrhythmia
a a, sweating
g, lid lag, ex
xophthalm
mos, etc.

Investiigations
- Meaasuring serrum T3, T4, and TSH (the mostt important test): T4 is ↑ and TS
SH is ↓.
- Asssay for possitive anti-thyroid anttibodies: 90
0% +ve in Grave's d
disease.
- Thyyroid scan for tumorss.

 
298  
Management

There are 2 main lines for treatment:

 Medical treatment: mainly for Grave’s disease.


 Surgical thyroidectomy: mainly for multiple nodular goiter

█ MEDICAL TREATMENT (ANTITHYROID DRUGS)

1. Thiouracil drugs (thioamides)


(Carbimazole – Methimazole - Propylthiouracil)

Pharmacokinetics
 Methimazole is the active metabolite of carbimazole.
 The t1/2 of propylthiouracil is 1.5 hr while the t1/2 of methimazole is 6 hrs.
 The short t½ of these drugs has little effect on their effect because they are
selectively accumulated in the thyroid.
 Propylthiouracil is preferable during pregnancy because it does not cross
placental barrier (because it is strongly bound to plasma protein).

Mechanism of action
 They inhibit oxidation of iodides into iodine by inhibiting peroxidase enzyme.
Consequently, they inhibit iodination of tyrosine and coupling of iodotyrosine to
form iodothyronines.
 Propylthiouracil also inhibits the peripheral conversion of T4 into T3.
 Clinical response appears after 3-4 weeks till the stored hormones are depleted from
the gland (but propylthiouracil has faster effect, so it is used in thyrotoxic crisis).

Doses and duration


 Carbimazole (Neomercazole): start with 30 mg/d till reach euthyroid state (after
~4-8 weeks) then maintain on 15 mg /d for 12-18 months (until the gland
undergo spontaneous remission).
 Propylthiouracil: start with 300 mg/d for 4-8 w then 150 mg/d for 12-18 months.

Adverse effects
– Agranulocytosis & bone marrow depression (<1%): it is the most dangerous
side effect but it is usually reversible (see chapter Blood).
– Hypothyroidism with increased size and vascularity of the gland due to ↑ TSH.
– Hypothyroidism of the infant (fetal goiter) if given during pregnancy (less
common with PTU).

 
  299
– Hepatotoxicity that may be fatal (more with propylthiouracil).
– Hypersensitivity reactions: may require stopping of the drugs.
– There is 50-68% incidence of relapse.

Precautions during thiouracil treatment


 Therapy with thiouracil drugs should continue for 1-2 years and should be
stopped gradually (to prevent relapse).
 Agranulocytosis is prevented by repeated WBC count and by observing early
manifestations of the disease (sore throat & fever).
 If used during pregnancy, propylthiouracil is the drug of choice.

2. Radioactive iodine

Radioactive iodine is an effective oral treatment for thyrotoxicosis caused by toxic


multinodular goiter. It is relatively ‘safe’ and has largely replaced surgery (unless
there is huge gland causing pressure symptoms).

Mechanism of action
 The isotope usually employed is 131I with a t ½ of 8 days.
 Radioactive 131I is selectively accumulated in the thyroid tissue (normal or
metastatic) and emits β rays that destroy the gland. After 6-12 weeks of
administration, the gland will shrink in size and the patient becomes euthyroid.
 Delayed hypothyroidism is the main adverse effect (80%); the majority of
patients will require thyroxin supplementation after 5 years.

Contraindications
– Pregnancy and lactation: 131I crosses placental barrier and excreted in milk.
– Age < 16 years for fear of delayed malignant changes and gonadal damage.
– Thyroid eye disease (exophthalmos; ophthalmopathy): radioiodine may
worsen the condition.

3. -blockers (Propranolol)

 It controls sympathetic overactivity e.g. tachycardia and arrhythmia.


 It ↓ insomnia, and tremors (blocks central and peripheral β2 receptors).
 It inhibits peripheral conversion of T4 to T3.
N.B. If propranolol is contraindicated, give diltiazem to control tachycardia and
arrhythmia (other CCBs are less effective).

 
300  
█ SURGICAL TREATMENT: subtotal thyroidectomy

Indications
 Failure of the medical treatment.
 Multinodular goiter with tracheal compression.
 Presence of malignancy.

Preparation of patient before operation


 Carbimazole: 10 mg t.d.s. 6 weeks before the operation to reach euthyroid
state.
 Potassium iodide: 10-14 days before operation to ↓ size and vascularity of
the gland (see below).
 Propranolol: to control HR and cardiac arrhythmia.
 Sedatives (diazepam): to ↓ anxiety.

Iodides

 Potassium iodides or Lugol’s iodine, 5 drops twice daily is given 10-14 days
before surgery in order to:
– Iodides inhibit synthesis and release of T4 & T3 from the gland by inhibiting
the proteolytic enzymes that release T4 & T3 from thyroglobulin (the main
mechanism).
– They inhibit release of TSH leading to ↓ size and vascularity of the gland.
 Improvement in thyrotoxic symptoms occurs within 2–7 days, but if therapy with
iodides is continued (>2-4 weeks), the beneficial effects disappear and
manifestations of hyperthyroidism reappear (iodine escape).

Adverse effects
– Iodides are secreted in saliva, nasal, and lacrimal secretions causing metallic
taste and irritation of the salivary glands, mucous membranes and gastric
mucosa. They increase lacrimal and nasal secretions.
– Iodine escape if used > 2-4 weeks.
– Iodides increase intraglandular stores of iodine, which may impair uptake of
thiouracil drugs or radioactive iodine by the gland. So, during preoperative
preparation, they must be given after thiouracil drugs (or radioactive iodine),
not before them.
– Allergic reactions: skin rash, drug fever, etc.

 
  301
▌Thyrotoxic crisis (thyroid storm)

It is a sudden severe exacerbation of the manifestations of thyrotoxicosis due to


sudden release of T3 & T4 (medical emergency). It is a common postoperative
complication if the patient was not well-prepared.

Manifestations – High fever with vomiting and sweating.


– Tachycardia, arrhythmia, acute heart failure, and shock.
– Convulsions, coma, and even death from heart failure.
Management
 Intravenous fluids and Paracetamol to control dehydration and fever.
N.B. Aspirin must be avoided, because salicylate displaces bound T4 and T3,
and also because of its uncoupling effect on oxidative phosphorylation renders
the metabolic state even more severe.

 Propranolol (1-2 mg slowly i.v. or 40 mg oral /6 h) to control arrhythmia.


– Esmolol is a short-acting -blocker can be also given by i.v.i.
– If -blockers are contraindicated give diltiazem by i.v. infusion.
 Potassium iodides: 10 drops orally/day to block hormone release and
peripheral conversion of T4 to T3.
 Propylthiouracil: 250 mg/6 hrs orally to block hormone synthesis. It acts more
rapidly than other thiouracil drugs.
 Hydrocortisone: 100 mg i.v./6-8 h to elevates BP and reduces toxemia. It also
blocks peripheral conversion of T4 to T3.
 Plasmaphoresis in severe cases to reduce circulating thyroxin.

▌Management of myxedema coma

Myxedema coma is an end state of untreated hypothyroidism. There is progressive


weakness, hypothermia, hypoventilation, hypoglycemia, hyponatremia, water
intoxication, shock, and death. It carries high mortality and is a medical emergency.

Management
 Hospitalization (ICU): artificial respiration may be required.
 L-thyroxine (T4): 400 μg i.v. initially, followed by 50 μg daily orally. Intravenous
T3 can be used.
 Hydrocortisone: 100 mg i.v./ 6-8 h because the patient usually has associated
adrenal insufficiency.
 Intravenous fluids with caution to avoid excessive volume overload.

 
302  
Part 4: Adrenocortical Steroids

Classification:  Glucocorticoids: have mainly anti-inflammatory action.


 Mineralocorticoids: have mainly Na & water retaining action.
 Sex hormones: are mainly anabolic.

Glucocorticoids

Anti-inflam Na+ retention Duration


potency potency of effect
Short-acting glucocorticoids:
Hydrocortisone (cortisol) 1 1 8-12 h
Prednisone and Prednisolone 4 0.5
Methylprednisolone 5 0.1
Intermediate-acting glucocorticoids:
Triamcinolone 5 0.1 12-36 h
Paramethasone 10 0.01

Long-acting glucocorticoids:
Dexamethasone 30 0.01 48 h
Betamethasone 30 0.01

Aldosterone: 0.3 3000

 Hydrocortisone (cortisol) is the natural (endogenous) glucocorticoid.


 Equivalent dose means 20 mg of hydrocortisone = 5 mg of prednisolone.
 Steroids with high anti-inflam action (e.g. dexamethasone) are better used for
inflammatory conditions e.g. rheumatic fever, while those with high Na & H2O
retention (hydrocortisone) are better used for treatment of hypotension and
shock states (they also have rapid action).

Pharmacokinetics
 All glucocorticoids are completely & rapidly absorbed by all routes.
 80% of hydrocortisone is bound to plasma globulin, 10% to albumin.
 Plasma t ½ varies according to type (60-90 min for hydrocortisone). However, the
effect of glucocorticoids is prolonged due to its effect on gene functions.
 Metabolism is by the liver and excretion is by the kidney.

Mechanism of action
Corticosteroids bind first to cell surface receptors then to cytoplasmic receptors

 
  303
(carrierss), then transported
d to the nucleus, where it interacts with man
ny DNA
recepto ors (steroid
d response
e elementss) and affect their fun
nction.

Pharm
macologica
al effects:

 On m
metabolis
sm: (Cushiing syndro
ome):
- CCarbohydrrate metab
bolism: hyyperglycem
mia (↓
pperipheral glucose uttilization).
- PProtein metabolism
m m: Catabo olic effect → ↓
mmuscle ma ass and thin limbs.
- FFat metabo olism: ↑ lip
polysis withh redistribution of
ffat (moon fa
ace & buffaalo hump).
- OOn water and
a electro olyte balannce: Na+ & water
rretention and
a hypoka alemia.

 Antii-inflamma
atory and anti-imm unologica
al
effe
ects:
- T They inhibbit B cell function n → ↓ an ntigen-
a
antibody re
eaction.
- T nctions → ↓ inflam me
They inhibiit T cell fun ediators annd cytokinee release.
- T They inhibiit macroph hage activitty and stabilize lysos
somal mem mbranes.
- T They inhibiit mast cells → ↓ hista
amine relea ase and ca apillary perrmeability.
- T They inhibiit phospho olipase A2 enzyme → ↓ synthesis of PGs & LTs.

 On C
CVS: Hypeertension due
d to:
- N +
Na & wateer retentionn.
- Increase seensitivity of
o BV and h
heart to cirrculating catecholam
mines.

 Antii-shock efffects: due


e to:
- HHypertensiive and CV VS effects (mention them).
- AAnti-inflam
mmatory ac ction (menttion them).

 Hem
matologicaal effects:
- ↑ RBCs and neutroph hils and ↓ lymphocyttes and eosinophils.
- ↑ coagulation factorss and bloodd choleste
erol.

 On g
growth: Growth
G reta
ardation, w
which is nott prevented by growtth hormon
ne.

 On b
bone: ↓ bo
one matrix and ↑ Ca22+ excretion
n (osteopo
orosis).

Administration
- Drug
g administtration sho
ould follow
w the circa hm: A douuble dose is given
adian rhyth
in th
he morning
g, and a sin
ngle dose is given in the afternoon.

 
304  
- Alternate-day therapy is clinically effective with minimal effect on the adrenal-
hypothalamic-pituitary axis. In this therapy, double the dose of short- or
intermediate-acting glucocorticoids is administered every other day.
- Glucocorticoids should be stopped gradually after long term administration.

Therapeutic uses

 Inflammatory disorders: e.g. iridocyclitis, neuritis, dermatitis, etc.


 Autoimmune disorders: e.g. vasculitis, rh fever, rh arthritis, hemolytic anemia,
nephrotic syndrome, ulcerative colitis, etc.
 Allergic disorders: e.g. anaphylactic shock, urticaria, eczema, bronchial asthma,
allergic conjunctivitis, etc.
 Malignancy: e.g. lymphoma, leukemias, multiple myeloma, etc.
 Shock and hypotension…..Why?
 Organ transplantation: as immunosuppressive to prevent graft rejection.
 Cerebral edema: dexamethasone is used after brain surgery to minimize
inflammatory edema associated with tissue injury.
 Acute hypercalcemia: to enhance Ca2+ excretion.
 As replacement therapy in acute and chronic adrenocortical insufficiency
(Addison’s disease).

N.B. Acute adrencortical insufficiency (acute Addisonian crisis):


- Prolonged corticosteroid therapy produces feedback inhibition of ACTH and
inhibition of endogenous cortisone secretion. When exogenous
corticosteroids are suddenly stopped, severe hypotension and shock occurs.
- Management:
- i.v. fluids (saline).
- Hydrocortisone 100 mg i.v./8h until the patient is stable.
- ACTH: 0.5 mg i.m.

 Stimulation of lung maturation in the fetus:


Lung maturation in the fetus is regulated by fetal secretion of cortisol. When
delivery is anticipated before 34 weeks of gestation, betamethasone is given to
reduce the incidence of respiratory distress syndrome in the infant.
Betamethasone is chosen because its maternal protein binding is less than other
steroids, allowing increased transfer across the placenta to the fetus.

Adverse effects
Most of these side effects occur after long duration of therapy:

 
  305
- Iatrogenic Cushing syndrome: occurs if doses up to 100 mg hydrocortisone are
used daily for > 2 weeks. It is characterized by moon face, buffalo hump, thin
limbs, osteoporosis, hypertension, DM, edema, etc.
- Immune suppression leading to flaring of infections (especially viral and TB).
- Hypertension due to salt & water retention
- Hyperglycemia.
- Peptic ulcer: due to prolonged inhibition of gastroprotective PGs.
- ↑ IOP (Glaucoma): due to ↓ aqueous humor drainage.
- Osteoporosis.
- Growth retardation in children.
- Skin atrophy & hypopigmentation after prolonged topical use.
- Sudden withdrawal after prolonged administration causes acute addisonian
crisis.

Contraindications
- Presence of infections: especially viral
N.B. Uses of corticosteroids in
infection and TB.
presence of T.B:
- DM. - TB meningitis: to prevent
- Hypertension & heart failure: they cause adhesions.
salt and water retention. - TB of the suprarenal gland: to
- Peptic ulcer: they ↓ synthesis of PGE2 and replace hypofunction.
I2 that protect the stomach. - Miliary TB: to ↓ TB toxemia.
- In early pregnancy: may cause cleft palate.

▌Mineralocorticoids

1. Natural mineralocorticoids: Aldosterone

Physiological actions: It binds to specific intracellular receptors in the DCT to


inhibit Na+ excretion (hypernatremia) and stimulates K+ and H+ excretion
(hypokalemia).

2. Synthetic mineralocorticoids:

 Deoxycorticosterone acetate (DOCA): it has mainly mineralocorticoid effect.


 Fluodrocortisone: it has both gluco- and mineralocorticoid effects.

 
306  
 
  307
 
308  
 
  309
▌Hype
ercalcemiia

Causess: In over 90% of caases hyperrcalcemia is i due to either


e hypeerparathyrroidism
or malignancy (eespecially myeloma) . Hypercallcemia norrmally sup presses PTH P and
so PTH
H is therefo
ore the bes
st first test to identity
y the cause
e of hyperccalcemia.

Manag
gement of acute hyp
percalcem
mia
- Saline diuresis: 500-1000 ml/houur plus furrosemide to
t increasse urine flo
ow and
enhance Ca2+ excretion..
- Hyddrocortison
ne: 100 mg hance Ca2++ excretion
g i.v. to enh n.
- Intraavenous bisphosphoonates.
- Hemmodialysis especially
y when ren al failure is
s present.

▌Hypo
ocalcaemia

Causess: Hypopa arathyroidissm (N.B. ttetany occcurs when serum Caa2+ falls < 7mg/dl),
7
chronic
c renal failu
ure, vitamin D deficie
ency, etc.

Treatm
ment
- Diett rich in calcium and low in pho osphate
- Calccium gluco onate: slow
wly i.v. (in a
acute condditions).
- amin D: to ↑ Ca abs
Vita +2
sorption fro om intestin
ne.
- Thia
azide diure etics.

▌Osteo
oporosis

Definittion: it is a condition
n of low bo
one mass that results in fractuures with minimal
trauma
a. It occurss in postme enopausal women (d
due to estro
ogen lack) and in old d age.

Preven
ntion and treatment
t t of osteop
porosis
- Diett rich in calcium.
amin D: to ↑ Ca+2 abs
- Vita sorption frrom intestin
ne
- Sele
ective Esttrogen Re
eceptor M
Modulators en): to ret ain the be
s (Raloxife eneficial
effec
cts of estro
ogen on bone while minimizing
g the risk of
o cancer b
breast and uterus.
- Bisp
phosphonates (e.g. risedron
nate): they preventt bone reesorption (inhibit
osteeoclastic activity) and
d reduce riisk of hip and
a spine fractures.
f T
They are effective
e
in bo
oth men an nd women n for variou
us causes of
o osteopo
orosis.
- Calc
citonin: to
o increase bone
b masss and redu
uce fracture
es.
- Teriiparatide: a recombin
nant form o
of PTH tha
at has been
n recently aapproved fo
or treat-
men
nt of osteop
porosis. It stimulates
s n
new bone fo
ormation and reducess risk of frac
ctures.

 
310  
- Slow e fluoride preparatiion: is a
w release
new
w treatmennt. It may
y reduce rates in
posttmenopausal osteop
porosis.

Part 6
6: Sex Hormon
nes

I. Estro
ogen

 Nattural estro
ogens: estrradiol, estrrone and estriol
e
 Sem
misynthetiic estrogeens: Ethinyyl estradiol and mestrranol.
 Syn
nthetic esttrogens: diethyl
d stilb
bosterol.

Physio
ologic effe
ects
 Normmal develoopment of genital tra
act and breeast.
 Devvelopment of ♀ secon ndary sex characters s.
 Metabolic effe
ects:
- IIncrease bone mass and preveent bone re esorption.
- IIncrease blood gluco ose and TGGs.
- S Salt and water
w retenttion.
 Increase bloodd coagulattion and pl atelet adhesiveness..

Therap
peutic use
es
 C
Contracepptive pills.
 Dysfunctio
onal uterine e bleeding .
 Replacement therapy in ovaria an hypofunnction.
 Postmenopausal sym c vaginitis and osteo
mptoms e..g. atrophic oporosis.
 C
Cancer proostate.

Advers ons: see contracepti


se effects and contrraindicatio c ive pills.

▌Anti--estrogen
ns:

1. Clomiphene citrate (C
Clomid)

 Clom
miphene blocks
b estrogen rece
eptors in hypothalam pituitary → ↑ FSH
mus and p
and LH → stim mulate ovulation.
 It is used to sttimulate ovulation
o i n infertile women
w witth normal p
pituitary fu
unction.
 Adv verse effeccts: Ovaria
an enlargem ment and hot flushess.

 
  311
2. Selective estrogen receptor modulators (SERMs):

 SERMs are ligands for the estrogen receptor that have agonist activity in one
tissue but may have antagonist activity or no activity in another tissue.
 Currently, there are three SERMs: tamoxifen, raloxifene, and toremifene.

Tamoxifen
 It is an estrogen antagonist in the breast but is an agonist in the uterus and
bone.
 It is used in the treatment of advanced, estrogen receptor positive breast
cancer and for primary prevention of breast cancer in women at high risk.
 Adverse effects: tamoxifen increases the risk of endometrial cancer and
thrombotic complications.

Raloxifene
 It is an agonist in bone but has no effect on the uterus or breast.
 It is used for the treatment and prevention of postmenopausal osteoporosis.
 Adverse effects: hot flashes and thrombotic complications.

3. Aromatase inhibitors:

 Aromatase is the enzyme that catalyzes the production of estrogens from


androgenic precursors within the ovary and peripheral tissues.
 Aromatase inhibitors are a new class of oral estrogen synthesis inhibitors.
 Anastrazole and letrozole (Femara) are nonsteroidal competitive inhibitors of
aromatase. Exemestane is a steroidal, irreversible aromatase inhibitor.
 These drugs are used for treatment of postmenopausal women with estrogen-
receptor positive breast cancer who have received two to three years
of tamoxifen and are switched to them for completion of a total of five years of
adjuvant hormonal therapy.

II. Progesterone and progestins

 Natural: progesterone injection


 Synthetic: medroxy progesterone acetate.

Therapeutic uses
 Contraceptive pills.
 Dysfunctional uterine bleeding
 Dysmenorrhea and endometriosis.
 Threatened abortion.

 
312  
Adverse effects
- Breakthrough bleeding.
- Increase risk of birth defects if given in early pregnancy.
- Liver dysfunction.

▌Antiprogesterone: Mifepristone

 It is a competitive blocker of progesterone receptors.


 It is used with PGF2α to induce medical abortion in the first trimester.

III. Androgens and anabolic steroids

 Natural androgens: androsterone and testosterone


 Synthetic androgens: testosterone propionate.
 Anabolic steroids: nandrolone and stanazol.

Uses of androgens and anabolic steroids:


 Chronic debilitating diseases e.g renal failure.
 Chronic refractory anemia: e.g. sickle cell anemia, aplastic anemia.
 Illicit use by athletes: to increase muscle bulk and strength.

Adverse effects
- Reduction in spermatogenesis after stopping.
- Precocious puberty and premature closure of epiphysis in children.
- Cholestatic jaundice.
- Verilizing effects in females.

Contraindications
- Prostatic tumors (benign and malignant).
- Liver diseases
- Children.

▌Antiandrogens

1. 5 α-reductase inhibitors: Finasteride

 It inhibits 5 α-reductase enzyme responsible for conversion of testosterone into


the active form dihydrotestosterone (DHT).
 It is used in:

 
  313
 Treatment of benign prostatic hyperplasia.
 Treatment of male baldness.
 Treatment of hirsutism in females.

2. Testosterone receptor blockers:

- Cyproterone acetate: is a competitive blocker of testosterone receptors. It


is used in male hypersexuality, and hirsutism in females.

- Flutamide: used in cancer prostate.


- Others: Spironolactone is a competitive blocker of both aldosterone and
testosterone receptors. It is used in hirsutism in females.

█ HORMONAL CONTRACEPTIVES

Types of hormonal contraceptives

 Combined preparations (the most effective type): contain both estrogen +


progesterone given from the 5th day of menstruation for 21 days.

 Single entity preparations:


 Progesterone alone (minipills): Don't affect lactation and don't carry risk of
thrombosis but can cause uterine bleeding.
 Estrogen alone (postcoital pills or morning-after pills): It is used within 72 hrs
after sexual intercourse for 5 days.
 Slow release progestins: Medroxyprogesterone acetate (Depo-Provera®) given
i.m. every 3 months. It is suitable for unreliable women.
 Implantable progestin preparation.

Mechanism of action
 They inhibit ovulation by exerting –ve feedback on LH (progesterone) and FSH
(estrogen) secretion.
 Produce endometrial changes and interferes with coordinated contraction of
the cervix, uterus, and fallopian tubes → ↓ sperm transport and fertilization.
 Increase viscosity of cervical mucus to inhibit sperm penetration.

Adverse effects
CVS: the most serious side effects especially in women above 35 years and in
women who are smokers:
- Hypertension and increase risk of myocardial infarction.

 
314  
- Thrombosis and thromboembolic catastrophes.
- Increase TGs levels.

CNS:
- Migraine headache.
- Cerebral hemorrhage (stroke) is 2-10 times higher.
- Mood changes and depression.

GIT:
- Nausea and vomiting.
- Cholecystitis and gall stones.
- Cholestatic hepatitis and hepatotoxicity.

Endocrinal:
- Hyperglycemia and DM.
- Weight gain and edema due to salt and water retention.
- Inhibition of lactation in lactating women.
- Menstrual irregularities: spotting bleeding, breakthrough bleeding, amenorrhea,
and dysmenorrhoea.
- Loss of libido, acne, and hirsutism.

Cancer: increased risk of breast cancer.

Contraindications
- Hypertension or ischemic heart disease (IHD).
- History of embolism, thrombosis or cerebral hemorrhage.
- History of cancer breast or estrogen-dependent neoplasm.
- Migraine headache.
- Chronic liver disease and gall stones.
- Diabetes mellitus.
- Obese, smokers, or women over 35 years.
- Pregnancy.
- Depression.

N.B Causes of failure of contraceptive pills:


 If taken with enzyme inducers e.g. rifampin, phenytoin, etc.
 If woman taking broad spectrum antibiotics e.g tetracyclines (see breakthrough
pregnancy in general pharmacol).
 Paraffin oil (laxative) ↓ intestinal absorption of contraceptive pills.

 
  315
Part 7: Hypothalamic and Pituitary Hormones

█ ANTERIOR PITUITARY HORMONES

▌Growth hormone (GH)

GH promotes longitudinal bone growth and cartilage synthesis through stimulation


of hepatic synthesis of insulin-like growth factors (somatomedins).

Therapeutic uses
- Replacement therapy in children with GH deficiency.
- Illicit use by athletes to increase body mass.

Adverse effects:
- Children may develop scoliosis during rapid growth.
- Peripheral edema and carpal tunnel syndrome.
- Hypothyroidism and gynecomastia.

▌Growth hormone antagonists: Octreotide

- It is a synthetic somatostatin analog.


- It is more potent and has longer duration than somatostatin.

Therapeutic uses
- Acromegaly.
- Hormone-secreting tumors (e.g. insulinoma, glucagonoma, gastrinoma, etc).
- Bleeding esophageal varices (given by i.v.i., it causes VC of splanchnic bl vessels
and controls variceal bleeding with fewer side effects than vasopressin; see GIT).

Adverse effects: Bradycardia and conduction disturbances.

▌Adrenocorticotrophic hormone (ACTH)

ACTH is secreted by the pituitary to stimulate release of glucocorticoids,


meniralocorticoids, and sex hormones. It also stimulates MSH → skin pigmentation.

Therapeutic uses
 To stimulate secretion of endogenous corticosteroids.
 Diagnostic: differentiation between 1ry or 2ry adrenal insufficiency.

 
316  
▌Gonadotropin-releasing hormone (GnRH)

 GnRH is secreted by the hypothalamus to stimulate release of LH and FSH from


anterior pituitary.
 Pulsatile injection of GnRH causes stimulation of LH & FSH but continuous
infusion of GnRH causes inhibition of LH & FSH release.

Therapeutic uses
 Treatment of infertility caused by hypogonadism in both sexes by pulsatile
injection of GnRH (to stimulate LH & FSH).
 Treatment of prostate cancer, endometriosis, and polycystic ovary syndrome by
continuous administration of GnRH (to inhibit LH & FSH).
 In-vitro fertilization (IVF) programs.

▌Follicle-stimulating hormone (FSH)

Therapeutic uses
 Treatment of infertility caused by hypogonadism in both sexes.
 To stimulate ovulation as a part of in-vitro fertilization (IVF) programs.

Adverse effects
- Hyperstimulation syndrome (enlarged ovaries, ascites, fever, embolism, etc.)
- Gynecomastia in men.

▌Human chorionic gonadotropin (hCG)

It is glycoprotein produced by the placenta to stimulate ovarian corpus luteum to


secrete progesterone (like LH). hCG can be used as LH substitute.

Therapeutic uses
 Treatment of infertility caused by hypogonadism in both sexes.
 To stimulate ovulation as a part of in-vitro fertilization (IVF) programs.
 Differentiation between undescended testes (cryptorchidism) and retracted
testes (pseudo-cryptorchidism).

 
  317
█ POSTERIOR PITUITARY HORMONES

▌Antidiuretic hormone (Vasopressin; ADH)

 It acts on 2 types of receptors: V1 and V2:


- V1: present in vascular sm ms → VC and spasm.
- V2: present in the renal collecting tubules → ↑ water reabsorption.

Therapeutic uses
 Desmopressin: synthetic long acting analog given by nasal administration in:
- Diabetes insipidus (cranial type only): its action on renal V2 receptors is 3000
times more potent than on vascular V1 receptors.
- Nocturnal enuresis: by reducing nighttime urine production.
- Hemophilia: because it stimulates hepatic synthesis of factor VIII
and endothelial cells to secrete von Willebrand factor.

 Terlipressin (Glypressin): used by i.v.i. to control acute variceal bleeding.


Octreotide, vasopressin, and terlipressin have been shown to have efficacy in the
control of acute variceal hemorrhage. Terlipressin, however, is the only
medication that has been shown to improve patient survival.

Adverse effects
- Facial pallor and hypertension due to cutaneous VC.
- Coronary spasm.

▌Oxytocin

It causes milk ejection from the breast and uterine contraction at labor.

Therapeutic uses
 Induction and maintenance of labor (10-20 units by i.v.i).
 Control of postpartum hemorrhage.
 To stimulate milk secretion in nursing mothers (nasal spray)

Adverse effects and contraindications


- Rupture uterus in cephalopelvic disproportion (obstructed labor).
- Fetal asphyxia (from uterine spasm).
- Hypertension and cardiac arrhythmia

 
318  
 

319
Review Questions

1. Classify types of insulin; mention its indications and side effects.


2. Classify types of oral antidiabetic drugs; mention their mechanism of action,
indications and side effects.
3. Classify antithyroid drugs; mention their mechanism of action, side effects, and
precautions of each group.
4. Classify glucocorticoids; mention their therapeutic uses, common side effects
and contraindications.
5. Give full account on pharmacological regulation of bone calcium homeostasis.
6. Classify oral contraceptives; mention their mechanism of action, side effects
and causes of failure.

Short questions:
1. Insulin resistance: definition, mechanism, and management.
2. Mention uses and side effects of sulfonylureas.
3. Mention drug interactions of oral hypoglycemic drugs.
4. Mention therapeutic uses and side effects of vasopressin.
5. Mention side effects (or contraindications) of oxytocin.
6. Mention drug interactions of vit D.
7. Mention treatment of osteoporosis.
8. Mention medical treatment of obesity.
9. Mention antiestrogens and their therapeutic uses.
10. Mention antiandrogens and their therapeutic uses.
11. Mention contraindications of anabolic steroids.
12. Mention side effects (or contraindications) of contraceptive pills.
13. Mention causes of failure of contraceptive pills.

Mention lines of treatment of the following emergency conditions:


1. Diabetic ketoacidosis.
2. Thyrotoxic crisis.
3. Myxedema coma.
4. Acute addisonian crisis.
5. Acute hypercalcemia.

 
320  
Of each of the following questions,
select ONE BEST answer: 5. Which of the following
glucocorticoids is an intermediate-
1. Glucocorticoids are hormonal acting drug?
steroids: A. Cortisone
A. Having an important effect on immune B. Triamcinolone
function C. Butamethasone
B. Having principally salt-retaining D. Prednisolone
activity E. Dexamethasone
C. Having androgenic or estrogenic
activity 6. Immunosupressive effect of
D. Having hypercalcemic activity glucocorticoids is caused by:
E. Having hyperkalemic activity A. Reducing concentration of
lymphocytes and inhibiting function of
2. Correct statements about cortisol tissue macrophages and other
(hydrocortisone) include all of the antigen-presenting cells
following, EXCEPT: B. Suppression of cyclooxygenase II
A. Cortisol is synthesized from expression which results in reducing
cholesterol amount of an enzyme available to
B. ACTH governs cortisol secretion produce prostoglandins
C. Most cortisol is inactivated in the liver C. Activation of phospholipase A2 and
D. Cortisol has equal anti-inflammatory reducing prostaglandin and
and salt-retaining activity leukotriene synthesis.
E. The half-life of cortisol in the D. Activation of angiotensin-converting
circulations is normally about 60 enzyme
hours. E. Suppression of histamine release

3. Correct statements about 7. Indication of glucocorticoids


glucocorticoids include all of the include of the following EXCEPT:
following, EXCEPT: A. Chronic (Addison’s disease) and acute
A. Effects of glucocorticoids are adrenocortical insufficiency
mediated by widely distributed B. Organ transplants (prevention and
glucocorticoid receptors that are treatment of rejection –
members of the superfamily of nuclear immunosuppression)
receptors. C. Inflammatory conditions of bones and
B. Glucocorticoids have high plasma joints (arthritis, bursitis, tenosynovitis).
protein binding D. Hypocalcemia
C. Glucocorticoids have dose-related E. Gastrointestinal diseases
metabolic effects on carbohydrate, (inflammatory bowel disease)
protein, and fat metabolism.
D. Glucocorticoids have pro- 8. Indication for 1,25-dihydroxyvitamin
inflammatory effects. D3 (calcitriol) administration is:
E. Glucocorticoids have catabolic effects A. Vitamin D resistance
in lymphoid and connective tissue, B. Elevated skeletal turnover
muscle, fat, and skin. C. Hypercalcemia of malignancy
D. Hypophosphatemia
4. Which of the following E. Primary hyperparathyroidism
glucocorticoids is relatively a short-
acting drug? 9. Indication for risidronate
A. Prednisolone administration is:
B. Dexamethasone A. Failure of vitamin D formation in skin
C. Triamcinolone B. Hypoparathyroidism
D. Paramethasone C. Elevated skeletal turnover
E. Betamethasone D. Hypophosphatemia

321
E. Metastatic bone disease A. Estrogens
B. Fluorides
10. Correct statements about fluoride C. Parathormone
include all of the following, EXCEPT: D. Bisphosphonates
A. Fluoride is effective for the prophylaxis E. Calcitonin
of dental caries
B. Fluoride is accumulated by bone and 16. Which of the following is an
teeth, where it may stabilize the important effect of insulin?
hydroxyapatite crystal A. Increased conversion of amino acids
C. Subjects living in areas with naturally into glucose
fluoridated water (1-2 ppm) had more B. Increased gluconeogenesis
dental caries and fewer vertebral C. Increased glucose transport into cells
compression fractures than subjects D. Inhibition of lipoprotein lipase
living in nonfluoridated water areas E. Stimulation of glycogenolysis
D. Chronic exposure to very high level of
fluoride results in thickening of the 17. Which of the following agents
cortex of long bones and bony should be administered to achieve
exostoses. rapid control of the severe ketoacidosis
in a diabetic boy?
11. Which one of the following is most A. Regular insulin
likely to be useful in the therapy of B. Glyburide
hypercalcemia? C. Insulin glargine
A. Calcitonin D. NPH insulin
B. Glucocorticoids E. Tolbutamide
C. 1-25 dihydroxy vitamin D3
D. Parenteral infusion of phosphate 18. Which of the following is the most
E. Thiazide diuretics likely complication of insulin therapy?
A. Dilutional hyponatremia
12. Which of the following conditions is B. Hypoglycemia
an indication for the use of calcitonin? C. Increased bleeding tendency
A. Chronic renal failure D. Pancreatitis
B. Hypoparathyroidism E. Severe hypertension
C. Intestinal osteodystrophy
D. Paget’s disease of bone 19. A 24-year-old woman with type 1
E. Rickets diabetes wishes to try tight control of
her diabetes to improve her long-term
13. Which of the following drugs can prognosis. Which of the following
cause rickets in children by increasing regimens is most appropriate?
Vitamin D metabolism? A. Morning injections of mixed insulin
A. Tetracycline lispro and insulin aspart
B. Phenylbutazone B. Evening injections of mixed regular
C. Phenytoin insulin and insulin glargine
D. Ciprofloxacin C. Morning and evening injections of
E. Ibuprofen regular insulin, supplemented by small
amounts of NPH insulin at mealtimes
14. Bone resorption is accelerated by: D. Morning injections of insulin glargine,
A. Estrogens supplemented by small amounts of
B. Fluorides insulin lispro at mealtimes
C. Parathormone E. Morning injection of NPH insulin and
D. Bisphosphonates evening injection of regular insulin
E. Calcitonin
20. Which one of the following drugs
15. Osteonecrosis of the jaw may be an promotes the release of endogenous
adverse effect of: insulin?

322
A. Acarbose 25. Which of the following patients is
B. Pioglitazone most likely to be treated with
C. Glipizide intravenous glucagon?
D. Metformin A. An 18-year-old woman who took an
E. Miglitol overdose of cocaine and now has a
blood pressure of 190/110 mm Hg
21. The combination of metformin and B. A 27-year-old woman with severe
ethanol increases the risk of which of diarrhea caused by a flare in her
the following? inflammatory bowel disease
A. A disulfiram-like reaction C. A 57-year-old woman with type 2
B. Excessive weight gain diabetes who has not taken her
C. Hypoglycemia glyburide for the last 3 d
D. Lactic acidosis D. A 62-year-old man with severe
E. Serious hepatotoxicity bradycardia and hypotension resulting
from ingestion of an overdose of
22. Which of the following drugs is atenolol
most likely to cause hypoglycemia E. A 74-year-old man with lactic acidosis
when used as monotherapy in the as a complication of severe infection
treatment of type 2 diabetes? and shock
A. Acarbose
B. Rosiglitazone 26. In Graves’ disease, the cause of the
C. Glyclazide hyperthyroidism is the production of an
D. Metformin antibody that does which of the
E. Miglitol following?
A. Activates the pituitary thyrotropin-
23. Which of the following drugs is releasing hormone (TRH) receptor and
taken during the first part of a meal for stimulates TSH release
the purpose of delaying the absorption B. Activates the thyroid gland TSH
of dietary carbohydrates? receptor and stimulates thyroid
A. Acarbose hormone synthesis and release
B. Exenatide C. Activates thyroid hormone receptors in
C. Glipizide peripheral tissues
D. Pioglitazone D. Binds to thyroid gland thyroglobulin
E. Repaglinide and accelerates its proteolysis and the
release of its supply of T4 and T3
24. The PPAR-γ receptor that is E. Binds to thyroid-binding globulin
activated by thiazolidinediones (TBG) and displaces bound T4 and T3
increases tissue sensitivity to insulin by
which of the following mechanisms? 27. Methimazole reduces serum
A. Activating adenylyl cyclase and concentration of T3 primarily by which
increasing the intracellular of the following mechanisms?
concentration of cAMP A. Accelerating the peripheral
B. Inactivating a cellular inhibitor of the metabolism of T3
GLUT2 glucose transporter B. Inhibiting the proteolysis of thyroid-
C. Inhibiting acid glucosidase, a key binding globulin
enzyme in glycogen breakdown C. Inhibiting the secretion of TSH
pathways D. Inhibiting the uptake of iodide by cells
D. Regulating transcription of genes in the thyroid
involved in glucose utilization E. Preventing the addition of iodine to
E. Stimulating the activity of a tyrosine tyrosine residues on Thyroglobulin
kinase that phosphorylates the insulin
receptor 28. Though rare, a serious toxicity
associated with the thioamides is
which of the following?

323
A. Agranulocytosis 33. A 62-year-old woman presents with
B. Lupus erythematosus-like syndrome complaints of fatigue, sluggishness,
C. Myopathy and weight gain. She needs to sleep
D. Torsades de pointes arrhythmia several times a day, which is unusual
E. Thrombotic thrombocytic purpura for her. She has been taking T4 for the
(TTP) past 15 yr without significant problems
regarding her energy level. Her recent
29. A 65-year-old man with history is significant for diagnosis of
multinodular goiter is scheduled for a arrhythmia, and she is currently taking
near-total thyroidectomy. Which of the an antiarrhythmic drug. What is the
following drugs will be administered for most likely cause of her current
10–14 d before surgery to reduce the condition?
vascularity of his thyroid gland? A. Amiodarone
A. Levothyroxine B. Lidocaine
B. Liothyronine C. Procainamide
C. Lugol’s solution D. Sotalol
D. Prednisone E. Verapamil
E. Radioactive iodine
34. A 25-year-old woman presents with
30. Which of the following is a sign or insomnia and fears she may have
symptom that would be expected to “something wrong with her heart.” Lab
occur in the event of chronic overdose tests confirm hyperthyroidism. Which
with exogenous T4? of the following is a drug that produces
A. Bradycardia a permanent reduction in thyroid
B. Dry, puffy skin activity?
C. Large tongue and drooping of the A. 131I
eyelids B. Methimazole
D. Lethargy, sleepiness C. Propylthiouracil
E. Weight loss D. Thiocyanate
E. Thyroglobulin
31. When initiating T4 therapy for an
elderly patient with longstanding 35. Glucocorticoids have proved useful
hypothyroidism, it is important to begin in the treatment of which of the
with small doses to avoid which of the following medical conditions?
following? A. Chemotherapy-induced vomiting
A. A flare-up of exophthalmos B. Essential hypertension
B. Acute renal failure C. Hyperprolactinemia
C. Hemolysis D. Parkinson’s disease
D. Overstimulation of the heart E. Type II diabetes
E. Seizures
36. A patient presents with pain and
32. A 27-year-old woman underwent stiffness in his wrists and knees. The
near total thyroidectomy. She was stiffness is worse first thing in the
started on levothyroxine. What morning. A blood test confirms
hormone is produced in the peripheral rheumatoid arthritis. You advise a short
tissues when levothyroxine is course of steroids. Which one of the
administered? following is the most potent anti-
A. Methimazole inflammatory steroid?
B. T3 A. Cortisol
C. T4 B. Dexamethasone
D. TSH C. Fludrocortisone
E. FSH D. Prednisone
E. Triamcinolone

324
37. A 34-year-old woman with 40. Which of the following drugs is
ulcerative colitis has required long- most likely to lower patient’s serum
term treatment with pharmacologic PTH concentration?
doses of a glucocorticoid agonist. A. Calcitriol
Which of the following is a toxic effect B. Cholecalciferol
associated with long-term C. Furosemide
glucocorticoid treatment? D. Gallium nitrate
A. A lupus-like syndrome E. Risedronate
B. Adrenal gland neoplasm
C. Hepatotoxicity 41. The patient began therapy with a
D. Osteoporosis nasal spray containing a protein that
E. Precocious puberty in children inhibits bone resorption. The drug
contained in the nasal spray was which
38. Which of the following drugs is of the following?
most useful for the treatment of A. Calcitonin
hypercalcemia in Paget’s disease? B. Calcitriol
A. Fluoride C. Cinacalcet
B. Hydrochlorothiazide D. Cortisol
C. Pamidronate E. Teriparatide
D. Raloxifene
E. Teriparatide
Answers
39. The active metabolites of vitamin D
act through a nuclear receptor to 1A 10 C 19 D 28 A 37 D
produce which of the following effects? 2E 11 B 20 C 29 C 38 C
A. Decrease the absorption of calcium 3D 12 D 21 D 30 E 39 D
from bone 4A 13 C 22 C 31 D 40 A
B. Increase PTH formation 5B 14 C 23 A 32 B 41 A
C. Increase renal production of 6A 15 D 24 D 33 A
erythropoietin 7D 16 C 25 D 34 A
D. Increase the absorption of calcium 8D 17 A 26 B 35 A
from the GIT 9E 18 B 27 E 36 B
E. Lower the serum phosphate
concentration

325
 

326
   
 
Part 1
1: CNS
S Stimullants

Classiffication:

█ RES
SPIRATOR
RY STIMUL
LANTS (A
ANALEPTIC
CS)

Definittion: drugss that stimu


ulate the d
depressed respiratory
y center (R
RC).

Classiffication:
 Spe
ecific analleptics:
 Naloxon and
a nalorph hine → usedd to treat opiates
o res
spiratory d
depression.
 Flumazenil → used to
o treat ben
nzodiazepin nes toxicity
y.

 Non
n-specific
c analeptic
cs:
 Direct RC stimulatio
on: e.g. Xa
anthines (c
caffeine, theophyllinne) – Etham
mivan -
Heptamino
ol. They an
ntagonize tthe GABA--mediated RC depresssion.
 Indirect RC
C stimulation (reflex)): e.g. Nico
otine and lobeline. T
They stimulate RC
indirectly through
t stiimulation o
of the chem
morecepto
ors in the ccarotid bod
dy.
 Both (direcct and indirect): Nikeethamide - Doxapram
m

Therap es: all case


peutic use es of RC ddepression
n (postanaesthetic, C
CNS depre
essants,
COPD and some premature e babies)

327
Adverse effects
– Tachypnea, tachycardia and hypertension.
– High doses can cause convulsions.

Contraindications
– Epilepsy (to avoid CNS stimulation).
– Severe hypertension, arrhythmia or IHD (to avoid cardiac arrhythmia).
– Thyrotoxicosis.
– Severe bronchial asthma.

█ PSYCHOMOTOR STIMULANTS

Definition: drugs that induce euphoria with increased motor activity.

Classification

 Amphetamine and related drugs: see ANS

 Cocaine
– Cocaine inhibits tissue uptake of catecholamines.
– Behavioral effects of cocaine are very similar to amphetamine.
– Cocaine is used topically as a local anesthetic eye drops.
 Xanthines: see respiratory pharmacology.

 Antidepressants: see later.

█ PSYCHOTOMIMETIC DRUGS (HALLUCINOGENS)

Definition: drugs that affect thought, perception and mood with no effect on
motor activity. They have no clinical applications but important for drug abuse.

Examples and Mechanisms


 LSD (Lysergic acid diethylamide): It stimulates central 5-HT receptors and
affects catecholamine action → severe hallucinations and delusions resembling
acute schizophrenia.

 Cannabis (hashish, marijuana): It acts on cannabinoid receptors in the CNS


(CB1) causing loss of judgment of time & place and dream-like state. It does not
cause physical dependence but psychological dependence. CB2 receptors are
found in peripheral tissue and immune system.

 Phencyclidine: It stimulates -opioid receptors and blocks the NMDA receptors


and may interact with other neurotransmitter systems.

328
Part 2
2: Ana
algesics

Definittion: drugss that reliev


ve or decre
ease pain sensation.
s

▌Class
sification of
o analges
sics:
 Opiioid (narco
otic) analg
gesics
 NSA
AIDs (see chapter 4).
 Ana ntipyretics: e.g. para
algesic an acetamol, Dipyron
D & Nefopam.
N
 Dru f speciffic painfull conditions e.g. ca
ugs used for arbamazep
pine for trig
geminal
neu
uralgia, ergotamine fo
or migraine
e.

█ OPIOID ANAL
LGESICS

 Theey are dru


ugs that decrease p
pain sensa
ation
withhout loss of o conscio ousness annd can ind duce
phyysical depe endence.
 Opiiates: are drugs derived from m opium plant
p
(Pappaver som miniferum).. They in nclude ma ainly
morrphine, cod deine and papaverin e.
 Opiioids: are drugs
d with
h morphinee-like activity.
 Opiiopeptins:: are endo ogenous p peptides with
w
opiooid-like acttivity e.g. en
ndorphins & enkepha alins
 Narrcotics: arre drugs th hat producce narcosiss i.e.
drowsiness or o stupor, with ana lgesia (stu upor
mea ans marke ed impairm ment, but not comp plete
losss of consciousness). These dru ugs are usu ually
adddictive. It is a legal, not a mediccal, term.

Classiffication off opioid drrugs

329
Mechanism of action
 Opioids such as morphine are believed to interact with three major receptors (μ,
δ, κ). Each opioid receptor has distinct subtypes (e.g., μ1, μ2). All three major
receptors are present in high concentrations in the dorsal horn of the spinal cord.
 Interaction with μ-receptors contributes to supraspinal and spinal analgesia,
respiratory depression, sedation, euphoria, decreased GI peristalsis, and
physical dependence (addiction).
 The significance of interaction with κ-receptors is unclear, but it may contribute
to analgesia (through inhibition of release of substance P at dorsal horn).

█ NATURAL OPIOID AGONISTS

1. Morphine

Pharmacokinetics
 Oral absorption: good (bioavailability is 25% due to significant first-pass effect).
However, the analgesic effect is greater when the drug is administered parentrally
 t½ : 4-5 hrs.
 Metabolism: in the liver by conjugation leading to inactive metabolites.
 Excretion: renal (90%) – bile (10% as conjugated morphine).

Pharmacological effects

 Analgesia
 Dose-dependent analgesia (sensory & emotional): consciousness is not
lost and the patient can still locate the source of pain. Analgesia may be
associated with euphoria and decreased anxiety.
 Analgesia results from direct activation of μ and δ receptors in the spinal cord
and possibly higher centers (thalamaus) leading to:
– Activation of descending inhibitory pathways.
– ↓ release of substance-P in pain transmission neurons in the spinal cord.
 The psychic effect results from ↓ NA release in some CNS areas leading to
decrease anxiety and reaction of the patient to pain.
 Morphine and other exogenously administered opioids may also have some
action on peripheral inflamed tissue.

N.B.Morphine can treat all types of pain except itching. Why?


a) Because morphine stimulates histamine release → ↑ itching.
b) Itching is different from pain sensation and has different receptors and
centers.

330
 Euphoria (large doses produce dysphoria).
 Miosis: due to central stimulation of Edinger-Wistphal nucleus. Severe miosis is
indicative of toxic doses.
 Respiratory center depression. This RC depression leads to CO2 retention and
cerebral VD → ↑↑ intracranial tension.
 Cough suppression.
 Vagal stimulation.
 Nausea & vomiting: due to stimulation of chemoreceptor trigger zone (CTZ).

CVS effects:
 Orthostatic hypotension: due to (a) Histamine release; (b) vagal stimulation.
 Bradycardia: due to vagal stimulation.

Smooth ms (Spasmogenic effects):


 Bronchoconstriction: due to: (a) Vagal stimulation. (b) Histamine release.
 Constipation: due to (a) spasmodic non-propulsive contractions of intestinal
smoothms& decreased peristalsis; (b) ↑ intestinal water absorption.
 Spasm of the sphincter of Oddi → ↑ biliary pressure.
 Feeling of urgency with difficult micturition due to ↑ detrusor muscle tone with
spasm of the internal urethral sphincter.
 Uterus: prolongation of labor by unclear mechanism.

Therapeutic uses

 Analgesia: for severe pain e.g. acute MI, cancer, surgery, etc.
 Acute pulmonary edema (cardiac asthma).Why?
– ↓ stress & anxiety of the patient.
– Venodilatation → ↓ VR & preload → ↓ pulmonary congestion.
– Decrease tachypnea caused by the CNS response to hypoxic drive (due to its
depressant effect on RC).

 In anesthesia:
– As adjuvant to anesthetic agents (preanesthetic medication).
– Regional anesthesia (epidural) to achieve long lasting analgesia by its effect
on the spinal cord.

 In severe colic: (morphine combined with atropine). Why?


– Because morphine is spasmogenic and atropine is spasmolytic.
– To counteract muscarinic effects caused by excessive vagal stimulation (e.g.
bronchoconstriction and bradycardia).

331
Preparations and doses
 Morphine sulphate: 10 mg s.c. or i.m. In acute MI it is given 5 mg i.v.
 Intrathecal (epidural) injection: produce long lasting analgesia which is useful for
critically ill patients at risk of RC depression.
 Sustained release preparations &transdermal patches are available.

Adverse effects:

CNS  Tolerance & physical dependence (addiction) with prolonged use:


– Physical dependence can occur within 24 h if given /4 h.
– Tolerance may occur to analgesia and euphoria but not to
respiratory depression.
 ↑↑ intracranial tension.
 RC depression: the most important effect and is dose-dependent.
Resp  Bronchoconstriction.
CVS  Postural hypotension
GIT  Nausea, vomiting, and constipation.
 Increased biliary tract pressure and biliary colic.
Genito-  Urine retention especially in patients with enlarged prostate.
Urinary  Prolongation of labor.
Eye  Miosis is a consistent finding in morphine addiction.

Contraindications & precautions


 Head injury & increased intracranial pressure: Morphine causes respiratory
depression & CO2 retention. The ↑ CO2 causes cerebral VD and ↑ intracranial
tension.
 Respiratory depression.
 Bronchial asthma: Morphine causes bronchoconstriction due to (a) vagal
stimulation; (b) histamine release. It also causes RC depression.
 Biliary colic & gallstones: due to spasm of the sphincter of Oddi → ↑ biliary
pressure.
 Senile enlarged prostate: Morphine ↑ detrusor muscle tone with spasm of the
internal urethral sphincter → feeling of urinary urgency with difficult micturition.
 Hypotension and hypovolemia: because morphine causes postural
hypotension.
 Hepatic damage: Due to: (a) morphine is metabolized by the liver; (b) morphine
increases the risk of hepatic encephalopathy due to marked CNS depression.
 Hypothyroidism and adrenal insufficiency (Addison’s disease). Why?
Because those patients have prolonged and exaggerated response to morphine.

332
 Undiagnosed acute abdominal pain: Morphine masks the pain (which may be
dangerous e.g. appendicitis) and interferes with the correct diagnosis.
 Infants and old patients: are more susceptible to respiratory depression.

▌Chronic opioid toxicity (addiction)

 There are behavioral changes, constipation, itching &miosis.


 Sudden withdrawal (abstinence syndrome):
– Consists of: irritability, nervousness, tremors, hypertension & ms cramps starts
after 6-10 hrs from the last dose - peak effect at 48 hrs - gradually subsides
over 5-10 days.
– Mechanism: chronic administration of opioids ↓↓ endogenous production of
endorphins and NA. Following sudden withdrawal, there is an immediate
deficiency of endogenous opioids with rebound ↑ of NA release.

Treatment of chronic morphine addiction:


– Gradual withdrawal of morphine with substitution by methadone, then
gradual withdrawal of methadone.
– Clonidine: to stimulate central α2 receptors and ↓ NA release.
– Sedatives: e.g. diazepam.

▌Acute opioid toxicity

Manifestations:  Coma with depressed respiration, miosis, and shock.


 Death occurs from respiratory depression.
Treatment:  Gastric lavage.
N.B.
 Establish a patent airway
Opioid blockers are
and artificial respiration if
indicated in acute morphine
needed.
toxicity but they are
 Opioid antagonists: absolutely contraindicated
– Naloxone: pure opioid in chronic morphine
antagonist and can addiction because they
reverse RC depression precipitate severe withdrawal
within minutes (0.4 - 0.8 syndrome.
mg i.v. for 2-3 doses).
– Nalorphine: it is mixed agonist-antagonist (partial blocker).

N.B.
 The duration of opioid antagonists is shorter than morphine. The patient should
be watched carefully because he may go back into coma.
 Care should be taken to avoid withdrawal syndrome.

333
2. Codeine

Similar to morphine with the following differences:


 Has greater oral bioavailability (60%) due to less first-pass effect.
 The analgesic potency is 20% of morphine, but more potent cough suppressant
 It has little euphoric effect.
 Most of the systemic effects of codeine are due to conversion in the liver into
morphine by demethylation (codeine is methylmorphine).

Therapeutic uses
– Analgesic for mild to moderate pain (usually combined with paracetamol).
– As central antitussive (see chapter 7).

Adverse effects: (less than morphine)


– Constipation
– RC depression and addiction liability but it is fewer than morphine.

Morphine Codeine
Oral bioavailability: 25% 60%
Analgesic effects: Strong Weak (20%).
Antitussive effect: Weak Strong
Uses: Mention its 4 uses Analgesic and antitussive

█ SYNTHETIC AND SEMISYNTHETIC OPIOID AGONISTS

1. Heroin and hydromorphone

 They are semisynthetic opioids (heroin is diacetylmorphine).


 They are more potent than morphine but with rapid onset and shorter duration.
 They are not used clinically because they are highly addictive.

2. Meperidine (Pethidine)

Synthetic opioid similar to morphine with the following differences:


 Better absorbed orally and has greater bioavailability than morphine.
 The analgesic potency is 10% of morphine.
 It is used as analgesic alternative to morphine in the following cases:
– Inferior MI because in this case the patient usually has bradycardia.
– It is preferred than morphine during labor because it has short duration
and doesn't prolong labor (little or no spasmogenic action).

334
Adverse effects
– It causes RC depression and addiction liability but weaker than morphine.
– It causes histamine release and bronchoconstriction
– It has weak atropine-like actions → dry mouth, tachycardia, etc.
– It has No GIT, No antitussive, and No vagal stimulant effects.

Morphine Meperidine
Chemistry Natural opioid Synthetic opioid
Bioavailability 25% Greater (50%)
Analgesic effect Strong Weak (10% of morphine)
Spasmogenic effects Present Absent
Autonomic effects Vagal stimulation Atropine-like action
Uses Mention its 4 uses Analgesic only

3. Fentanyl and alfentanil

 They are synthetic derivatives of meperidine. They are the most potent and the
shortest duration opioid agonists.
 They are used as analgesic in severe pain (as long-acting transdermal skin
patch). A transdermal fentanyl 12 microgram patch equates to approximately 30
mg oral morphine daily.

4. Methadone

Synthetic opioid similar to morphine with the following differences:


 The analgesic effect is equal to morphine.
 Has longer duration of action than morphine (t1/2 is 24h).
Uses:
 As analgesic.
 Treatment of chronic opiate addiction and heroin users:
– It can satisfy the craving needs of the patient with less addictive features.
– Methadone withdrawal symptoms are less severe than other opioids.

5. Tramadol

 It has two different mechanisms. First, it binds to the μ-opioid receptor. Second,
it inhibits the reuptake of serotonin and NA.
 Uses: as analgesic for moderate to severe pain, especially musculoskeletal pain
 Adverse effects: It has relatively fewer side-effects than most opioids (but
addiction can occur). It may induce seizures in epileptic patients.

335
6. Diphenoxylate and loperamide (see chapter 8)

█ SEMISYNTHETIC MIXED AGONISTS-ANTAGONISTS (partial agonists)

Nalorphine – Nalbuphine – Pentazocin – Butorphanol

 All these drugs have agonist activity on  receptors and antagonist or partial
agonist activity on  receptors.
 They are used as analgesics alternative to morphine but their analgesic activity
and respiratory depression are less marked than morphine.
 All these drugs (except nalbuphine) increase
systemic and pulmonary vascular resistance N.B.
leading to ↑ cardiac load, so they are, thus,
For opioid analgesics,
contraindicated to relieve pain of acute MI.
potency of the analgesic
 They can lead to withdrawal symptoms if should be considered more
given to opioid addict patients. important than efficacy
because respiratory
depression is dose-
█ SYNTHETIC FULL ANTAGONISTS dependent.

Naloxone and naltrexone

 They are competitive blockers of all opioid receptors.


 Naloxone is given i.v. and has short t½ (~1h) but naltrexone could be
administered orally and has longer t½ (~48 h).
 They can precipitate severe withdrawal syndrome if administered to opioid-
addict patient.

Therapeutic uses of naloxone


 Acute opioid toxicity: given i.v., the adverse respiratory and CVS effects of
opioids are reversed within 1-2 min and lasts for 1-2 hrs.
 It is given during labor to mothers who received opioids to prevent neonatal
respiratory depression, or it can be given to the neonate via the umbilical vein.

336
█ NON
N-OPIOID ANALGES
SICS

1. Ace
etaminophen (Para
acetamoll)

Pharm
macokinetics
 Abssorption is complete and rapid from GIT with
w peak levels afteer 30 min.
 Mettabolism: liver by conjugation
c n. At highh doses, it is convverted into
o toxic
mettabolite (N
N-acetyl-be ne) that is responsible for hepaatotoxicity..
enzoquinon
 Exc
cretion: maainly renal.

Mecha
anism & Ph
harmacolo
ogical effe
ects
 It iss a selectivve Cox III inhibitor sso it inhibitts PGs syn
nthesis in tthe brain only
o and
hass analgesic & antip pyretic acttions witho
out effects on the eenzymes that t are
respponsible fo or synthesis of perip
pheral PGss and so it has no a nti-inflammatory
action.
 It ha
as little orr No effects on the C
CVS, GIT, re
espiratory or platelett functions
s.

Therap
peutic use
es
As anaalgesic andd antipyretic when aspirin is s contraindicated (ee.g. patien
nts with
peptic ulcer, hemmophilia, etc).
e Aceta
aminophen
n can be administer
a red in preggnancy
with greater safetty than asp
pirin.

Advers
se effects
 At therapeuttic doses: acetami nophen iss
well-tolerated but may cause:
c
– Skin rash h& drug fever (a as allergic c
reactions)..
– Long term m use may lead
l to ren
nal failure.

 In ttoxic dose
es: dose-d
dependen
nt hepato--
toxiicity (centtrilobular necrosis): It occurs
s
with
h large dosses (about 15 gm in a adults and
d
m in childrren)
4 gm

Mec
chanism of
o hepatottoxicity
 A
Acetamino
ophen is converted
d to toxic
c
metabolite
e (N-acety
yl-benzoquuinone) inn
tthe liver that need
ds detoxifiication by
y
reduced glutathione
e.

337
 When glutathione store is depleted, the toxic metabolite binds covalently to
cellular proteins producing hepatocellular damage.
 Clinical symptoms of toxicity (e.g. vomiting) occur within 24 hrs but signs of
hepatic damage (e.g. jaundice) occur after 2-6 days.

Treatment of toxicity
The 20 hour IV protocol of
 Gastric lavage with activated charcoal. acetylcysteine
 Sulfhydryl donors (acetylcysteine) to
– First, administer an initial
restore hepatic glutathione. It must be
loading dose of 150 mg/kg IV
started within 8 hours of toxicity. over 60 minutes.
 Hemodialysis: better within the first 12
– Next, administer 12.5 mg
hrs after ingestion. /kg per hour IV for 4 hours.
– Finally, administer 6.25 mg
Nefopam (Acupan) /kg per hour IV for 16 hours.

Mechanism and effects


 It is a central analgesic without antipyretic or anti-inflammatory activity. It is
more potent than NSAIDs.
 The analgesic mechanism is unclear but may be related to inhibition of many
transmitters reuptake or blocking central voltage-gated Na+ channels.
 Nefopam is also used to combat severe hiccups.

Adverse effects
– Precipitation of epileptic convulsions in patients with epilepsy.
– Weak atropine-like actions: dry mouth, urine retention, etc.

Contraindications: history of epilepsy

Dipyrone (Novalgin)

Mechanism and effects


 Analgesic antipyretic that is more potent than aspirin. It has no anti-
inflammatory action.
 Its use was restricted in many countries because of risk of agranulocytosis
which is not dose-dependent.

Adverse effects
– Agranulocytosis: reversible in 10 days after stoppage of the drug but lethal in
10% of cases.
– Allergic reactions and anaphylaxis.
– It can trigger bronchoconstriction in patients with Asthma.

338
Part 3: Sedative-Hypnotic Drugs

 These are drugs that cause sedation and relieve anxiety (anxiolytics), or can
induce sleep. They are used primarily to treat anxiety and insomnia.
 Because there is considerable chemical variation within the group, these drugs
are classified based on their clinical uses rather than on chemical structure.

Drugs with main use as sedatives: Drugs with main use as hypnotics:
 Benzodiazepines  Barbiturates
 Buspirone  Ramelteon
 Chloral hydrate

1. Benzodiazepines

 Benzodiazepines (BDZ) have a great margin of safety over previously available


sedative–hypnotic agents (e.g., barbiturates).
 Most benzodiazepines have qualitatively similar therapeutic actions but differ in
their relative lipid solubility, metabolism, and elimination half-life.

Classification
 Short acting (t½ < 5h): midazolam – triazolam
 Intermediate acting (t½ 5-24 h): alprazolam – lorazepam - clonazepam
 Long acting (t½ > 24 h): diazepam – clorazepate - flurazepam

Pharmacokinetics
– Oral absorption is good and rapid. Highly lipid soluble drugs (e.g., midazolam,
triazolam) have fast onset of action.
– Long acting drugs are metabolized by oxidation (CYP450) into active
metabolites giving them long duration of action (e.g. diazepam).
– Short acting drugs are metabolized by conjugation into inactive metabolites
followed by renal clearance.
– In a patient with liver dysfunction, lorazepam and oxazepam, which are metabo-
lized extrahepatically, are less likely to cause excessive CNS depression.

Mechanism of action
 BDZ have special receptors in the CNS and peripheral tissue.
 By acting on these receptors, BDZ cause allosteric modulation of GABA action
on GABAA receptors resulting in ↑ Cl- conductance and hyperpolarization.
 Six BDZ receptor subtypes have been discovered; subtype 1 is the most widely
expressed and mediates most of the effects of BDZ.

339
Pharm
macologica
al effects
 Redduction off anxiety (anxiolytic
( effect) in
smaall dose producing
p calming effect in
mann & tamingg effect in animals
a
 Hyp ect: in high
pnotic effe her doses.
 Cenntral skele
etal musc cle relaxattion: this
is u
useful sinc
ce increas sed ms to one is a
commmon feature in anx xiety and mmay lead
to h
headache and
a ms pa ain.
 Antticonvulsa
ant effect.
 Acuute amnes sia: after high doses..

peutic use
Therap es
 Anx
xiety disorrders: e.g..
– A
Acute anxiety.
– G
Generalizeed anxiety disorders (GAD).
– S
Social phoobia (social anxiety d
disorder).

BDZ Z are effec


ctive for th
he short-tterm mana agement (<<6 weeks)) of these anxiety
diso
orders (seelective se erotonin-re
euptake inhibitors [S
SSRIs; seee later] are now
connsidered thhe first-lin
ne choicess for the long-term m manageement of anxiety
diso
orders.

 Insoomnia: BDDZ that ha


ave a rapid
d onset an
nd sufficien
nt durationn are wide
ely used
(e.g
g., temazep
pam).

 Antticonvulsaants: Lorazepam (a and diazep pam), give


en by i.v. may be used
u for
initial treatm
ment of status
s ep
pilepticus and dru ug-inducedd seizures. The
devvelopment of toleranc
ce preclud
des their long-term us
se.

 Shoort surgica dures: Sho


al proced orter acting
g benzodia
azepines ((e.g., mida
azolam)
are preferred before durring surgerry or endos
scopy.
 Pre
eanesthetic medicattion.

Advers
se effects
– Seddation, meemory distturbance, dull atten ntion (interfere with leearning ab
bility).
– Tole
erance an nd physica al depende ence (treatted by gradual withd drawal).
– Rebbound inso omnia afte er disconti nuation.
– Hanngover: a state of psychomot
p or depress sion occurrs in the fo
ollowing day after
the use of long acting drugs (i.e. re esidual efffect).
– Apnnea after ra
apid i.v. injjection (flu
umazenil is the antido
ote)

340
Fluma
azenil_
Flumazzenil is a competitiv
c ve antago nist at be
enzodiazep
pine recepttors. It is used to
prevent or reverrse the CN NS effectss from beenzodiazep
pine overd
dose or to o speed
recovery from th he effects of benzo
odiazepines s used in anestheticc and dia agnostic
proced
dures.

Precautions: avvoid BDZ use


u in the ffollowing conditions:
c :
– Drivvers and machine
m woorkers neeeding high attention.
– Pre
egnancy an nd lactatio
on: fetal m
muscular hypotonia anda impairred suckling may
occ
cur.
– Hep patic ence
ephalopathy.
– Resspiratory depression
d .
– Com mbination with otherr CNS dep ressants e.g.
e alcohol.

2. Bus
spirone

 It iss partial agonist on 5--HT1A rece


eptors in th
he midbrainn.
 Sele ective anxiolytic with
hout hypno otic or mus
scle relaxant effect.
 It su uppresses anxiety affter a long delay (2 or more weeks).
 It iss the anxiolytic of cho
oice in the elderly.
 No liability forr drug dependence.
 Adv verse effects include e dizzinesss, headach
he, tachycaardia and nnervousne
ess.

3. Barrbiturates
s

 Barrbiturates have
h been largely re
eplaced by
y the more
e safe benzzodiazepin
nes and
the SSRIs, forr the treatm
ment of an xiety and sleep
s disorders.
 Pheenobarbitaal is a long acting bbarbituratee (~6-8h) used
u as ann anticonv
vulsant;
opental is ultrashort agent (~15
thio 5-20 min) used
u as an
n i.v. generral anesth
hetic.

Mecha
anism of action
a
 Barrbiturates have eithe
er GABA-llike action
n
OR enhance the
t effects s of GABA
A at GABAA
eptors ressulting in ↑ Cl- co nductance
rece e
and
d hyperpolaarization.
 Thee action off barbitura
ates is non
n-selective
e
i.e. increasing
g the dos
se of barb biturates →
genneralized CNS and me epression.
edullary de

341
Therap
peutic use
es
 As sedative and hypn
notics in the treatm
ment of in
nsomnia. They are largely
replaced by benzodiaze
b epines.
 Phe al is used in the treattment of grand mal epilepsy.
enobarbita e
 Phe al is micro
enobarbita osomal en
nzyme ind
ducer. It is
s used in the treatm
ment of
hypperbilirubinaemia in neonates
n (p
physiologic
cal jaundicce) to activvate liver enzymes
(glu
ucuronyl traansferase) and fasten
n metaboliism of bilirubin.
 Thio
opental iss used as short
s naesthetic in short prrocedures..
i.v. an

Advers
se effects
– Phyysical depe
endence.
spiratory and myo
– Res ocardial
pression (in acute toxicity)
dep
enobarbital
– Phe is hepatic
crosomal enzyme inducer.
mic
– Pheenobarbital may increase
porrphyrin synthesis.
s It can
preccipitate th
he sympttoms of
acu
ute intermitttent porph
hyria.

4. Ram
melteon

 Ram
melteon is prescribe
ed for patiients who have difficulty fallinng asleep.. It is a
sele
ective agonist at me
elatonin MMT1 and MT2M recepttors that aare involved
d in the
promotion of sleep and that mainttain the noormal circadian rhyth m.
 Advverse effects include
e dizzinesss and fatig
gue.

5. Chloral hydrrate

 It iss one of thee oldest hyypnotic druugs. It is an


n alcohol derivative.
d
 It prroduces hyypnosis in 30 min annd last for 6 hrs.
 It iss used maainly as a hypnotic i n children n and elderly before short surgical or
den
ntal proced
dures but itts use now
w is very lim
mited.

342
Part 4
4: Sk
keletal Muscle
M Relaxan
nts

Classiffication
 Dru
ugs act on n brain hig ers: e.g. ca
gher cente arisoprodo
ol, benzodi azepines, general
ane
esthetics, anticonvuls
a sant drugs , and antip
parkinsonia
an drugs (ssee later).
 Dru
ugs act on
n neurona al cord: e..g. tizanidine and
al transm ission in the spina
bac
clofen.
 Dru
ugs act onn the neu on: e.g. ne
uromuscullar junctio euromuscuular blocke
ers and
boto
olinum tox
xin.
 Dru
ugs act dirrectly on the
t muscle anisms: e..g. dantrole
e contracttile mecha ene.

1. Carrisoprodo
ol

 Carrisoprodol – (and its metabolitee mebrobamate) – are centraally acting skeletal


musscle relaxxants. Their mechan nism is unclear but seems to be sim milar to
barrbiturates.. They bind
d to GABAAA recepto ors in the CNS
C leadi ng to antia
anxiety,
antiiconvulsan
nt, and ske
eletal musc
cle relaxatio
on.
 Thee drug and its metabo
olite have a
abuse potential, so so
ome countrries limit its
s use.

343
2. Baclofen

 Baclofen is a GABA derivative that selectively stimulates GABAB receptors in the


spinal cord → ↓ release of excitatory transmitters.
 It is used as a skeletal muscle relaxant in neurological spastic conditions.

3. Tizanidine

 Tizanidine is a centrally acting α2 agonist with greater effect on presynaptic α2 in


the spinal cord, so it inhibits neurotransmission and reduces muscle spasm with
minimal effect on blood pressure.

4. Dantrolene

 Dantrolene inhibits Ca2+ release from the sarcoplasmic reticulum of skeletal ms


cells. It acts directly on the muscle contractile mechanisms.
 It is used for emergency management of malignant hyperthermia (1 mg/kg i.v.)

5. Botulinum toxin (Botox)

 Botulinum toxin is a neurotoxic protein produced by the bacterium Clostridium


botulinum. It inhibits the release of ACh from motor nerve terminals leading to
skeletal muscle paralysis.
 It is injected locally to treat local muscle spasm e.g. in cervical dystonia
(spasmodic torticollis), and blepharospasm (uncontrolled muscle contraction or
twitch of the eyelid). It is also used for cosmetic reduction of facial wrinkles.

6. Neuromuscular blockers: see ANS.


Mephensin

Therapeutic uses of skeletal muscle – Mephensin is a selective


relaxants inhibitor of polysynaptic
excitation of the spinal motor
 Neurological spastic conditions such as neurons through stimulation of
multiple sclerosis, back pain, and spine glycine receptors in the spinal
injuries. cord.
 Dantrolene is used for emergency man- – It was historically used as
agement of malignant hyperthermia. antidote for treatment of
 Botolinum toxin is injected locally to relive strychnine poisoning but its
blepharospasm, and for cosmetic clinical use now is very limited
because of serious side
reduction of facial wrinkles.
effects including respiratory
 Neuromuscular blockers are used to depression and hemolysis.
produce muscle relaxation during surgical
procedures.

344
Part 5
5: Antiiepileptiic Drugs
s

█ Basiic inform
mation

 Epilepsy, a chronic disease, occurs in


appproximatelyy 1% of the popu lation. The
cauuse of most cases of epilepsy iss unknown n,
alth
hough som me people develop e epilepsy as
a
the result of brain
b injury
y, stroke, b
brain tumor,
andd drug toxicity. Ge enetic muttations arre
linke
ed to a sm
mall proportion of the e disease.
 Epilleptic seizu
ures resultt from exc
cessive and
d
abn
normal cortiical nerve cell
c activity in the brain
n.
 Thee diagnosiss typicallyy involves ruling outt other conditions thhat cause similar
neuurological symptoms. This m may be confirmed d by braain imagin ng and
elec
ctroencephhalogram (EEG)
( but a normal te
est does not excludee the condition.
 Antiepileptic drugs
d (AED
Ds) are effe
ective for about
a 80%
% of these patients. Lifelong
L
trea
atment mayy be neces
ssary.
 It m
may take we
eeks to es
stablish ade
equate dru
ug plasma levels and
d to determ
mine the
ade
equacy of therapeutiic improve
ement. Lac mpliance iis respons
ck of com sible for
manny treatme
ent failures.
 AED
Ds are mo
ost effectiive and ha
ave the le
east adverse effectts when they are
use
ed as mono
otherapy.
 Add
dition or withdrawal
w of any drrug should
d be gradual, becauuse seizurres may
occ
cur on withdrawal.
 Som
me AEDs are
a terato
ogenic; thiis may ca
all for the reduction or termina
ation of
therrapy during
g pregnanc
cy.

Classiification of
o epileps
sies:

Epilepssies are characteriz


c zed by eitther focall or generalized ab bnormal neuronal
n
discharrges. Drugg selection
n, based o
on seizure classifica
ation, is lissted below
w in the
order o
of general choice.
c

Simple Loc
calized disc
charge; co
onsciousne
ess is 1 . Carbama azepine
partial not altered. 2 . Lamotriggine
seizures
Partial

3 . Valproic acid
Complex x Loc
calized disc
charge tha
at becomes
s
partial widespread; aaccompanied by loss
s of
con
nsciousnesss.

345
Tonic–clo
onic Draamatic convvulsions with
w either jerking 1 . Valproic acid
(grand ma
al) of the extremiities or rigidity of the entire 2 . Lamotriggine
boddy; accomppanied by loss of 3 . Carbama azepine
connsciousnesss.
Generalized

Absence Suddden onsett of altered


d consciouusness 1 . Ethosuximide
seizures

(petit mal)) thatt lasts 10–4


45 secondds, with up to 2 . Valproic acid
hun
ndreds of sseizures peer day; beg
gins in 3 . Clonazeppam
chilldhood or adolescen nce.

Myoclonic Lighhtning-like
e jerks of one or more e 1 . Valproic acid
syndromes extrremities occcurring sin
ngly or in bursts
b 2 . Lamotriggine
of up
u to a hun ndred; accompanied by
alte
eration of c
consciousn ness.

 Status epilep
pticus:
Pro
olonged se
eizure (>20 0 min) of a
any of the types prev viously desscribed; th
he most
com
mmon is liffe-threaten
ning generaalized tonic–clonic sttatus epileepticus.

1. Dip
phenylhyd
dantoin (P
Phenytoin
n)

Mechaanism: it blocks Na a+, K+ andd Ca2+


channeels in the brain (and
d heart) le
eading
to deccrease prropagation n of abn normal
impulse
es. It prooduces so ome degrree of
drowsin
ness.

Therap
peutic use
es
 Parttial and geeneralized seizures
s
 Stattus epileptticus: it sho
ould be givven i.v. in the
t form of fosphenyytoin (prod
drug).
 Ven
ntricular arrrhythmia.

Advers
se effects
– CNS S: Nystagmus, diplo opia, ataxia
a.
– Heppatotoxicityy.
– Miccrosomal enzyme ind duction.
– Bonne marrow depressio on & Mega loblastic anemia
a e to ↓ folicc acid).
(due
– Teraatogenicityy: craniofacial abnorm
malities.
– Gingival hype erplasia: 2ry
2 to incrreased ex xpression of platelett derived growth
facttor (PDGF)).
– Lym
mphadenop pathy.

346
2. Carbmazepine (Tegretol)

Mechanism: it blocks Na+ channels & ↓ excitability of cortical neurons.

Therapeutic uses
 Partial and generalized seizures (grand mal epilepsy).
 Trigeminal neuralgia.

Adverse effects
– CNS: diplopia & ataxia.
– Hepatotoxicity.
– Microsomal enzyme induction.
– Bone marrow depression.
– Congestive heart failure (CHF).

3. Valproic acid (Depakene)

Mechanism: it activates glutamic acid decarboxylase enzyme→ ↑ GABA synthesis.

Therapeutic uses: all types of epilepsy.

Adverse effects
– Sedation
– Microsomal enzyme inhibition
– Teratogenicity.
– Alopecia
– Pancreatitis

4. Ethosuximide (Zarontin)

Mechanism: blocks neuronal voltage-dependent Ca2+ and Na+ channels.

Therapeutic uses: absence seizures (petit mal epilepsy) (1st choice).

Adverse effects
– Sedation
– Vomiting
– Leucopenia

5. Benzodiazepines: Clonazepam and diazepam

Mechanism: allosteric modulation of GABA action to facilitate its effects.

347
Therapeutic uses
 Clonazepam: petit mal epilepsy.
 Diazepam: status epilepticus.

Adverse effects: see before.

6. Barbiturates: Phenobarbitone

Mechanism: enhancement of GABA-mediated inhibition of glutamate excitation.

Therapeutic uses: grand mal epilepsy (contraindicated in petit mal epilepsy).

Adverse effects: see before.

█ NEWER ANTIEPILEPTIC DRUGS

1. Felbamate

Mechanism
 Block glycine site on the N-methyl-D-aspartate (NMDA) receptors.
 Block voltage-dependent Ca2+& Na+ channels.

Therapeutic uses: wide variety of partial and generalized seizures.

Adverse effects
– Hepatotoxicity
– Microsomal enzyme induction.
– Bone marrow depression.

2. Lamotrigine

Mechanism
 Decreases glutamate and aspartate, which are excitatory neurotransmitters
 Blocks sodium channels and high voltage-dependent calcium channels leading
to ↓ excitability.

Therapeutic uses: wide variety of partial and generalized seizures and typical
absence seizures in children and adults.

3. Gabapentin

Mechanism: unknown but may interfere with voltage-dependent Ca2+ channels

348
Therapeutic uses: as adjuvant therapy in wide variety of partial and generalized
seizures.

Adverse effects: headache, nystagmus, dizziness & ataxia.

4. Tiagabine

Potent and specific inhibitor of GABA uptake into glial and other neurons. Thus, it
enhances the action of GABA by decreasing its removal from the synaptic space.

Precautions during antiepileptic therapy


 Proper choice of the antiepileptic drug according to type of epilepsy.
 Start with single drug and if fails, substitute it or add another drug.
 Dose of the drug is adjusted according to:
– Plasma concentration of the drug.
– Patient's response.
 Therapy is given for 2-3 years then withdrawn gradually.

Drugs used for treatment of Status epilepticus


Diazepam (drug of choice) 10 mg i.v. or 500 g/kg rectal (in children) OR:
 Fosphenytoin: 100 mg i.v.
 Phenobarbitone: 200 mg i.v.
 In severe cases:
a) Thiopental i.v.
b) Artificial respiration.

349
Part 6
6: Antiidepress
sant Dru
ugs

█ Basiic inform
mation

 Dep
pression iss a disord
der of mo
ood rather
thann disturban
nce of thought or coognition. It
is p
postulated that depreession is d
due to de-
ency of NA
ficie N and/orr 5-HT in the CNS
while mania results from functtional ex-
cesss. Psychicc depression is cha aracterized
by both emo otional andd biologiccal symp-
tomms.
 Reccent studiees suggestt that overractivity of
posst-synapticc 5-HT2A receptors in some
braiin areas iss involved in the path
hogenesis
of d
depression and psychosis.
 Unipolar dep
pression (major depressive
disoorder): mo
ore commo on, may be e reactive
(70%%) or endoogenous (25%),
( cha
aracterized
by low mood anda loss of interest o or pleasure
in normally enjjoyable acttivities.
 Bipolar dep
pression (manic-de
epressive
order): lesss common, charactterized by
diso
osc
cillating periods
p off depresssion and
mannia. There is strong hereditary
h o
origin.
 Thee therapeuttic effect occurs
o onlyy after 2-3
wee eks of dru
ug adminisstration an d is more
clossely assoc
ciated with
h adaptivee changes
in neuronal re
eceptors an
nd brain neeurotropic
facttors.

▌Class
sification of
o antidep
pressant d
drugs:

 Tric
cyclic anntidepress sants (TC
CA) e.g.
imip
pramine, amitriptyline
e.
 Sele
ective serotonin re euptake iinhibitors
SRI): e.g. flu
(SS uoxetine, sertraline.
s
 Aty
ypical hete ssants: e.g
erocyclic antidepres
a g. maprotiline, trazod
done.
 Monoamine oxidase
o in MAOI) e.g
nhibitors (M g. clorgyline
e, selegilinne.

350
1. Tricyclic antidepressants (TCA)
Imipramine, Desipramine, Clomipramine, Amitriptyline, Nortriptyline

Pharmacokinetics
 They are well absorbed after oral administration. They have large Vd.
 Most TCA have long t1/2 because they are metabolized into active metabolites
and undergo enterohepatic cycling.

Mechanism of action: (inhibition of the amine pump)


 TCA inhibit neuronal reuptake of both 5-HT & NA leading to their accumulation in
synaptic spaces and the brain tissue.
 It has been suggested that improvement of the emotional symptoms is related
to enhancement of 5-HT transmission while improvement of biological
symptoms is related to enhancement of NA transmission.
 Elevation of mood in depressed patients occurs after 2-3 weeks

Therapeutic uses
 Major depressive disorder.
 Nocturnal enuresis in children (imipramine).
 Chronic pain syndromes, neuropathic pain, and prophylaxis of migraine (unclear
mechanism).

Adverse effects
– Sedation is common at the start of therapy but tolerance develops later. It may be
due to antagonism with histamine H1 and/or muscarinic receptors.
– CNS troubles: memory dysfunction, agitation, seizures, and suicidal thoughts.
– Atropine-like action: very common - dry mouth, blurred vision, urine retention, etc.
– Orthostatic (postural) hypotension: due to peripheral α1 receptor blockade.
– Cardiac arrhythmias: tachycardia, widening of QRS, and ↑ QT interval.
– Hepatotoxicity: cholestatic hepatitis.
– Weight gain.
TCA overdose

Drug interactions  Metabolic acidosis


 Atropine-like effects
– Toxic synergism with MAOIs and SSRIs  Cardiac arrhythmia
(irritability and convulsions).
– TCA antagonize the antihypertensive effect of Management

clonidine and methyldopa.  IV NaHCO3 (1st step).


– TCA have additive anticholinergic effect with  IV lidocaine
other drugs having anticholinergic activity.  Dialysis is ineffective

351
2. Selective serotonin reuptake inhibitors (SSRIs)
Fluoxetine, Paroxetine, Sertraline, Citalopram, Escetalopram

 They are the most commonly prescribed antidepressants due to their limited
toxicity. They are also used for some other psychiatric disorders.
 Sertraline is the preferred antidepressant following myocardial infarction as there
is more evidence for its safe use in this situation than other antidepressants.
 When stopping an SSRI the dose should be gradually reduced over a 4 week
period, this reduces the risk of relapse.

Mechanism of action
 They selectively block 5HT reuptake leading to accumulation of 5-HT in brain tissue.
 Their effect appears after 2-3 weeks like other antidepressants.

Therapeutic uses
 Major depressive disorder.
 Obsessive-compulsive disorder (OCD).
 Anxiety disorders (generalized anxiety disorder, social phobia, panic disorder).

Adverse effects
– GIT irritation is the most common side effect. A proton pump inhibitor should be
prescribed if a patient is also taking a NSAID to avoid GIT bleeding.
– Sedation or insomnia at the start of therapy but tolerance develops later.
– Muscle cramps and twitches.
– Sexual dysfunction in up to 40% of patients – the main cause of noncompliance.
– Dangerous “serotonin reaction” may occur if given with MAOIs or TCA
(hyperthermia, muscle rigidity, cardiovascular collapse).

3. Atypical antidepressants

The pharmacological properties of atypical heterocyclic antidepressant agents are


similar to those of TCAs.

Mechanism of action
 Trazodone: blocks mainly 5HT2A receptors in addition to H1, and α1 receptors.
It is highly sedating and can cause postural hypotension.
 Mertazapine: blocks mainly 5HT2A receptors in addition to H1, and α2
receptors. It causes weight gain.
 Maprotiline: selective blocker of NA reuptake. It is highly sedating and can
cause seizures.

352
4. Monoamine oxidase inhibitors (MAOIs)
Clorgyline, Selegiline, Pargyline, Moclobemide

Mechanism of action
N.B. There are 2 isotypes of MAO
 They inhibit MAO enzyme leading to
enzyme:
accumulation of active monoamines (NA,
5-HT, dopamine) in neuronal tissue. MAO-A enzyme
 Most MAOIs are irreversible inhibitors. – Present in the cytoplasm of
Recovery of MAO takes several weeks. neurons (CNS) and peripheral
tissues (e.g. liver).
Moclobemide is a reversible inhibitor.
– It acts non-specifically on NA, 5-
HT, and dopamine.
Therapeutic uses – Clorgyline is a specific inhibitor.
 Major depression: they are not used as MAO-B enzyme
a first-line, but usually reserved as a last – Present mainly in the CNS and
line after other classes of drugs have acts more on dopamine.
failed. – Selegiline is a specific inhibitor.
 Selegiline (selective MAO-B inhibitor) is
used for treatment of Parkinsonism (see later).

Adverse effects
– CNS stimulation: irritability, insomnia, tremors, hyperthermia, convulsions
– Hepatotoxicity: occurs more with the old members.
– Orthostatic (postural) hypotension and sexual dysfunction.

Interactions

 Drug-drug interactions:
– Toxic synergism with tricyclic antidepressants and SSRIs.
– Potentiation of sympathomimetics (including cold remedies & nasal
decongestants).

 Drug-food interactions: Hypertensive crisis (cheese reaction):


– Tyramine is an indirect sympathomimetic present in some food and normally
metabolized by MAO-A in the liver.
– When the patient takes MAO-A inhibitor or non-selective MAOIs, severe
hypertension can occur after eating tyramine-rich food e.g. fermented cheese,
yogurt, beer, herrings.
– Treatment: by giving combined α + β blockers (prazosin + propranolol).

353
Moood Stabiilizing D
Drugs (trreatmen
nt of man
nia and
Part 7
7:
bipo
olar diso
order)

 Sod proate is
dium valp s the onlly specific
c
antiimanic ag gent and is the tre eatment of
o
chooice in the acute stagges.
 hium is the
Lith e drug of choice
c for long-term
m
trea
atment to prevent
p relapse.

Lithium
m carbon
nate

Mecha
anism of action
a
 It ↓ cAMP in neuronal
n ells and ↓ NA release
ce e
→ ↓ neuronal firing.
f
 nhibits many metabo
It in olic processses in the
e
nervve tissue.

Therap
peutic use
es
 Trea
atment off mania (valproate
( is the 1sts
cho
oice).
 Trea
atment of manic-deepressive disorder (bipolar
( de
epression).. It is given
n in the
mannic phase while
w TCA or SSRIs are given in
i the deprressive phaase.

Advers
se effects
 Lith
hium has a very na
arrow thera
apeutic index, monitoring of plasma le
evels is
essential.
 It haas long plaasma half-life being eexcreted entirely
e by the
t kidneyys.
 Toxxicity may be precip pitated by dehydratio on, renal failure,
f diuuretics (especially
thia
azide) or AC
CE inhibito
ors.

– Anoorexia, nau
usea, vomitting and d iarrhea.
– Nepphrogenic diabetes insipidus
i leading to polyurea and
a thirst.
– Hyppotension and
a cardia ac arrhythmmia.
– Thyyroid dysfu
unction
– Fine
e tremors s (coarse trremors are
e seen with
h toxic leve
els).
– Tera
atogenicityy.

354
Part 8
8: Antiipsychotic Drug
gs (Neurroleptics
s)

█ Basiic inform
mation

Centra
al dopamin nergic patthways
and drrugs affectting them:

 Thee term “p
psychosis”” denotes a variety
y of
men ntal disord
ders. Schizzophrenia is a partic
cular
kind
d of psychosis cha aracterized by abnormal
soccial behavioor and failure to reco
ognize what is
reall.
 Com
mmon “po
ositive” symptoms
s include false
f
beliefs (delu
usions), unclear or confu used
thin
nking, a
and auditory hallucinations.
“Neegative” symptoms
s include re
educed so ocial
enggagement, blunted em
motions, a
and inactivity.
 It is suggeested that centrall dopam
mine
ove
eractivity and 5H HT2A rece eptors play
p
imp
portant role
e in the patthogenesiss.

355
▌Classification of drugs

 Typical (classic; old) neuroleptics:


– Phenothiazines: chlorpromazine, promethazine.
– Butyrophenones: haloperidol, droperidol.
– Thioxanthenes: thiothixene, chlorprothixene.

 Atypical (newer) neuroleptics:


– Risperidone– Olanzapine – Clozapine

Mechanism and therapeutic action


 The typical antipsychotic drugs primarily block central D2 receptors in the
mesolimbic and mesocortical areas of the CNS leading to ↓ hallucinations and
delusions but can lead to serious extrapyramidal adverse effects.
 The atypical antipsychotic drugs combine between blockade of D2 (to a lesser
degree) and 5-HT2A receptors, so they have fewer extrapyramidal side effects.
 The typical antipsychotic drugs have greater effect on the control of positive
symptoms (hallucinations and thought disturbances), while atypical drugs have
greater effect on negative symptoms (social withdrawal).

Therapeutic uses
 Schizophrenia & mania (typical and atypical antipsychotics):
– Antipsychotic drugs produce an immediate quieting action. However, their
antipsychotic effects typically take longer time to occur (a week or more).

 Other therapeutic uses of typical antipsychotics:


– Chloropromazine is sometimes used to relieve intractable hiccough and as
anaesthetic adjuvant in certain operations to produce hypothermia.
– Severe nausea or vomiting associated with, e.g., radiation treatment and
cancer chemotherapy. Typical antipsychotic agents have strong antiemetic
activity due to D2-receptor blockade in the chemoreceptor trigger zone (CTZ) of
the medulla. The most commonly used are the phenothiazines chloropromazine
and promethazine

Adverse effects

 CNS:
 Extrapyramidal manifestations:
– These adverse effects are related to a D2-receptor blockade in the basal
ganglia. They occur most likely with old generation drugs.

356
– They in
nclude:
 Parkinsoniian-like synndrome
 Dystonia: sustained muscle co
on-
ttractions cause twiisting mov
ve-
ments or abnormal
a p
postures
 Dyskinesia
a: involunta
ary repetitive
ffacial, lip, and ton ngue mov ve-
ments occ curs with cchronic the
er-
apy. It mayy be irreve
ersible.

 Neurolepttic malignant syndro


ome:
– It is a serious compliication with w
mortality rate 20 0%, occurrs more with
w
old genneration drrugs.
– It consists of autonomic distu
urbance, ms
m rigidity, hypertherm
h mia, and sweating.
– It needs immedia ate stoppin
ng of the an
ntipsychottic therapyy.

 Auttonomic side effects


s:
– α
α-recepto
or blockadde leading to postura sion and seexual dysfunction
al hypotens
– de → atropiine-like actions
Muscariniic blockad

 End
docrine dis
sturbance
e (occurs m
more with
h atypical agents):
– W
Weight gain
– Hyperglycemia and precipitatio
p on of DM.
– G
Gynecoma astia, amen
norrhoea

 Oth
her advers
se effects:
– C
Cardiac arrrhythmia
– C
Cholestaticc jaundice
e
– A
Agranulocytosis (parrticularly asssociated with cloza
apine)

Typic
cal drugs Atypic
cal drugs
Mechanism Blockk mainly D2
2 receptors
s Block D
D2 (less) and
a
5HT2A (more)
Effect More effect on +ve
+ More eeffect on – ve
sympttoms symptooms
Extrapyyramidal side effects
s Comm
mon Less ccommon
Neurole
eptic malig
gnant synd
d Comm
mon Less ccommon
Endocrrinal side effects
e Less c
common Comm
mon
Agranu
ulocytosis Less c
common Comm
mon

357
Part 9
9: Anttiparkins
sonian D
Drugs

Parkinssonism is a movem ment


disorde er charactterized by ms
rigidity, tremors, and posttural
instabillity due too loss of do-
pamine ergic neu urons in sub-
s
stantiaa nigra an nd locus ce-
ruleus resulting in imbala ance
of do opaminergic (inhibittory)
and c cholinergic
c (excitattory)
influencces on thee extrapyraami-
dal sysstem. Rece ently, a he
eroin
substitute (MPTP P) was fo ound
to cau use irreverrsible dammage
of the nnegrostriattal dopaminergic neu
urons and a parkinso
onism-like sstate.

▌Class
sification of
o antipark
kinsonian drugs:

 Dop
paminergiic drugs:
– Dopamine e precurso ors: Levod
dopa (L-do opa).
– COMT inh hibitors: To
olcapone - Entacapo one
– Selective MAO-B in nhibitors: S
Selegiline
– Dopamine e agonists s: Bromocrriptine
– Release of
o dopamin ne and inh
hibition of its
reuptake: Amantadin
A ne

 Anttichloinerg
gic drugs:
– Synthetic atropine substitutes
s s: Benztropine –
Orphenadrrine – Trihex
xyphenidyl

1. Lev
vodopa (L
L-dopa)

Pharm
macokinetics
 Abssorbed rap
pidly from the
t small in
ntestine.
 It has short t1/2 (1-2 h)). This sho ort t1/2 ma
ay producee "on-off p
phenomen non" i.e.
rapiid fluctuation of the clinical sta
ate in the form of su
udden tremmors & immmobility
afte
er short period of recovery.
 Amino acids (e.g. leuc cine & isol eucine) co
ompete with L-dopaa absorptio
on from
the gut so it should
s be taken
t on an empty sttomach.

358
Mecha
anism of action
a
 Dop
pamine can’t cross BBBB but LL-dopa can n. It is
st
con
nsidered the
e 1 line tre
eatment of parkinsonism.
 Morre than 95% 9 of th
he adminiistered do ose is
rapiidly decarboxylated d into do opamine in the
y a small f raction escapes
periipheral tisssues. Only
and
d crosses BBB.
B
 Peripheral deecarboxylattion can b
be minimizzed by
adm
ministration
n of a decaarboxylase
e inhibitor which
can
n't cross BBBB e.g. ca
arbidopa oor bensera azide.
 Suc
ch a combination heelps to red
duce the do
ose of
L-dopa and hence the adverse
a efffects

Advers
se effects
 T: nausea & vomiting (domperid
GIT done is the e drug of choice
c to ttreat this vomiting
v
bec
cause it does not cros
ss the BBB
B. It acts on
o the baree area of thhe barrier).

 CNS
S:
– Mood cha anges, halluucinations and nightm mares.
– Dyskinesia a: involunta
ary movemments of th he head, lip
ps, and tonngue.
– On-off ph henomenon n: rapid fl uctuation of the clinical stat e in the form
f of
sudden tre emors and immobilityy after a shhort period of recoverry due to th
he short
t1/2 of the drug.
d

 Auttonomic: postural
p hy
ypotension
n, arrhythm
mias, mydriasis and aacute rise of
o IOP.

 ownish bod
Bro dy secretio
ons & red u
urine due to
t the meta
abolite hom
movanilic acid.
a

Interac
ctions

 od: amino acids


Foo a (e.g. leucine & issoleucine) compete with
w L-dopaa absorptio
on.
 Doppamine bllockers (a
antipsycho tic drugs and metoc
clopramidee) antagon
nize the
effe
ects of L-do
opa.
 Pyrridoxine (vvitamin B6) enhancess periphera
al decarbox
xylation intto dopamine.
 COMT inhibittors and se
elective MA bitors: mak
AO-B inhib ke synergissm with L-d
dopa.

2. COM
MT inhibiitors (L-D
Dopa adju
uvants): Tolcapone
T - Entacap
pone

Mecha
anism of action
a
 The
ey inhibit COMT
C enzy
yme selecttively & rev
versibly, th
he enzymee that conv
verts L-
dop
pa to 3-O-mmethyldop
pa (3OMD) in the gut and liver.

359
 By inhibiting COMT enzyme, tolcapone increases the efficacy of L-dopa and
stabilizes dopamine levels in the striatum and improves motor function.

Adverse effects
– Same side effects of L-dopa (nausea, vomiting, hallucination, mood changes).
– Tolcapone causes fulminating hepatic necrosis.

3. Selective MAO-B inhibitors: Selegiline

Mechanism of action
 It is a selective MAO-B inhibitor decreasing breakdown of dopamine in the brain.
 Unlike MAO-A inhibitors, it does not precipitate "cheese reaction" since
tyramine is metabolized in the liver by MAO-A.

Adverse effects
– Nausea, vomiting & GIT upset.
– Hallucinations & mood changes.
– Insomnia.

4. Dopamine agonists: Bromocriptine

 It is dopamine (D2) agonist derived from ergot alkaloids.


 It differs from L-dopa in: Faster onset, longer duration, No on-off effect.

Other uses
 To treat hyperprolactinemia and to suppress lactation (dopamine agonists ↓
prolactin secretion from pituitary gland).
 Acromegaly (dopamine agonists ↓ GH secretion from pituitary gland).

Adverse effects
– Nausea, vomiting & GIT upset.
– Hallucinations & mood changes in large doses.
– Postural hypotension.

5. Dopamine release stimulation: Amantadine

 It is an antiviral drug for influenza A2 virus.


 The antiparkinsonian mechanism is unclear but may be due to ↑ dopamine
release, ↓ dopamine reuptake, or direct action on dopamine receptors.

360
Advers
se effects
– Nauusea, vomiiting & GITT upset.
– Halllucinations & mood chhanges in la
arge doses
s.
– Posstural hypootension.
– Skin
n pigmenta ation.

6. Antticholinerrgic drugs s:
Benztroopine – Orrphenadrin
ne – Trihex
xyphenidyl

 Theese drugs have gre ectivity fo


eater sele or
bloc
cking central than pe muscarinic receptors..
eripheral m
 Theey producee moderate
e improvem
ment of tremors and
d sialorrhe
ea but little effect
on m
muscle rigidity.
 Theey are used
d also to co
ontrol acutte drug-ind
duced extrrapyramidaal side effe
ects.
 Theese drugs are con
ntraindicatted in pa
atients with prosta
atic hyperrtrophy,
obsstructive GI disease (e.g.,
( paralyytic ileus) and narrow
w-angle gl aucoma.
 In p
patients suffering
s frrom Parkin
nsonism, there is some
s deg ree of de
ementia
associated wiith atrophy
y of cortica
al neurons.. Anticholin
nergic drug
gs may aggravate
dem
mentia and
d memory loss.

361
Part 1
10: Dru
ugs Used in Alz
zheimer''s Disea
ase

Alzheim
mer is a ne
eurodegene erative dise
ease
charactterized by progres ssive losss of
memorry and de ementia du ue to losss of
cholineergic neuurons and depositio on of
abnormmal prote ein “amylo oid plaqu ues”.
Overstiimulation of
o glutamate recep ptors
may be e responsible for this
s neurode gen-
erative process.
The aim of therrapy is to either impprov-
ing cholinergic transmission within the
CNS or preventinng the exc
citatory acttions
of NMDDA glutamaate receptors in sele
ected
brain areas.

1. Cho
olinestera
ase inhib
bitors: Don
nepezil - Rivastigm
R ine

– The
ey are morre selective
e for ChE e
enzyme in the CNS
– The
ey provide a modest reduction in the rate
e of loss off cognitive functions..
– de effects: nausea, vomiting,
Sid v a
anorexia, and muscle e cramps.

2. NMDA recep
ptor antag
gonist: M
Memantine
e

– It iss a derivative of amantadine


– It prevents ratee of memory loss in b
both vasculaar-associatted and Alzzheimer dem
mentia.
– Sidde effects are rare: confusion,
c agitation, restlessne
ess.

3. Rec
cent trials
s:

 β & γ-secreta
ase inhibittors:
Thee enzymes β & γ-seccretase aree responsiible for forrmation of amyloid proteins
p
Aβ440 and Aββ42 that arre found in
n the brain of Alzhe eimer patieents. Inhib
bition of
thesse enzyme
es reduces depositio n of theses
s amyloid deposits.

 Ibup
profen and indomethacin
The
ese NSAID Ds reduce formation of Aβ42 by inhibition of γ-ssecretase enzyme
e
which is unreelated to their
t COX inhibition. Aspirin and
a corticcosteroids do not
duce this effect.
prod e

 Cop
pper and zinc
z chela
ating agen
nts:
Rem
moval of th
hese metals promote
es dissolutiion of amy
yloid plaquees in brain
n tissue.

362
Part 11: General Anesthetics

General anesthesia is characterized by a loss of consciousness, analgesia,


amnesia, skeletal ms relaxation, and inhibition of autonomic and sensory reflexes.

Balanced anesthesia
 Balanced anesthesia refers to a combination of drugs used to take advantage of
individual drug properties and minimizing their adverse actions.
 In addition to anesthetic drugs and neuromuscular blocking drugs, other drugs
are administered preoperatively, intraoperatively, and postoperatively to ensure
smooth induction, sedation, and smooth recovery (e.g., benzodiazepines, opioids).

Stages and planes of anesthesia


 The stages and planes of anesthesia identify the progression of physical signs
that indicate the depth of anesthesia. Newer, more potent agents progress
through these stages rapidly, and therefore, the stages are often obscured.
 Mechanical ventilation and the use of adjunct drugs also obscure the signs
indicating the depth of anesthesia.
– Stage I: Analgesia and amnesia.
– Stage II: Excitation.
– Stage III: Surgical anesthesia: loss of consciousness.
– Four planes have been described relating to increased depth of anesthesia.
– Plane IV includes maximal pupil dilation, apnea, and circulatory depression.
– The loss of the eyelash reflex and a pattern of respiration that is regular in
rate and depth are the most reliable signs of stage III anesthesia.
– Stage IV: Medullary depression: respiratory and cardiovascular failure.

Mechanism of anesthetic action


Inhalation and IV anesthetic agent interaction with discrete protein binding sites in
nerve endings to activate ligand-gated ion channels:
 GABA-A-receptor chloride channels: anesthetic agents directly and indirectly
facilitate a GABA-mediated increase in chloride conductance to hyperpolarize
and inhibit neuronal membrane activity.
 Ligand-gated potassium channels: anesthetic agents increase potassium
conductance to hyperpolarize and inhibit neuronal membrane activity.
 NMDA receptors: certain anesthetics (e.g., nitrous oxide, ketamine) inhibit
excitatory glutamate gated ion channels.

363
▌Class
sification of
o generall anesthettics

█ INTR
RAVENOU
US ANAES
STHETICS
S

1. Ultrrashort acting barrbiturates


s: thiopen
ntal

Advanttages
 Pro
oduces rap pid and pleeasant indu uction with
h rapid recoovery.
 Doees not incrrease intracranial preessure.
 Useed for indu
uction befoore stronge er anesthettic and for short proccedures.

Disadv
vantages
 Poo or analgesia and ske eletal musccle relaxatio
on.
 Induce cough h, laryngo ospasm, brronchospa asm and apnea
a avooided by atropine
a
andd ready arttificial resp
piration.
 Exttravasationn induces necrosis.
n
 Con ntraindicatted absolu utely in ca
ases of prophyria
p (defect
( of ALA syntthetase)
barrbiturates increase synthesis
s of porphy yrins which
h precipitaates acute
e attack
inducing paraalysis and death.
d

2. Eto
omidate

Advanttages: It iss very similar to thiop


pental but:
 Larg ger marginn between the anaessthetic dos se and the dose prod ducing respiratory
and d cardiovasscular depression tha an with thiopental.
 It iss more rappidly metab bolized tha
an thiopenttal and, therefore, leess likely to
o cause
a prrolonged hangover.
h
 It iss used onlyy as an ind duction age
ent, and is preferablee to thiopeental in pattients at
riskk of circulattory failure
e.

364
Disadvantages
 More likely to cause involuntary movements during induction.
 And to cause postoperative nausea and vomiting.
 Suppresses the adrenal cortex with prolonged use which has been associated
with an increase in mortality in severely ill patients.

3. Propofol

Advantages
 Rapid and pleasant induction with rapid recovery and clarity of mental status.
 Does not increase intracranial pressure.
 Used for induction or maintenance.

Disadvantages
 Respiratory center depressant less than thiopental.
 Markedly decreases the blood pressure.

4. Midazolam (see under Benzodiazepines)

 They are used in conjunction with anaesthetics to sedate the patient.


 The most commonly employed is midazolam.
 Diazepam and lorazepam are alternatives. All three facilitate amnesia while
causing sedation.

Advantages
 Used for induction
 3-4 times as potent as diazepam.
 Does not cause local irritation after injection as diazepam.
 Little cardiovascular or respiratory depression.

Disadvantages
 Light anaesthesia.
 Can cause respiratory depression.
 Anterograde amnesia which lasts for at least 2 hours.

5. Ketamine (Dissociative Anaesthesia)

Advantages
 Profound analgesia and amnesia
 The only intravenous anesthetic that routinely produces cardiovascular
stimulation through central sympathetic stimulation (increases Bl.P. H.R and
C.O.). So, it is beneficial in cases of shock.
 No respiratory depression and has a potent bronchodilator effects.

365
Disadvantages
 Increases intracraneal pressure (due to increased cerebral blood flow).
 Unpleasant dreams.
 Recovery often is accompanied by delirium and psychomotor activity).
 Diplopia and nystagmus may occur due to increased muscle tone.
 Contraindicated in cases of hypertension and stroke.

█ INHALATION ANAESTHETICS

An important characteristic of an inhalation anaesthetic is the speed at which the


arterial blood concentration, which governs the pharmacological effect, follows
changes in the concentration of the drug in the inspired air.

Measure of potency
 Through the minimum alveolar concentration (MAC). It is the anaesthetic (MAC)
alveolar concentration at 1 atmosphere that will produce loss of movement in
50% of subjects exposed to a noxious stimulus.
 Methoxyflurane is highly potent while nitrous oxide is of low potency.

Rapidity of induction
 Gas of little solubility in blood as nitrous oxide has a high blood gas tension, a
small Vd, with a consequent rapid induction and rapid recovery.
 Gas of high solubility in blood e.g. methoxyflurane has a low blood gas tension,
a big Vd, with a slow induction and a slow recovery.

Pharmacological effects
 Inhalation anaesthetics produce descending depression of all brain functions in
the following order: Cortex >Subcortex> Mid-brain > Spinal cord > Medullary
centers.
 The highly developed functions like memory are the first to be lost, but the vital
functions as respiration and circulation are the last.
 With modern balanced anaesthesia no more notable staging. Loss of
consiousness, analgesia and muscle relaxation is produced with a combination
of drugs rather than with a single anaesthetic agent.
 Unconsciousness rapidly produced with an i.v. induction agent (e.g. propofol), to
maintain unconsciousness and produce analgesia with one or more inhalation
agent which might be supplemented with an i.v. analgesic agent, and to produce
muscle paralysis with a neuromuscular blocking drug.
 The following table shows points of difference in between the currently used
inhalation anesthetics:

366
367
Preanesthetic medications (anesthetic adjuvant):

They are drugs given to reduce anxiety, providing analgesia and amnesia, and
prevent salivation, bradycardia, and other side effects of anesthesia.

 Anxiolytics (e.g. diazepam): given at night preoperatively to provide sedation &


amnesia.

 Opioid analgesics (e.g. morphine, fentanyl, or alfentanil):


– They are given 1½ hrs before anesthesia to inforce low potency anesthetics as
nitrous oxide.Alfentanyl may offer faster recovery than fentanyl.

 Anticholinergic drugs (e.g. atropine or scopolamine):


– They are given 1½ hrs before anesthesia to reduce respiratory secretions and
prevent occurrence of aspiration pneumonia.
– They block the reflex vagal stimulation induced by some inhalation anesthetics.

 Muscle relaxants & others drugs: e.g. vasopressors to prevent hypotension


and hypothermics (chlorpromazine) to lower the normal body temperature in
cases of cardiac surgery.

368
Part 12: Local Anesthetics

Classification

There are two classes of local anesthetics: amides and esters. The primary
differences between the two classes are in their relative metabolism (amides have
primarily a hepatic metabolism, whereas esters are metabolized by plasma
cholinesterases) and their potential for allergic reactions (esters more than amides).

Esters Amides
Members Cocaine, procaine, Lidocaine, prilocaine,
tetracaine, benzocaine mepivacaine,
bupivacaine
Metabolism Plasma cholineestrase Liver
Hypersensitivity reactions May occur Rare

Mechanism of action
 Local anesthetics block voltage dependent Na+ channels within the nerve fibers
→↓ nerve conduction.
 In general, small nerve fibers (that carry pain sensation) are more sensitive to
local anesthetics than large fibers (motor and other sensations).
 Myelinated fibers are blocked before non-myelinated fibers of the same
diameter.

Pharmacological properties

Chemistry Anesthetic Duration Onset


potency
Procaine Ester Low Short Moderate
Mepivacaine Amide Moderate Moderate Fast
Prilocaine Amide Moderate Moderate Fast
Lidocaine Amide Moderate Moderate Fast
Tetracaine Ester High Long Moderate

Adjuvants of local anesthetics

 Vasoconstricotors e.g. adrenaline:


– They are added to produce local VC; this will reduce wash of the anesthetic
by the local blood flow and thus prolongs the duration of the anesthetic.

369
– Vasoconstrictors should not be added for ring block of hands, feet, fingers,
toes, and ear pinna to avoid tissue damage.

 Sodium bicarbonate:
– All local anesthetics are weak bases. Addition of bicarbonate to the
anesthetic solution maintains the anesthetic in the non-ionized state and this
increases lipid solubility and enhances penetration of the anesthetic into the
nerve sheath.

Techniques and clinical uses

 Surface (topical) anesthesia: is used for anesthetization of the skin, mucous


membranes, or cornea.

 Treatment of itching caused by insect bites or irritant condition.


 To relieve pain caused by oral, laryngeal, or rectal disorders e.g. piles.
 Corneal anesthesia in ophthalmic surgery.

 Infiltration anesthesia: injecting an anesthetic directly into the subcutaneous


tissue for minor surgical procedures e.g. dental procedures. Epinephrine may be
added to prolong the duration of action.

 Nerve block: injecting the local anesthetic close to the appropriate nerve trunk
proximal to the intended area of anesthesia e.g. radial nerve block or retro-orbital
block for ocular surgery.

 Spinal anesthesia: is used for surgeries of the lower limb or pelvic structures. A
local anesthetic is injected into the subarachnoid space below the terminal end
of the spinal cord (usually between 3rd and 4th lumber vertebra).

 Epidural anesthesia: the anesthetic is infused into the space between the dura
mater and the connective tissue lining the vertebral canal as alternative to
subarachnoid anesthesia.

Adverse effects
– Surface anesthesia: allergic dermatitis.

– Infiltration anesthesia: faulty intravascular injection can lead to:


 CNS excitation: irritability and convulsions
 CVS depression: bradycardia, hypotension, myocardial depression or
cardiac arrest.

– Vasovagal syncope: vasovagal (also called neurocardiogenic) syncope is


usually associated with bradycardia (rather than tachycardia) and pallor (rather

370
than flushing). These differences can be helpful in distinguishing it from
anaphylaxis.

– Spinal anesthesia:
 Spinal shock due to sympathetic outflow paralysis.
 Headache due to CSF leakage.
 Respiratory paralysis.
 Septic meningitis.

Recommendations
 The choice of local anesthetic for infiltration depends on several factors,
including duration of the procedure, need for hemostasis, patient sensitivity to
catecholamines, and patient allergy to local anesthetics.

 Prior to infiltration, the clinician should determine if the patient has any history of
allergy. Patients with a true allergic reaction to a local anesthetic need evaluation
by an allergy specialist.

 Local anesthetic infiltration is contraindicated in the following conditions:


– If the area needed to be anesthetized is so large because the amount of
anesthetic needed exceeds the maximal safe dose.
– If the patient has history of allergy to local anesthesia.

Treatment of spinal shock


 Place the patient in the supine position with the legs elevated.
 Airway support: oxygen and artificial respiration.
 If there are convulsions, give i.v. diazepam.
 If there is hypotension, give i.v. fluids plus vasoconstrictors.
 Cardiopulmonary resuscitation (CPR) is necessary when cardiac function is
interrupted.

371
 

372
Review Questions

Long questions:
1. Classify opioid analgesics and discuss mechanism, uses, &contraindications of
morphine.
2. Anxiety is a common medical disorder. Classify antianxiety drugs and discuss
in details the differences between barbiturates and benzodiazepines.
3. Depression is an important medical disease. Classify antidepressant drugs;
mention their mechanism, indications and side effects.
4. Classify antiepileptic drugs; mention their mechanism, indications and side
effects.
5. Dopamine is found in many areas in the CNS. Discuss the drugs that act by
inhibiting these central dopamine sites.
6. Dopamine is found in many areas in the CNS. Discuss the drugs that act by
activating these central dopamine sites.

Short questions:
1. Give an account on:
a) Mechanism and treatment of paracetamol toxicity
b) Drug treatment of Alzheimer's disease.

2. Discuss the pharmacodynamic principles underlying the use of:


a) Morphine in acute pulmonary edema.
b) Acetylcysteine in paracetamol toxicity
c) Naloxone in acute opioid toxicity.
d) Selegiline in parkinsonism.

3. Discuss on pharmacological basis of each of the following:


a) Nalorphine is contraindicated to relive pain in addict patient.
b) Reserpine is contraindicated with MAOIs.
c) Anticholinergic drugs are not preferred to treat parkinsonism in old patients.

4. Mention drug interactions of each of the following drugs:


(1) L-Dopa (2) MAOIs

5. Mention the main differences between:


a) Valproic acid and lamotrigine
b) L-Dopa and bromocriptine.
c) Chloropromazine and olanzapine

373
Of each of the following questions, E. Less gastric irritation than other
select ONE BEST answer: NSAIDs

1. Morphine, all are true EXCEPT: 6. Aspirin reduces the synthesis of


A. Acts as an agonist at opioid receptors the following eicosanoids EXCEPT:
(especially μ) in the brain and spinal A. TXA2
cord B. PGE2
B. Causes pupillary constriction by C. PGF2α
stimulation of the Edinger–Westphal D. LTB4
nucleus in the mid-brain E. PGI2
C. Acts as an antihistamine
D. Is subject to presystemic metabolism 7. The following statements about
E. Stimulates the chemoreceptor trigger NSAIDs are correct EXCEPT:
zone A. Diclofenac may cause permanent
platelet dysfunction
2. Which of the following is not true B. Indomethacin may cause neurological
for meperidine? side effects (neurotoxicity).
A. It has less analgesic potency than C. Sulindac is a prodrug
morphine D. They may impair renal function
B. It has less spasmogenic effect than E. Aspirin may displace coumarin
morphine anticoagulants from plasma proteins
C. It can cause histamine release and
bronchoconstriction 8. The following statements about
D. It has greater bioavailability than aspirin are correct EXCEPT:
morphine A. May cause GIT hemorrhage after a
E. It does not cause physical single dose
dependence B. Enteric-coated tablets cause less
gastric bleeding
3. Morphine causes all of the C. May cause metabolic alkalosis in high
following EXCEPT: doses
A. Diarrhoea D. May cause Rye’s syndrome in children
B. Increased intrabiliary pressure E. Its toxicity may require treatment with
C. Histamine release hemodialysis
D. Reduced sensitivity of the respiratory
centre to carbon dioxide 9. Which one of the following statem-
E. Hypotension ents concerning Cox-2 inhibitors is
correct:
4. The following are particularly A. They show greater analgesic activity
sensitive to the pharmacological than traditional NSAIDs
actions of morphine EXCEPT: B. They show anti-inflammatory activity
A. Young children greater than traditional NSAIDs
B. The elderly C. They harm the stomach as do non-
C. Patients with hepatic failure selective cox inhibitors
D. Patients with renal failure D. They increase platelet aggregation
E. Patients with hyperthyroidism E. They are cardio protective

5. The pharmacologic effects of 10. Non-narcotic analgesics are all of


acetylsalicylic acid include: the following drugs EXCEPT:
A. Reduction of high body temperature A. Paracetamol
B. Promotion of platelet aggregation B. Acetylsalicylic acid
C. Reduction of pain by stimulation of C. Codiene
PGs synthesis D. Ketorolac
D. Efficacy equals to that of probenecid E. Dipyrone
as uricosuric agent

374
11. Which one of the following non- 17. The following drug can be given to
narcotic agents inhibits mainly reverse benzodiazepine overdose:
cyclooxygenase (COX) in CNS? A. Cocaine
A. Paracetamol B. Flumazenil
B. Ketorolac C. Buspirone
C. Acetylsalicylic acid D. Picrotoxin
D. Ibuprofen E. Diazepam
E. Celecoxib
18. Indicate the anxiolitic agent, which
12. For which of the following relieves anxiety without causing
conditions could aspirin be used marked sedative effects:
prophylactically? A. Diazepam
A. Noncardiogenic pulmonary edema B. Buspirone
B. Peptic ulcers C. Lorazepam
C. Thromboembolism D. Clorazepate
D. Metabolic acidosis E. Zolpidem
E. Periodontitis
19. Local anesthetics produce:
13. Nefopam: A. Analgesia, amnesia, loss of
A. Is associated with gastrointestinal consciousness
haemorrhage B. Blocking pain sensation without loss
B. Causes miosis of consciousness
C. Causes more respiratory depression C. Alleviation of anxiety and pain with an
than morphine altered level of consciousness
D. Potentiates the dysrhythmogenic D. A stupor or somnolent state
effect of halothane anaesthesia E. Physical dependence
E. Is contraindicated in epilepsy
20. Local anesthetics act by:
14. Indicate the benzodiazepine, which A. Blocking voltage-gated sodium
has the shortest elimination half-life: channels
A. Nitrazepam B. Blocking voltage-gated calcium
B. Alprazolam channels
C. Triazolam C. Blocking voltage-gated potassium
D. Diazepam channels
E. Clorazepate D. Blocking chloride conductance
E. Blocking NMDA receptors
15. Which of the following
benzodiazepines is preferred for elderly 21. Which of the following local
patients? anesthetics has short duration?
A. Clorazepate A. Procaine
B. Clonazepam B. Mepivacaine
C. Triazolam C. Prilocaine
D. Prazepam D. Lidocaine
E. Diazepam E. Tetracaine

16. Which of the following 22. Which drug does not activate
benzodiazepines is preferred for opioid receptors, has been proposed as
patients with liver disease? a maintenance drug in treatment
A. Clorazepate programs for opioid addicts, and with a
B. Nitrazepam single oral dose, will block the effects
C. Lorazepam of injected heroin for up to 48 h?
D. Prazepam (A) Fentanyl
E. Diazepam (B) Nalbuphine
(C) Naloxone

375
(D) Naltrexone 27. A 9-year-old child is having
(E) Propoxyphene learning difficulties at school. He has
brief lapses of awareness with eyelid
23. Which drug is a full agonist at fluttering that occur every 5–10 min.
opioid receptors with analgesic activity Which drug would be effective in this
equivalent to morphine, a longer child without the disadvantages of
duration of action, and fewer excessive sedation?
withdrawal signs on abrupt (A) Clonazepam
discontinuance than morphine? (B) Diazepam
(A) Fentanyl (C) Ethosuximide
(B) Hydromorphone (D) Gabapentin
(C) Methadone (E) Phenobarbital
(D) Nalbuphine
(E) Oxycodone 28. Which antiepileptic drug is most
likely to elevate the plasma
24. Which drug used in the concentration of other drugs
maintenance treatment of patients with administered concomitantly?
tonic-clonic or partial seizure states (A) Carbamazepine
increases the hepatic metabolism of (B) Clonazepam
many drugs including both phenytoin (C) Phenobarbital
and warfarin? (D) Phenytoin
(A) Buspirone (E) Valproic acid
(B) Clonazepam
(C) Eszopiclone 29. With chronic use in seizure states,
(D) Phenobarbital the adverse effects of this drug include
(E) Triazolam coarsening of facial features, hirsutism,
and gingival hyperplasia.
25. A patient with liver dysfunction is (A) Carbamazepine
scheduled for a surgical procedure. (B) Ethosuximide
Lorazepam or oxazepam can be used (C) Zonisamide
for preanesthetic sedation in this (D) Tiagabine
patient without special concern (E) Phenytoin
regarding excessive CNS depression
because these drugs are 30. The mechanism of antiseizure
(A) Actively secreted in the renal proximal activity of carbamazepine is
tubule (A) Block of sodium ion channels
(B) Conjugated extrahepatically (B) Block of calcium ion channels
(C) Eliminated via the lungs (C) Facilitation of GABA actions on
(D) Reversible by administration of chloride ion channels
naloxone (D) Glutamate receptor antagonism
(E) Selective anxiolytics like buspirone (E) Inhibition of GABA transaminase

26. This drug used in the management 31. Which statement about phenytoin
of insomnia facilitates the inhibitory is accurate?
actions of GABA. Its actions are (A) Displaces sulfonamides from plasma
antagonized by flumazenil. proteins
(A) Buspirone (B) Drug of choice in myoclonic seizures
(B) Temazepam (C) Half-life is increased if used with
(C) Eszopiclone phenobarbital
(D) Ramelteon (D) Isoniazid (INH) decreases steady-state
(E) Phenobarbital blood levels of phenytoin
(E) Toxic effects may occur with only
small increments in the dose

376
32. Which of the following drugs is the 37. Which of the following drugs is
most effective in the emergency most likely to be of value in obsessive-
management of malignant compulsive disorders?
hyperthermia? (A) Amitriptyline
(A) Atropine (B) Bupropion
(B) Dantrolene (C) Sertraline
(C) Haloperidol (D) Trazodone
(D) Succinylcholine (E) Venlafaxine
(E) Vecuronium
38. A patient under treatment for a ma-
33. Which drug is most likely to cause jor depression is brought to the
hyperkalemia leading to cardiac arrest emergency department after ingesting
in patients with spinal cord injuries? 30 tablets of imipramine. Which of the
(A) Baclofen following would be LEAST useful?
(B) Dantrolene (A) Administer bicarbonate to correct
(C) Pancuronium acidosis.
(D) Succinylcholine (B) Administer lidocaine to control cardiac
(E) Vecuronium arrhythmias.
(C) Initiate hemodialysis to hasten drug
34. Tolcapone may be of value in elimination.
patients being treated with levodopa- (D) Maintain heart rhythm by electrical
carbidopa because it: pacing
(A) Activates COMT (E) Use intravenous diazepam to control
(B) Decreases the formation of 3-O- seizures
methyldopa
(C) Inhibits monoamine oxidase type A 39. A 36-year-old woman presents with
(D) Inhibits neuronal reuptake of dopamine symptoms of major depression. Drug
(E) Releases dopamine from nerve treatment is to be initiated with
endings sertraline. In your information to the
patient, you would tell her that
35. Concerning the drugs used in (A) Sertraline may take 2 wk or more to
parkinsonism, which statement is become effective
accurate? (B) It is preferable that she take the drug
(A) Dopamine receptor agonists should in the morning
never be used in Parkinson’s disease (C) Muscle cramps and twitches can
before a trial of levodopa occur
(B) Levodopa causes mydriasis and may (D) She should notify you if she
precipitate an acute attack of glaucoma anticipates using other prescription drugs
(C) Selegiline is a selective inhibitor of COMT (E) All of the above
(D) The primary benefit of antimuscarinic
drugs in parkinsonism is their ability to Answers
relieve bradykinesia
(E) Therapeutic effects of amantadine 1C 10 C 19 B 28 E 37 C
continue for several years 2E 11 A 20 A 29 E 38 C
3A 12 C 21 A 30 A 39 E
36. Which drug is an antagonist at 5- 4E 13 E 22 D 31 E
HT2 receptors and can be used for the 5A 14 C 23 C 32 B
management of insomnia? 6D 15 C 24 D 33 D
(A) Estazolam 7A 16 C 25 B 34 B
(B) Flurazepam 8C 17 B 26 B 35 B
(C) Trazodone 9D 18 B 27 C 36 C
(D) Triazolam
(E) Zolpidem

377
 

378
   
 
Part 1
1: Bas
sic Princ
ciples o
of Antim
microbiall Drugs

Chemo otherapy iss the killin


ng of a li ving organism whe
ether being
g bacteria
a, fungi,
protozo
oa, or viruss.
Bacteriia are prokaryotic cells. Some e bacteria are patho ogenic to humans and a can
cause sserious infections; th
he principa
al treatmen
nt of infections is withh antibiotics.

█ CLA
ASSIFICAT
TION OF ANTIMICR
A ROBIAL DR
RUGS

A. Acc
cording to source:

 N
Natural co
ompounds s: e.g. peniicillin, chloramphenic col.
 S
Synthetic compound
c ds: e.g. su
ulfonamide es, quinolon
nes.
 S
Semisynth pounds: e .g. ampicilllin.
hetic comp

B. Acc
cording to the effectt on micro
oorganism
ms:

 B dal agents:: that kills tthe microo


Bactericid organism e.g.
e penicillllin.
 B
Bacteriosttatic agents: arrest g growth of the
t microo organism ee.g. sulfona
amides.

cording to mechanis
C. Acc sm of acti on:

 A
Agents actt by inhibittion of celll wall synthesis: e.g. penicillin.
 Inhibition of
o cell mem
mbrane fu
unction e.g
g. amphote
ericin B and
d azoles.
 A
Agents actt by inhibittion of nuc
cleic acid synthesis:
s e.g. quino
olones.
 A
Agents actt by inhibittion of pro tein synthesis:
‒ By actin
ng on ribos
somal 30 S subunit e.g.
e aminog
glycosidess.
‒ By actin
ng on ribos
somal 50 S subunit e.g.
e macrollides.
 A
Agents actt by inhibittion of bac
cterial meta
abolism: e.g.
e sulfonaamides.

 
  379
D. Acc
cording to antimicro
obial spec
ctrum: es pluralis ((spp.), Latin
Specie n
abbreviation for mmultiple speecies. 
 N
Narrow sp
pectrum drrugs:
‒ Drugs affect
a main
nly Gram +
+ve spp. e.gg. benzyl penicillin.
p
‒ Drugs affect
a main
nly Gram –vve spp. e.g
g. aminoglyycosides.
 E
Extended spectrum drugs: ag gents that affect
a Gram
m +ve & G Gram –ve spp..
 B
Broad spe
ectrum dru ugs: agentts act on wide
w range
e of Gram m +ve & Grram –ve
sspp. and otthers (prottozoa) e.g. tetracyclin
nes.

█ COM
MBINATIO
ON OF ANT
TIBIOTICS
S

Indicattions:

 To o
obtain broader spec
ctrum e.g. amoxicillin nic acid → co-amoxic
n + clavulan clav.
 To oobtain syn
nergism e.g. sulfonam mides + trimethoprim m → co-trim
moxazole.
 In m
mixed bactterial infections e.g. d
diabetic fo
oot or peritonitis.
 In s
serious bacterial infe
ections e.g . bacterial meningitis
s or septiceemia.
 To oovercome bacterial resistanc e e.g. TB and pseud domonas i nfection.
 To rreduce toxxicity of one drug byy using smmaller doses of two drrugs.

Results:

 Bac dal → syne rgism: e.g


ctericidal + bactericid g. penicillin
n with aminnoglycosides.
 Bac
cteriostatic ostatic → a
c + bacterio addition: e.g.
e tetracy
yclines withh sulfonam
mides.
 Bac static → anttagonism or synerg
ctericidal + bacterios gism:

 Antagonis sm: when bactericid dal drug acts by in


nhibiting ceell wall sy
ynthesis
(i.e. kills rapidly multiplying
m g bacteria
a e.g. penicillin) is combine ed with

 
380  
bacteriostatic drug (e.g. erythromycin) which arrests the organism and
prevents its multiplication.
 Synergism: in special cases e.g. combination between sulfadiazine +
penicillin in meningococcal meningitis (both drugs attain high concentrations
in CSF when meninges are inflamed).

█ CLINICAL APPROACHES FOR RATIONAL PRESCRIBING OF ANTIBIOTICS

A. Confirm the presence of an infection

 Fever:

 Fever is considered a hallmark of most infectious diseases.


 Fever is defined as repeated oral temperatures > 37.2 ◦C.
 Fever may be present in absence of infection e.g. in autoimmune disorders
and several malignancies.
 Fever may be absent in presence of infection if the immune system is
depressed.

 White blood cell count:

 Normal WBC is 4000-10,000 cells/mm3.


 Bacterial infections are associated with elevated granulocyte counts
(neutrophils, basophils, and eosinophils).
 Viral, TB and fungal infections are associated with elevated lymphocytic count.
 Parasitic infections and allergic reactions are associated with increased
eosinophilic count.

B. Selection of antimicrobial agents

 Identification of the infecting organism:

 Infected body materials (e.g., blood, sputum, urine, wound drainage, etc.) must
be sampled and cultured before initiating treatment.
 Empirical therapy before identification of the organism is necessary in the
following conditions:
‒ In all acutely ill patients with infections of unknown origin.
‒ Infection in a neutropenic patient, or a patient with meningitis
(characteristically described by severe headache, neck rigidity, and sensitivity
to bright lights). In such conditions any delay in the treatment could be fatal.

 
  381
 Patient factors:

 In neonates, the use of chloramphenicol can lead to shock and cardiovascular


collapse (gray baby syndrome) because liver functions are not well developed.
Also, the use of sulfonamides may lead to kernicterus (brain damage) due to
displacement of bilirubin from serum albumin.
 In growing children, the use of fluoiroquinolones can lead to arthropathy (joint
swelling, stiffness and cartilage damage). Also, the use of tetracyclines can bind
to growing bones and teeth resulting in abnormal teeth and bone formation.
 In old age (> 65 years) the incidence of renal toxicity with aminoglycosides is greater
than in younger patients.
 In immunocompromised patients, the use of bactericidal agents is necessary,
as the host’s immune system is not capable of final elimination of the bacteria.
 Pregnancy: many antibiotics cross the placenta and cause adverse effects to
the fetus e.g. aminoglycosides and tetracyclines.

 Tissue penetration:

 The capillary lining in some tissues, e.g. prostate, the vitreous body of the eye,
and brain form natural barriers to drug delivery due to presence of tight
junctions of the capillary wall.
 Lipid soluble antibiotics e.g. chloramphenicol and metronidazole can cross these
barriers in normal conditions. Penicillin is ionized at physiologic pH and cannot
cross these barriers unless inflammation is present.
 Poor perfusion of some area, e.g. diabetic foot, reduces the amount of antibiotic
reaching this area, making treatment is difficult.

C. Determinants of the rational dosing

The dose and frequency of an antibiotic is based on three important properties.


Understanding of these concepts can improve clinical outcomes and decrease the
development of resistance.

 Concentration-dependent killing: Minimum inhibitory


concentration (MIC):
Certain antibiotics (e.g. aminoglycosides) show
enhanced bacterial killing in concentration above The MIC is the lowest
the MIC. Giving these antibiotics by a single large concentration of antibiotic
dose per day achieves high peak levels and cause in body tissues and fluids
that inhibits bacterial
rapid killing of bacteria.
growth.

 
382  
 Time-dependent killing:

By contrast, β-lactam antibiotics, macrolides, clindamycin, and linezolid do not


exhibit this concentration-dependent property; instead, the clinical efficacy depends
on the time of the drug concentration remains above the MIC. So, preparations with
long duration kill more bacteria.

 Post-antibiotic effect (PAE):

The PAE is a persistent bacterial suppression after levels of antibiotic fall below the
MIC. Antimicrobials with long PAE (e.g. aminoglycosides and fluoroquinolones)
usually require one dose per day.

█ ANTIBIOTIC RESISTANCE

The development of antibiotic resistance is a major problem in clinical practice.


Methicillin-resistant Staph aureus (MRSA) and some strains of Mycobacteria
tuberculosis are examples of multidrug-resistant bacteria.

 Innate resistance:

Innate resistance is a feature of a particular species of bacteria e.g Pseudomonas.


The gene(s) of resistance can be transferred between bacteria by transfer of naked
DNA (transformation), by conjugation with direct cell-to-cell transfer of
extrachromosomal DNA (plasmids), or through bacteriophage (transduction).

 Acquired resistance:

Acquired resistance is when bacteria that were sensitive to certain antibiotic


become resistant with time. Mechanisms responsible for this resistance include:
 Production of enzymes that inactivate the drug.
 Alteration of drug binding site.
 Reduction in drug uptake by the organism.
 Development of altered metabolic pathways.

█ ADVERSE EFFECTS OF ANTIMICROBIAL AGENTS

The adverse effects associated with the use of antimicrobial agents include:

 General adverse effects

 Hypersensitivity or allergic reactions: In form of fever, skin rash, arthralgia,


cholestatic jaundice or hemolysis. More serious reactions are agranulocytosis,
bone marrow aplasia or anaphylactic reaction.

 
  383
 Reactions related to alterations in normal body flora, superinfection or vitamin
B deficiency may follow the use of broad-spectrum antimicrobials. It is due to
inhibition of bacterial flora that suppresses commensal micro-organisms which
present in gut or that forms these vitamins, respectively.
 Resistance

 Direct toxic reactions, resulting from high doses or drug interactions, on


hemopoietic system, liver, kidney, GIT, nervous system or CVS.

█ SUPERINFECTION (Opportunistic infection):

Administration of antimicrobials usually alter bacterial flora but with no ill effect in
most cases however, broad-spectrum antibiotics if used for long time may alter or
kill bacterial flora. So, the bacteria and fungi that are normally inhibited by bacterial
flora will multiply leading to superinfection (its early manifestation may by diarrhea).
It is caused by staphylococci, Pseudomonas, proteus, Candida albicans or
Clostridia difficile... etc. Superinfection may be vaginal, oral, pharyngeal or even
systemic infection e.g. staphylococcal enterocolitis, candidiasis or
Pseudomembranous colitis (= antibiotic-associated diarrhea).

Treatment:
Stop the causative agent and give drug, which kill the organisms responsible for
superinfection e.g. staphylococcal enterocolitis, which is treated by metronidazole
or vancomycin orally, antifungal nystatin for candidiasis.

█ GENERAL PRINCIPLES OF THERAPY WITH ANTIMICROBIALS

 Antimicrobials should only be given when necessary and after antimicrobials


susceptibility test whenever possible.
 The pharmacokinetics of the drug should be taken into consideration e.g. the
state of hepatic and renal functions of the patient.
 In serious infection it is better to start with a parentral loading of a bactericidal
agent to avoid emergence of resistant strains by giving adequate dosage for
sufficient duration and adapting proper combination regimens.
 Antimicrobials should be continued for 3 days after apparent cure is achieved to
avoid relapse.

 
384  
Part 2
2: Individuall Classe
es of Anttibiotics
s

A. CEL
LL WALL INHIBITOR
RS

-LACT
TAM ANTIBIOTICS

They sshare a -lactam ring in the


eir molecular
structuure which is required for antibacterrial
activityy.
 Pen nicillin
 Cep phalosporrins
 Mo onobactam ms
 Carrbapenem m

Peniciillins

Antibacterial spe
ectrum:
 Gra
am-positivee cocci, e.g. sstreptococ cci,
pne
eumococci and staph hylococci.
 am-negativve cocci: gonococci a
Gra and menin ngococci.
 Gra
am-positivee bacilli: an
nthrax baccillus, C. diphtheria and clostrid
dia.
 am-negativve bacilli: shigella,
Gra s sa
almonella, pseudomo
p onas…etc.
 Spiirochetes: Treponem ma pallidumm.
 Acttinomyces..

Mecha
anism of action:
a
 Pennicillins bin
nd to pennicillin bin
nding pro oteins
(PBBPs) on th he bacterrial cell w
wall and in nhibit
tran
nspeptidattion reactio
on essentia al for bactterial
cell wall syntthesis.
 Thee next sttep is ac ctivation o of intraceellular
autolytic enzzymes (autolysins) leading to o cell
rup
pture. Thus, the antibacterial efffect of pennicillin
is the resultt of both h inhibitionn of cell wall
synnthesis andd destructio
on of existting cell waall by
autoolysins.
 Pennicillins are
e bactericcidal espe ecially for Gram-positive baccteria whic ch have
thic
ck cell wallss.
 Thee major cau use of resis
stance is th
he producttion of β-la
actamase (penicillina
ase).

 
  385
Pharmacokinetics:
 Absorption of oral penicillins is decreased by food. They must be administered
1hr before or 2 hr after meals.
 Route of administration of a B-lactam antibiotic is determined by the stability of
the drug to gastric acid and by the severity of the infection.
 Oral in moderate infection and acid stable preparations e.g. penicillin V.
 Paraentral in severe infection and acid sensitive preparations e.g. penicillin G
 Penicillins can penetrate to CSF and ocular fluid only during meningitis. They
cross placental barrier but they are not teratogenic.
 Most penicillins are excreted through organic acid secretory system via the
kidney. The renal excretion in proximal tubules could be decreased by co-
administration of probenecid which prolongs its duration of action.

PREPARATIONS OF PENICILLINS:

1.BENZYL PENICILLIN (PENICILLIN G) and RELATED DRUG:

Effectiveness: active against gram-negative and positive cocci, gram-positive


bacilli and spirochetes. It is sensitive to penicillinase enzyme.

Classification:

A. Injectable Short-acting preparation e.g. benzyl penicillin (penicillin-G):

- It is the prototype of all penicillin family.


- Primarily covers gram-positive organisms (Pneumococci, streptococci,
staphylococci (non-penicillinase-producing), Cl. perfringens,C. diphtheria) with
little gram-negative coverage (Gonococci and meningococci[but not bacilli]),
making it ineffective in urinary tract infection.
- Others: Treponema pallidum (syphilis).
- It has some limitations:
 It is acid labile and must be given parentrally.
 It has a short half-life, so frequent injections are required.
 It is easily inactivated by -lactamase enzyme.
 It has narrow spectrum.

B. Injectable Long-acting preparation

i. Penicillin G Procaine:
It is a suspensions of combination of penicillin G with procaine that have longer
duration (12-24 hrs) allowing reduced frequency of injections. So it is injected just
once every day, intramuscularly.

 
386  
ii. Benzathine Penicillin: This preparation produces low blood levels lasting from
few days to 4 weeks depending on the dose. So it is used in chemoprophylaxis
and in the treatment of syphilis.

C. Oral Penicillin V: it is an acid stable form of penicillin G (given orally).

2.PENICILLINASE OR -LACTAMASERESISTANT PENICILLINS (ANTI-STAPH PENICILLINS)


[Methicillin, cloxacillin, dicloxacillin, flucloxacillin, and nafcillin]

- They are effective against -lactamase producingStaphylococci.


- They have low or no activity against other gram-positive and gram-negative
organisms.
- The use of these agents is now declining due to increased incidence of
methicillin-resistant S. aureus (MRSA).

3.EXTENDED SPECTRUM PENICILLINS


[Ampicillin and amoxicillin]

- They maintain gram-positive coverage with extended coverage of more gram-


negative bacteria e.g. Salmonella, H. influenza, Proteus, and Shigella (but not
Pseudomonas).
- They can be given orally (acid resistant) and
by injection (IM and IV). Common organisms
capable of producing
- They are inactivated by -lactamase, so they penicillinase:
are usually combined with clavulanic acid or
sulbactam (-lactamase inhibitors) to cover Staphylococcus aureus,
Escherichia coli,
-lactamase producing bacteria. H. influenzae
[Amoxicillin/clavulanic acid (co-amoxiclav) Pseudomonas aeruginosa
Neisseria gonorrhoeae
- Ampicillin/sulbactam (Unasyn)].
Proteus
- Ampicillin is concentrated in bile so it is Bacteroidesspecies.
effective in gall bladder disease and typhoid
carrier.
- Ampicillinis more active on shigella and H. influenza.
- Pro-drugs of ampicillin e.g. pivampicillin and talampicillin are esters of
ampicillin which themselves are microbiologically inactive but after oral
administration they are de-esterified in the gut mucosa or liver to release
ampicillin to the systemic circulation. They are better absorbed from the gut
than ampicillin itself, so gives higher levels in blood and tissues and they have
no effect on the gut flora and less G.I. upset.
- Amoxicillin has better absorption and tissue penetration than ampicillin. So, it
has no effect on the gut flora and less G.I. upset.

 
  387
- Amoxicillin is more active against Salmonella and Streptococcal fecalis.
- Amoxicillin can penetrate mucoid and purulant sputum, so it is useful in
chronic bronchitis.

4.ANTIPSEUDOMONALPENICILLINS
[Ticarcillin, Azlocillin, and Piperacillin]

- They are broad-spectrum, but should only be used for pseudomonas infection
and ampicillin resistant proteus. Also they have activity against anaerobic
gram negative bacteria e.g. Bacteroids fragilis (a common pathogen in intra-
abdominal sepsis)
- They have synergistic effect when they are used with aminoglycosides.
- They are inactivated by -lactamase so they are combined with clavulanic
acid forming Timentin to cover -lactamase producing bacteria.

PENICILLIN G ANTI-STAPH EXTENDED ANTI-


PENICILLINS SPECTRUM PSEUDOMONAL
PENICILLINS PENICILLINS
Cloxacillin Ampicillin Ticarcillin
Flucloxacillin Amoxicillin Azlocillin
Nafcillin Pipracillin
Spectrum Gram +ve spp. Staphylococci Gram +ve and Pseudomonas
and few Gram Gram –ve and Gram –ve
–ve cocci spp. (except spp.
T. pallidum Pseudomonas)
-lactamase Yes No Yes Yes
susceptibility They are combined with
clavulanic acid or sulbactam
to cover -lactamase producing
bacteria.

Therapeutic uses of penicillins:

A. Treatment: Penicillins may be used in the treatment of:

 Streptococcal infections, e.g. wound sepsis, acute throat infections, subacute


bacterial endocarditis,.. etc.
 Staphylococcal infections of skin, mucous membrane and bone.
 Pneumococcal infections e.g. pneumonia.
 Syphilis (5 million units single dose) and gonorrhoea.
 Meningococcal infections: Penicillin diffuses into the CSF only when the
meninges are acutely inflamed.
 Typhoid and paratyphoid fevers: ampicillin and amoxycillin.

 
388  
 Pse
eudomona as infection
n: (ticarcillin
n or pipera
acillin)
 Acttinomycosis, Anthrax x and H. in fluenza inffections.
 Dip
phtheria, teetanus andd gas gang grene. (Pennicillin may
y be used together with
w the
spe
ecific antito
oxins).

phylaxis: Penicillins
B. Prop P may be ussed prophy
ylactically in the follo
owing cond
ditions:

 To prevent reecurrence of rheuma atic fever: Benzathine


B e penicillinn, 1.2 millio
on units
IM once a mo onth.
 To prevent subacute
s bacterial
b e
endocarditis due to bacteraem mia resultin ng from
opeerative pro
ocedures such
s as deental extraaction, tonsillectomyy…etc. in patients
p
with
h congenittal or acqu
uired valvullar disease
e or immunnocomprom mised patient.

Advers
se effects::

■ Hyp
persensitiivity reactions:
‒ It occurs in ~10% off patients rreceiving penicillin.
p
‒ It occurs with
w all typ pes of peniicillin. Allerrgy is not due
d
to peniciillin itself but to degradattion product
common to t all peniccillin
‒ It is mo ore comm mon after parentera al than oral
o
administraation.
‒ All types of reactio ons, from simple ra ash to acu ute
anaphylax xis and angioedem
a ma, can oc ccur withinn 2
minutes (e ype I hype
early or ty ersensitivityy) or up to 12
days (delaayed or type II) afterr administrration.
‒ True anap phylaxis is rare (1:10’ 000 of cas ses).
‒ Penicillin allergy
a is unpredicta
u able i.e. an n individuaal who toleerated penicillin in
the past may
m develo op allergy l ater on and vice vers sa.

Pre
evention:
 Never give penicillin
n if there iss history off penicillin allergy.
 Test for hypersensittivity

Ma
anagement of anaph hock: see CVS.
hylactic sh

■ Oth
her advers se effects:
‒ Penicillin in
i high dos ses can ca ause neuro otoxicity and seizurees in patien nts with
renal failurre.
‒ Nafcillin iss associate
ed with neu utropenia and
a thrombocytopennia.
‒ Methicillinn causes innterstitial n ephritis (an
nd is no longer used for this re eason).
‒ Co-amoxiclav and flucloxacilli n cause he epatotoxiccity (cholesstatic jaund
dice).

 
  389
Drug in
nteraction
ns:
 Bac cteriostatic drugs s (e.g. te etracyclinee, chloroa amphenico ol, erythro omycin)
inte
erfere with the action n of penicilllin becausse penicillin acts by iinhibiting cell
c wall
synnthesis (i.e. kills rapid
dly multipl ying bacteeria) while bacteriosttatic drugs arrest
the organism and preve ents its muultiplication
n.
 Anttipseudom
monal penic cillins (acid
dic drugs, ―ve
― charge ed) form coomplex with amin-
oglycosides (basic
( drugs, +ve charrged) if they
y are mixed
d in the sam
me infusion
n fluid.

Cepha
alosporins

 Cep
phalosporin
ns also hav
ve a β-lac
ctam ring and
a have
the same mechanisms s of action like penicillins.
 Eacch newer generatiion of ccephalospo orins is
incrreasingly re
esistant to
o β-lactammase.

Pharm
macokinetics:
 Cep
phalosporin
ns are wid
dely distrib
buted in bo
ody fluids;; memberss of the th
hird and
fourrth generattions can penetrate
p tto CSF (exxcept cefop perazone).
 Likee penicillin
ns, most cephalospo orins are excreted
e via the kidnneyand hence the
dosse must be e adjusted in patientt with rena al insufficie
ency. The renal excrretion is
alsoo decrease ed by probenecid.
 Cefoperazonee and ceftrriaxone (thiird generattion) are primarily exxcreted in bile
b and
its sserum leveel is not gre
eatly influe
enced by re enal failure
e.

Antibacterial spe
ectrum An
nd Classiffication:
They a d into 1st, 2nd, 3rd, a
are divided and 4th gennerations. The differrence amo ong the
groupss is markeed by cha anges in a antibacteria
al spectruum. In genneral, the activity
againstt gram-possitive bacteria decreeases from first to third generaation while activity
againstt gram-neg
gative orgaanisms inccreases.

1. FIRSTT-GENERATTION CEPHA ALOSPORINSS:


[Cepha alexin (Kefflex), ceph
hradine (Ve
elosef), ceffadroxil (D
Duricef)].

 Prim
marily cover gram-p positive oorganisms t penicilliin G) with
s (similar to h some
gram-negativ ve covera age (E. coiil, Klebsiellla), making
g them eff ffective in urinary
trac
ct infection
n.
 Theey are sens sitive to β--lactamas se.
 Theey do not penetrate
p to CSF (evven in pres
sence of meningitis).
m .
 Theere is partia
al cross allergenicity between them
t and penicillins.
p

 
390  
2.SECOND-GENERATION CEPHALOSPORINS
[Cefoxitin, cefuroxime (Zinnat), cefaclor].

 They maintain gram-positive coverage with enhanced coverage of gram-negative


bacteria e.g. H. influenza, Neisseria, Proteus, (but not Pseudomonas).
 They are used primarily in the management of urinary and respiratory tract,
bone, and soft-tissue infections and prophylactically in various surgical
procedures.
 They are relatively resistant to -lactamases.
 They do not penetrate to CSF (even in presence of meningitis, except cefuroxime).

3.THIRD-GENERATION CEPHALOSPORINS
[Cefotaxime (Clarofan), Ceftazidine, Cefoperazone (Cefobid), Ceftriaxone (Rocephin)

 They maintain gram-positive coverage with excellent coverage of gram-negative


bacteria(including Pseudomonas)and active against anaerobes.
 They can penetrate to CSF in presence of meningitis (except cefoperazone).
 They are excreted by the kidney, except cefoperazoneand ceftriaxone which are
excreted biliary allowing their use in renal failure.
 They are highly resistant to -lactamase.
 They have long duration (12 hrs) and ceftriaxone has longer duration (24 hrs)

4.FOURTH-GENERATION CEPHALOSPORINS

 Cefepime has a powerful coverage against most Gram-positive and Gram-


negative bacteria (including Pseudomonas) and anaerobes.
 Ceftaroline is sometimes described as “FIFTH GENERATION” drug. .

Therapeutic uses:
 Severe undiagnosed sepsis especially in immunosuppressed patient.
 Treatment of infection of respiratory tract, urinary tract, skin, soft tissue, bones
and joints due to susceptible organisms.
 Gram-negative bacterial meningitis may be treated by cefotaxime (third
generation) and ceftriaxone that reach the C.N.S.
 Biliary infection: 3rd generation (cefoperazone or ceftriaxone).
 Gonorrhoea due to penicillin-resistant Gonococci. It is treated by single IM
injection of ceftriaxone.
 Pseudomonal infection when aminoglycosides are not desirable.

 
  391
1ST GENERA
ATION 2NDD GENERATIION 3RD GE
ENERATION
N 4TH GENE
ERATION

Cephalex xin Ceefoxitin Cefootaxime Cefipim


me
Cephradine Ceefuroxime e Cefooperazonee
Cefadroxxil Ceefaclor Ceftrriaxone
Spectrrum Mainly Grram Ma aintain Gra
am Excellent Wide Grram
+ve spp. and +vve coveragge Gramm –ve +ve andd Gram
few Gramm –ve an
nd enhance ed coverage –ve cov verage
bacilli Grram –ve (inclu
uding including
co
overage Pseudomonas) Pseudomonas
-lacta
amase Yes Yees No No
suscepptibility
Penetrration to No even in No
o (except Yes (except
( Yes
CSF meningitis
s ce
efuroxime) cefop
perazone)
Cross allergy High Mo
oderate Low Low
with pe
enicillin
Elimina
ation Renal Re
enal Rena
al & biliary Renal & biliary

Advers
se effects::

‒ Hyp
persensitivvity reac ctions: 1 0% of penicillin--sensitive persons share
hyp
persensitivity to ceph halosporin
ns. The crooss hypersensitivity with penicillin is
rd th
mucch lower with
w the 3 and 4 ge eneration drugs.
d
‒ Som
me first generatio on cephallosporins are neph hrotoxic especially
y when
adm
ministered with loop diuretics
d o
or aminogly
ycosides.
‒ Somme third generation
g n cephalossporins (e.g
g. cephope
erazone) innhibit the enzyme
e
vitam
min K ep poxide re eductase leading to o hypoproothrombin nemia (co ould be
prevvented by vitamin
v K).
‒ Supperinfectio membranous colitis)): Cephalo
on with C.. difficile (pseudom osporins
are the main n cause off hospital--acquired C. difficile
e colitis, a potentia
ally life-
eatening in
thre nfection.

Pseud
domembra
anous collitis

Pseud domembran nous colitis (= antibioti c-associate


ed diarrhea or C.
le colitis), can occ
difficile cur followwing antibiotic treatment.
When antibiotics are given, most of thee resident in ntestinal ba acteria
are killled with sp paring of soome types like Clostri e. The
ridia difficile
norma ally harmlesss C. difficcile grow raapidly because of lack of
compe etition with
h other florra, and pro oduce toxin ns. These toxins
t
damag ge the inner wall of the
e intestines and cause abdominal pain,
fever aand diarrhe ea. Severe cases
c can b
be life-threatening.

ment:
Treatm
‒ Stop all antibiotics when never possibble.
‒ Orral metroniddazole 250 1st choice).
0 mg /6 hr (1
‒ If m
metronidazoole failed, give
g oral van
ncomycin solution
s 125
5 mg/6 hr

 
392  
Drug interactions:
 If IV ceftriaxone is added to any IV calcium-containing solution (e.g. Ringer’s
or parenteral nutrition) dangerous particulates of ceftriaxone-calcium can result
and precipitate in the lungs and kidney. Separate between IV ceftriaxone and any
IV calcium-containing solution by at least 48 hours.
 Some third generation cephalosporins inhibit the enzyme aldehyde
dehydrogenase. If they are co-administered with alcohol, acetaldehyde
accumulates in blood leading to nausea and vomiting (disulfiram-like reaction).

█ OTHER beta-LACTAM DRUGS

Aztreonam (Azactam)

 Aztreonam is a monobactam that share similarity in structure and mechanism


with penicillin but is highly resistant to β-lactamase.
 Aztreonam is especially effective against β-lactamase producing gram-negative
bacteria. It could be used as alternative to aminoglycosides in severe
respiratory, urinary, biliary, GIT and female genital tracts infections.
 It has no cross-hypersensitivity with penicillins or cephalosporins.
 Most of the Adverse effects reported have been, for example, hematological
effects, nephrotoxicity and pseudomembranous colitis,.

Carbapenems [Imipenem-cilastatin, meropenem]

 They are broad spectrum antibiotics that share similarity in structure and
mechanism with penicillin but they are highly resistant to β-lactamase.
 They are one of the antibiotics of last resort for many bacterial infections.
 The ease of penetration (due to its low molecular weight) and resistance to -
lactamase imparts a broad-spectrum of antimicrobial activity against most
aerobic and anaerobic bacteria (Gram-positive and gram-negative) with the
exception of occasional Pseudomonas strains.
 If imipenem is given alone it is inactivated by dihydropeptidase enzyme in kidney
leading to low urinary excretion and significant renal toxicity in animals, therefore
it is combined with cilastatin (Tienam) to inhibit renal dihydropeptidase enzyme.
 Meropenem does not undergo metabolism by renal dihydropeptidase enzyme.

Adverse effects:

 Blood disorders.
 Seizures in high doses. Meropenem is less likely to provoke seizures.
 G.I.T: nausea, vomiting, etc.

 
  393
█ OTH
HER CELL
L WALL INHIBITORS
S

Vanco
omycin

 Van ncomycin inhibits ce ell wall synnthesis and enhance es cell lysiss.
 It is active agaainst gramm-positive organism ms.
 It iss given byy slow i.v. infusion tto treat se erious MR RSA infec ctions in patients
p
allergic to pennicillins or cephalosp porins.
 Bec cause it iss not abso orbed orallly, it is us sed by thiss route too treat anttibiotic-
ass sociated enterocolit
e tis (C. diffficile colitis
s) and oth
her infectioon by susc ceptible
orga anisms.
 Rap pid infusio
on of vanc comycin m may cause e anaphylactoid reaactions an nd “red
man n” syndroome (skin rash and flu ushing due e to histam
mine releasse).
 Rarrely, high levels of vancomyc
v in may ca ause perm manent aud ditory imp
pairment
(otootoxicity) and
a nephrrotoxicity.

Bacitrracin

 Bac hibits the building of p


citracin inh peptidoglyycan blocks (cell walll units).
 It is most activve against gram-pos sitive bacteria.
 It iss used only topically y in comb bination witth neomyccin or pollymyxin fo or minor
skin n infectionss due to seerious nep hrotoxicity
y.

B. INHIBITORS OF
O BACTE
ERIAL PRO
OTEIN SY
YNTHESIS

Macro olides
[Erythrromycin (p
prototype)), azithrom
mycin, clarrithromycin, roxithrromycin]

Mecha
anism of action:
a
 The
ey bind reversibly
r to the 50S ribo osomal
bunit and inhibit bac
sub cterial prootein synthhesis.
 ey are bactteriostatic in low co
The oncentration
ns and
bac
ctericidal in high conce
entrations.

Antibacterial spe
ectrum:
Macrolids are effective against a numb ber of
organissms, inclu
uding gram m-positivee bacteriaa, e.g.
pneumoncocci, staphyloco
s occi, strep
ptococcus species, C.C diphtheeria, some e gram-
negativve bacteria, e.g. Neisseria, H H. influenzza, and in
ntracellularr microorg
ganisms
(Mycopplasma spe ecies, legio
onella and Chlamydia a).
 
394  
Pharmacokinetics:
 Absorption of erythromycin is affected by food and HCl. To minimize destruction
and enhance absorption, erythromycin is administered as esters salts.
 Newer macrolides are acid stable and can be given orally.
 They reach all body fluids including prostate, placenta and milk but only small
amounts can penetrate to CSF.
 Azithromycin and roxithromycin are concentrated in neutrophils, macrophages
and lung tissue, so they have long t1/2 (given once daily) and they are useful
against intracellular organisms.
 Macrolides are concentrated in liver and excreted primarily in active form via
bile with only low levels found in urine.
 Azithromycin and clarithromycin are converted into active metabolites.

Therapeutic uses:

 Treatment and prevention of infections caused by staphylococci, streptococci


and pneumococci (gram-positive) in penicillin-hypersensitive individuals.
 Eradication of C. diphtheriae from pharyngeal carriers (1st choice).
 Azithromycin and roxithromycin are especially effective against intracellular and
atypical bacteria (chlamedia, mycoplasma, legionella)
 Clarithromycin has activity against Toxoplasma. It is also effective against
Helicobacter pylori in peptic ulcer.
 It is a second line drug for the treatment of gonorrhea and syphilis.
 Treatment of middle ear and sinus infections, since the causative agents, H.
influenza and Strep pneumonia are usually sensitive.

Adverse effects:

‒ Epigastric distress especially with erythromycin.


‒ Erythromycin may cause cholestatic hepatitis probably due to hypersensitivity
reaction to the oral estolate form or hepatoxic effect when drug therapy lasts
longer than 10 days or repeated courses are prescribed.
‒ Erythromycin and clarithromycin inhibit hepatic CYP450– metabolism of some
drugs e.g. warfarin and phenytoin (Azithromycin is devoid of this side effect).
‒ Bacterial resistance if it is used more than one week.
‒ Hypersensitivity reactions: fever, rashes and cholestatic hepatitis.

Interactions:

 Combination of the therapeutic dose of erythromycin with penicillin antagonizes


the bactericidal effect of penicillin.
 Erythromycin decreases CYP450 enzymes, thus increase serum concentration of
theophylline, oral anticoagulants, cyclosporins, and digoxin.

 
  395
Lincos
samides: Clindamycin

 Thee lincosam
mide family
y of antib
biotics inc
cludes linc
comycin aand clinda
amycin.
Clin ndamycin is semisyn nthetic derrivative of lincomycin and is m more poteent than
lincomycin.
 Clin ndamycin is i similar, in the mecchanism and kinetics s, to erythhromycin but has
activity againsst anaerob bic bacterria.
 It iss used as an alterna ative drug for treatment of ana aerobic in nfections. Topical
prep parations are
a used fo or treatme nt of acnee.
 Foo od in the sttomach do oes not intterfere withh the abso
orption of cclindamycin. So it
is completely absorbed after oral a administra ation.
 App proximatelyy 90 % are a plasma a protein bound. Clindamycinn penetratte most
tissues well, including bone. The erefore, bone and joint infecctions cau used by
susceptible organisms respond
r w
well to treattment with clindamyccin.
 It iss associateed with higgh incidencce of diarrh hea and ps
seudomem mbranous s colitis
as side effec ct due to superinfec
s ction by re esistant clostridia inn addition to side
effe ects of erytthromycin.

Amino oglycosides
[strepto
omycin (prrototype), Gentamiciin, tobramycin, amik
kacin, kana
amycin,
neomy ycin]

Mecha
anism of action
a
 Am
minoglycosiides are transportedd across the inner
celll membranne by active transpo
ort system
m present
onlly in Gram
m-negative
e aerobic sspp.
 Insiide the ce
ell, they bin
nd to 30S ribosoma
al subunit
and
d inhibit ba
acterial pro
otein synth
hesis.
 Theey are bac ctericidal. Bacterial
B kkilling is co
oncentra-
tion
n-depende ent i.e., the
e higher the e concentrration, the
more bacteria a are killed,, so they aare better given
g as a
sing
gle large do
ose daily.
 Appost-antibiotic effec ct is also p
present i.e., residual
bac
ctericidal activity is s present after the serum
con
ncentrationn falls belo
ow the MIC C.

Antimicrobial sp
pectrum:

They have narrow spectru um againstt mainly Gram


G ative bacillli, very few
nega w Gram
positive
e cocci, an
nd Mycoba
acteria TB..

 
396  
Pharmacokinetics:
Penicillins and
 All aminoglycosides are not absorbed orally aminoglycosides
(must be given parentrally) and cannot
The antibacterial effects of
penetrate to CSF because they are highly
all β-lactam antibiotics are
polar compounds. synergistic with the amino-
 They can cross the placental barrier and may glycosides. Because penicil-
cause congenital deafness. lin inhibits cell wall synthesis
and facilitate the entry of
 They are excreted unchanged in urine by
aminoglycosides to inside
glomerular filtration (dose adjustment is the bacterial cell.
necessary in renal dysfunction). They become
[Note: these drugs should
more active in alkaline urine.
never be combined in the
same infusion fluid because
Therapeutic uses: the basic aminoglycosides
form inactive complex with
The role for aminoglycosides has decreased
the acidic penicillin]
substantially due to their narrow spectrum and
potential toxicity.

 Streptomycin is currently used only for plague, brucellosis and it is one of the
1st line drugs for TB.
 Gentamicin and tobramycin are used in the treatment of gram-negative
infections e.g. urinary tract and respiratory infections.
 They are used in gram-negative septicemia, usually combined with penicillin
and/or metronidazole.
 Pseudomonas infections: they are more efficient when combined with
antipseudomonal penicillin
 A combination of vancomycin and gentamicin is useful in the treatment of
enterococcal endocarditis.
 Neomycin is used orally for hepatic encephalopathy to suppress intestinal
bacteria that produce ammonia.
 Neomycin is used orally for sterilization of intestine before surgery and
bacillary dysentery.
 Neomycin and gentamycin are used topically for skin and eye infections often in
combination with polymixin B or bacitracin.

Adverse effects:

Aminoglycosides have a narrow therapeutic index; it may be necessary to monitor


serum concentrations and individualize the dose especially in old age.

■ Nephrotoxicity:
‒ It is due to accumulation of aminoglycosides in the renal tubular cells.

 
  397
‒ It ranges from mild reversible effect to severe irreversible toxicity.
‒ It is increased by co-administration of other nephrotoxic drugs e.g.
cephalosporins.

■ Ototoxicity:
‒ It is due to accumulation of aminoglycosides in the endolymph and perilymph
of the inner ear causing damage to the hair cells in the organ of Corti.
‒ It may affect the cochlear (auditory) or vestibular functions.
‒ Deafness may be irreversible.
‒ It is increased by co-administration of other ototoxic drugs e.g. loop diuretics
and in old age and with renal diseases.

■ Neuromuscular block:
‒ Often occurs after direct intraperitoneal or intrapleural application of large
doses of aminoglycosides leading to skeletal muscle weakness.
‒ It is treated by immediate administration of calcium gluconate or neostigmine.

■ Malabsorption:
- Occasionally observed following the oral administration of streptomycin,
neomycin, kanamycin. It is due to binding with bile salts and inhibition of
pancreatic lipase leading to steatorrhoea and diarrhoea.

Drug interactions:

 With antibiotics:
 With cephalosporins nephrotoxicity increases.
 Anti-pseudomonal penicillins, and cephalosporins decrease the antibacterial
effect of gentamicin if combined together in the same syringe (in-vitro)
because penicillins are acidic and aminoglycosides are alkaline.

 Skeletal muscle relaxants:


 Aminoglycosides increase the effect of non-depolarizing NMB agents (see ANS).
It could be reversed by neostigmine and calcium gluconate.
 Aminoglycosides should be administered with great caution during surgery or in
the post-operative period.

 Anticoagulants:
 With oral anticoagulants: oral aminoglycosides impair vitamin K production by
intestinal bacteria potentiating the effect of anticoagulants.
 Heparin precipitate aminoglycosides (avoid their mixing in the same syringe).

 Diuretics: Diuretics e.g. ethacrynic acid, and mannitol potentiate ototoxicity of


aminoglycosides.

 
398  
Tetrac
cyclines
[Oxyte
etracycline
e (prototyp
pe), doxyc
cycline, tig
gecycline]

Mecha
anism of action:
a
 Tetracyclines bind re eversibly to the 30S
3
ribo
osomal subunit
s and
a inhib
bit bacte
erial
prootein synth
hesis.
 Theey are bactteriostatic
c.

Antibacterial spe
ectrum:
Tetracyyclines display broadd-spectru m activity and are efffective agaainst:
 Mosst gram-po ositive, ma
any gram-nnegative ba
acteria andd Brucella.
 Rick kettsia, Co
oxiella, Myccoplasma and Chlam mydia (intra
acellular orrganisms).
 Spirrochetes, Actinomyc
A cines, Proto
ozoa.
 Heliicobacter pylori.
p

macokinetics:
Pharm

 Theese antibiiotics are partiallyy absorbe ed from the stom mach and upper
gasstrointestin
nal tract an
nd the amo ounts rema aining may alter bactterial flora leading
l
to s
super infe ection. Beccause abso orption of doxycycline is rapid d and commplete, it
hass weak effeect on intestinal flora
a.
 Callcium (milk and Ca. C antaciids), mag gnesium (Mg hydro oxide), alu uminum
hyd
droxide an nd iron intterfere witth their absorption since theey form in nsoluble
cheelates with tetracyclin
nes.
 The a body fluids and pllacenta du
ey reach all pid solubiliity but only small
ue to its lip
mounts can
am n penetratee to CSF (DDoxycyclin ne is the most
m lipid--soluble).
 Beccause of th heir chelatting propeerties with calcium th hey tend to o be depoosited in
gro
owing bone es and teetth causing g yellow dis
scoloration n of teeth.
 Thee primary route
r of elimination iis renal. Doxycyclin
D ne is excre eted in sto ool and
doees not acc cumulate in patientss with renal impairme ent.

Therap
peutic use
es:

 Tetracyclines have brooad spectrrum activiity againstt both Grram-positive and


Graam-negatiive organis
sms, but t heir use is
s now declining becaause of inc
creased
resistance an
nd the deve
elopment o
of safer druugs.
 Dox
xycycline is used as s alternatiive to ma
acrolides in Chlamyd
dia, Mycop
plasma,
and
d Legionellla infection
ns.
 The
ey have broader spe ectrum tha
an macrollides again
nst other atypical bacteria
b
(Bo
orrelia, Rickkettsia, Cox
xiella):

 
  399
- Borrelia bu
urgdorferi:: is the cau
use of Lym
me
disease, a sp
pirochetal infectio on
transmitted
d by infectted ticks. T The diseas se
consists of skin rash,, fever, and
d arthritis.
- Rickettsia:: is a gen
nus of sm
mall obligatte
intracellula
ar bacteria
a causing ttyphus an
nd
Rocky Mo ountain sppotted fevver.
- Coxiella burnetii:
b is a sma
all obligatte
intracellula
ar bacteriu
um causing
g Q fever.
 xycycline is used orally (100 m
Dox mg/12h for 3
months) for tre
eatment of acne vulgaris.
 xycycline is effective
Dox e in treatm
ment of cho
olera (300 mg single oral dose)).

Advers
se effects::

- Epiigastric distress (nausea, voomiting, ep


pigastric burning
b and hyperacidity)
a
resulting from m irritation
n of gastrric mucosa a. This ca
an be prevvented by
y giving
tetrracycline after
a meals.
- Yelllow stain ning of bo oth the d deciduous and
perrmanent te eeth and dental enam mel hypoplasia
(inc
creased su usceptibilitty to cariees, as well as
retaardation of
o bone grrowth) bec cause of their
cheelating pro operties with
w calciu um. This can
occcur if tetrac
cyclines arre adminisstered afterr the
fourth month of gestatio on or if the
ey are give
en to
chilldren lesss than 8 years
y of a age, this is a
furtther reasoon for avo oiding the eir use duuring
preegnancy.
- Cho
olestatic hepatitis
h particularly
p y in pregnaant women n and at laarge doses.
- Pho y: skin reaction in the
ototoxicity e form of sunburn,
s occurs wheen a patiennt taking
tetrracyclines is exposed
d to sunlig ht or ultrav
violet rays..
- Sup on with re
perinfectio esistant sttaphylococ cci or C. difficile (C
C. difficile colitis).
Also, decreassed syntheesis of Vit. B12 leads to sore tongue & blaack hairy to ongue
- Terratogenessis, when administere
a ed early in pregnancy.

Contra
aindication
ns:

 Sevvere hepattic disease and renal disease.


 It should be avoided
a in patients w
with peptic ulcer,
 In c
children (le
ess than 8 years
y of ag
ge)
 In p
pregnancy and lactattion.

 
400  
Interac
ctions:

 Avooid simultaaneous inggestion of ddairy products (milk & cheese) , antacids,, or iron
withh tetracycllines becauuse of che
elating actio
on of tetracyclines.
 Tettracycliness decrease vitamin K synthesis in intestinal lumen d due to inhib
bition of
inte
estinal flora
a leading to potentiat
ation of ora
al anti-coag
gulant effecct.

Chlora
amphenic
col

anism of action:
Mecha a

 Chlloramphen nicol binds to the 50S ribo osomal


subbunit and inhibits
i ba
acterial prrotein syntthesis.
 Thee drug is either
e c (more co
bactteriostatic ommon)
or b
bactericida
al dependin
ng on the o organism.

Pharm
macokinetics:

 Duee to its lipo


ophilic natuure, chloraamphenico ol is absorb
bed rapidlly, so there
e is less
alte
eration of bacterial flora wiith less susceptibil
s ity to suuperinfectioon and
disttributed throughout body
b fluidss.
 Theerapeutic le evels can penetrate
p tto the CSF
F and placeenta.
 Chlloramphen nicol is inac
ctivated in the liver by
b glucuronyl transfferase.
 Elim
mination iss mainly re enal. Abou ut 10 % of o the adm ministered drug is excreted
uncchanged (sso it is not useful in U UTI).

peutic use
Therap es:

 Chlloramphennicol has broad


b spec
ctrum against both Gram-neg
G gative and Gram-
sitive spp.., but its us
pos se is limite
ed because of its sev
vere toxicitty.
 It c
could be used as ann alternativve to macrrolides and
d tetracycllines for diseases
cauused by aty
ypical bac
cteria (Myc coplasma, Legionellaa, Rickettsiia, Coxiella
a, etc).
 As an alterna ative to pe enicillin an
nd cephaloosporins fo
or treatmeent of meningitis
(due to H. inflluenza), typ
phoid feve er, and anaerobic in
nfections.
 Use
ed as eye drops
d (0.5%) for eye
e infection
ns.

Advers
se effects::

 mia and bo
Apllastic anem one marrow
w suppres
ssion:
‒ It is dose dependent
d t – rare bu t fatal.
‒ It may occcur weeks or monthss after stop pping of the drug.
‒ It may be a result of the inhibittion of hum
man mitoch hondrial prrotein synthesis.
‒ Managem ment of BM suppressiion: see blood.

 
  401
 Gra
ay baby sy
yndrome:
‒ It occurs in neonate es becausse their live
er
cannot me etabolize chloramphe
c enicol.
‒ It consissts of cyanosis, collapse,
abdomina al distensio
on, and sho
ock.
‒ Mortality is high (40%
%).

 Inhibition of cytochro ome P-450 0 enzyme


es
can
n increase levels of other
o drugss.

C. INHIBITORS OF
O BACTE
ERIAL NU CLEIC AC
CID SYNTH
HESIS

Fluoro
oquinolo
ones
[Ofloxa
acin, norflo
oxacin, cip
profloxacin
n, levofloxacin, moxifloxacin, g
gemifloxac
cin]

Quinoloones are synthetic


s analogues
a of Nalidixic acid. The earlier q
quinolone nalidixic
n
acid did not achieve systtemic anti bacterial levels and d is usefuul only as urinary
antisep
ptic. Fluo orination was fou und to produce
compo ounds with greater antibacteria al activity, achieving
useful d
drug levelss in the blo
ood and tisssues.

Membe
ers:

 Firsst generatioon: Nalidix


xic acid.
 Sec cond generration: Pipemidic aci d.
 Thirrd generattion: Mostt are fluorrinated. Th hey include ciproflooxacin, ofloxacin
(tarrivid), levoffloxacin ettc…
 Fou urth generation: e.g. moxiflloxacin, trovafloxa acin, theyy are like third
genneration witth enhance ed gram-ppositive and
d anaerobees activity..

anism of action:
Mecha a
 The
ey inhibit type
t II DN somerase (DNA gyrrase) that is necess
NA topois sary for
bac
cterial repliication.
 Quiinolones are
a bactericidal. Likke aminoglycosides, they exhib
bit concen
ntration-
dep
pendent killing and a post-antib
biotic effec
ct.
 Ressistance iss due to po
oint mutatio
ons in the target enzy
yme.

Pharm
macokinetics:
 Thee absorptio nesis is ↓ by
on of fluorroquinolon b stomac
ch contentss especially milk,
n and anta
iron acids.

 
402  
 The
ey reach alll body tiss
sues but on
nly small amounts
a can
c reach C
CSF.
 Newwer fluorooquinolone es (e.g. leevofloxacin
n and mo n) accumu
oxifloxacin) ulate in
neu
utrophils and macrop phages, soo they have long t1/2 (given oncce daily) and they
are useful aga
ainst intracellular orrganisms.
 The
ey are excrreted prima
arily uncha
anged in urine,
u so th
hey are useeful in UTIs
s.

Therap
peutic use
es:

 Fluo
oroquinolo
ones are es
specially in
ndicated in
n resistantt infection
ns. Empiric
c use of
thesse agents in minor innfections s hould be discourage
d ed.
 rd
3 generation n e.g. Cipprofloxacin have gre eater activ
vity againsst gram-ne egative
bac cteria and moderate activ ity against gram--positive organism ms and
ana aerobes. Newer
N com
mpounds (m moxifloxacin, trovaflo
oxacin) havve equal activities
a
aga ainst all.
 Levvofloxacin and moxiffloxacin arre known as “respirratory quiinolones” due to
theiir high activity against S. pneumoniia and other o atyppical resppiratory
pathogens (le egionella, mycoplasm
m ma).
 rd
3 g generation are used for most s severe sy ystemic inffections ee.g. GIT in
nfection,
urin nary tract infections
s and prosstatitis, reespiratory infections , skin andd Bone
infeections (ostteomyelitis
s).
 Trea atment of pseudomo onas infec
ction.
 Ciprofloxacin for salmonella; ente eric fever

Advers
se effects::

 GIT
T upset: na
ausea, vommiting (the most common).
 Arth
hropathy (in
( experim mental animmals):
‒ It was mannifested as
s articular d
damage & erosions in weight-b
bearing join
nts.
‒ AAlthough this side effect ha as not be
een reported in hum man, the use of
ffluoroquino
olones is not
n recom n <18 years
mmended in children rs.

 Ten
ndinitis an
nd tendo e: there is increas
on rupture sed risk o
of tendinittis and
spo
ontaneous tendon rupture espe
ecially with
h ciprofloxa
acin. The A
Achilles te
endon is
the most frequently affe
ected.

 CNS
S (1-2%): seizures and
a psychiiatric problems.
Fluo
oroquinolo ones shoulld be used
d cautious
sly in
patiients with epilepsy.
e

 Pro and ↑ ris


n of QT interval a
olongation sk of
torssade de po
ointes.

 Inhiibition of cytochrom s can


me P-450 isozymes
incrrease levels of other drugs.

 
  403
D. INHIBITORS OF
O BACTE
ERIAL ME
ETABOLISM

Sulfon
namides
s and trim
methopriim

Mecha
anism of action:
a

 Baccteria cannnot utilize preformed


p folic acid;; they use
aminobenzzoic acid (PABA) a
P-a as a precursor for
syn
nthesis of dihydroffolic acid d (by the enzyme
dihyydrofolate synthase) e), then too the acttive form
tetrrahydrofoolic acid (b by the enzzyme dihyydrofolate
ductase), necessary
red n for bactterial nucleic acid
syn
nthesis.
 Sullfonamidees are struc
ctural anallogs of PA
ABA. They
com
mpete with h PABA at the enzymme dihydrofolate synth
hase resultting in inhib
bition of
dihydrofolic acid
a synthe
esis.
 Trim
methoprimm acts by y inhibitionn of next step (formation off the activ ve form
olic acid) via inhibitio n of dihydrrofolate reductase ennzyme.
tetrrahydrofo
 Botth sulfonam
mides and trimethop
prim are ba
acteriostatiic drugs.
 zole is a combina
Co--trimoxaz ation of sulfamethoxazole (400 mg) and
trim m (80 mg) to
methoprim t produce
e sequential block of
o folic acid
d synthesis
s.
 Bac cterial resisstance occ
curs by de
evelopmentt of alterna
ative metaabolic path
hway for
folic
c acid syntthesis.

Pharm
macokinetics:

 Moost sulfonam mides are adequatel y absorbed from the e GIT (exceept sulfasallazine).
 Theey reach alll body fluid
ds includin
ng CSF (evven without meningititis).
 Sulfonamidess are metabolized i n the live er and exc creted by the kidne ey. The
aceetylated prroducts ca an precipittate and crystalize
c in acidic urine leaading to
urin
nary stoness (crystalluria).
 Alk
kalinization n of urine can en nhance ac ctivity of sulfonami des and reduce
crysstalluria.

 
404  
Therap
peutic use
es:

 Co-trimoxaz
zole:
 T
Treatment of upper respiratorry tract inffections an
nd bronchhitis cause
ed by S.
p
pneumoniaa and H. in
nfluenza.
 T
Treatment of non-co
omplicated
d UTIs and prostatitiis.
 T
The drug of
o choice for
f treatme ent of Pneeumocystis
s carinii p
pneumonia
a (PCP)
a
and toxop
plasomosiss in AIDS p
patients.

 Oth
her sulfonamide com
mbination
ns:
 Fansidar is a comb bination o
of sulfadox
xine + pyrrimethaminne is used
d as an
a
alternative
e treatme ent of m malaria caused
c by
b chlorroquine-re
esistant
P
Plasmodiu um falcipa
arum mala aria
 S zine is poorly
Sulfasalaz p ab
bsorbed combinatio
c on of sullfapyridine
e + 5-
a
aminosalic
cylic acid used
u to tre
eat ulcerative colitis.
 S
Silver sulffadiazine is used top
pically for the
t treatme
ent of burn
n.

Advers
se effects::

- Hyp
persensitiivity reactions:
- Fever andd rash are e the mosst
common.
- Steven-Jo ohnson syndrome e
is a rare
e, but fata al form o
of
extensive skin an nd mucuss
membrane e lesions s due to o
hypersenssitivity reacttion.

- Cryystalluria: due to precipitation


p n
of ssulfonamid
de metabo
olites in ac
cidic urine leading to
o colic, hem
maturia an
nd even
anuuria.

- Kerrnicterus: sulfonamides causee displacement of biilirubin from


m plasma protein
binding sitess. When th hey are g iven to neonates (or( in the last 2 we
eeks of
pre
egnancy), high
h free bilirubin le
evels in th
he blood can
c cross BBB andd cause
perrmanent brrain damag ge.

- Ane
emia:
 Megalobllastic ane
emia: due to folic acid deficie
ency (can be preven
nted by
c acid supplementattion)
giving folic
 Aplastic anemia
a lea
ading to grranulocyto
openia and thromboccytopenia.
 Hemolytic c anemia in patientss with G-6--PD deficie
ency.

 
  405
E. MISCELLANEOUS AND LESS COMMON ANTIBACTERIAL AGENTS

Metronidazole

 Metronidazole was introduced as an antiprotozoal agent but it is also active


against anaerobic bacteria such as Bacteroides, Clostridia, and some
Streptococci (see antiamebic drugs)
 It is effective in the therapy of pseudomembranous colitis, and serious
anaerobic infections (e.g. sepsis secondary to bowel disease).

Fusidic acid

 Fusidic acid is a narrow-spectrum steroid antibiotic active mainly against Gram-


positive bacteria. It acts by inhibiting bacterial protein synthesis.
 It is used in combination with other antistaphylococcal agents in staphylococcal
sepsis, and topically for staphylococcal infections (e.g. as eye drops).
 It is associated with GIT upset (common), skin eruptions and jaundice.

Streptogramins: Quinupristin and dalfopristin

 Quinupristin and dalfopristin are new class of protein synthesis inhibitors. They
inhibit bacterial protein synthesis by binding to the 50S ribosomal subunit.
 Individually, they exhibit only mild bacteriostatic activity, but combined together as
an intravenous injection, they exert bactericidal activity against drug-resistant
Gram-positive spp. including vancomycin-resistant Enterococci and MRSA.
 Their use is limited to serious bacterial infections when other antibiotics fail.
 Bacterial resistance is still uncommon.
 Both drugs undergo extensive first-pass hepatic metabolism and must be given
as an intravenous infusion.
 Adverse effects include inflammation and pain at the infusion site, arthralgia,
myalgia and GIT upset.

Oxazolidinones: Linezolid

 Linezolid is the first member of this new class of protein synthesis inhibitors
(approved in 2000). They inhibit bacterial protein synthesis by binding to the
50S ribosomal subunit.
 It is active against a wide variety of drug-resistant Gram-positive bacteria such
as vancomycin-resistant Enterococci and MRSA. The drug is also effective
against some anaerobes, such as C. difficile.

 
406  
 Its use is limited to serious bacterial infections when other antibiotics fail.
 Bacterial resistance is still uncommon.
 Adverse effects include thrombocytopenia and GIT upset.

Rifaximin

 It is a poorly-absorbed antibiotic related to rifampicin (see antiTB drugs). It has


activity against many Gram positive and Gram negative bacteria. Like rifampin, it
inhibits bacterial DNA and RNA synthesis.
 It is used orally for hepatic encephalopathy to suppress intestinal bacteria that
produce ammonia.
 It is recently approved to reduce symptoms of abdominal bloating and flatulence
associated with irritable bowel syndrome.

Nifuroxazide (Antinal)

 It is a poorly-absorbed antibiotic related to nitrofurantoin (see UTI). It has


activity against many Gram positive and Gram negative enteropathogenic
bacteria. The exact mechanism is unclear.
 Bacterial resistance is still uncommon.
 It is used orally as intestinal antiseptic in cases of infectious diarrhea.

Polymixins

 The polymixin antibiotics are polymixin B and colistin (polymixin E). They exert
their antibacterial action by disrupting the outer cell membrane.
 They have selective and rapid bactericidal action on Gram-negative bacilli,
especially Pseudomonas and coliform spp.
 They are not absorbed from the GIT.
 Due to their high toxicity, they are used only locally for eye and skin infections.

Antipseudomonal drugs

 Antipseudomonal penicillins: e.g. ticarcillin, piperacillin, etc.


 Monobactams: e.g. aztreonam
 Carbapenems: e.g. imipenem, meropenem.
 3rd and 4th generation cephalosporins: e.g. ceftazidime & cefipime
 Fluorinated quinolones: e.g. ciprofloxacin.
 Tobramycin and gentamycin. 

 
  407
CAS
SE STUDY
Y
Part 3
3:
Tre
eatmentt of Urin
nary Trac
ct Infections (U
UTI)

Definittion: infecttion of the urinary tra


act indicate
ed
by presence of o > 10 pus ce ells/HPF in
centrifu
uged urine sample (p pyuria).

Sterrile pyuria: presence e of pus ccells witho


out
orga
anisms. It occurs in some co onditions e.g.
renaal TB, ana algesic nep phropathy, and bladdder
tum
mors.
Baccteriuria: urine
u culturre proves presence of
bactteria in urine
e with or without symp
ptoms of UTTI.

▌Pred
disposing factors

 Fem
males > maales due to
o short uretthra – Children > aduults due to
o bad hygieene.
 Imm
munosupprression e.gg. in cases of diabetees mellitus.
 Pressence of obstructio
on to urin e flow e.g g. congennital abnorrmalities, urethral
stric
ctures, and
d stones.

▌Caus
sative org
ganisms

 E-c
coli is the most
m commmon organ nism (80%
%) in uncom
mplicated iinfection.
 Stap
phylococc hyticus (10
cus saproph 0%).
 Oth
her Gram-n negative bacilli
b e.g.. Proteus, Pseudem
monus & K Klebsiella are the
com
mmonest organisms.
o in complic
cated infec
ction.

▌Clinical picture

 Upp
per UTI (p hritis): infla
pyeloneph ammation of the
rena
al tissue orr pelvis.
Mannifestations: loin pain
n, chills, & fever.

 Low cystitis): in
wer UTI (c nflammatio
on of the bladder
b
& urrethra.
Mannifestations: dysuria (burni ng mictu
urition),
urgeency & freq
quency of urination.

 Unc
complicate ccurs for the 1st
e) UTI: oc
ed (simple
time
e without complicatio
c ons. It is usu
ually lower UTI.

 Com nt) UTI: ussually upp


mplicated (recurren per UTI
with
h complica
ations e.g. renal ston es or anatomical dys
sfunction.

 
408  
▌Laboratory investigations

 Urine analysis: the most common test.


‒ Macroscopic: color, pH, glucose, protein, etc.
‒ Microscopic: WBCs, RBCs, epithelial cells, crystals, casts, ova, etc.

 Urine culture & sensitivity test:


‒ Culture and sensitivity test is essential in recurrent UTI because mixed
infection and drug resistance are common.

‒ Does the in-vitro test necessarily reflect the in-vivo response to drugs?
NO because:
- The antibacterial drug is not excreted in sufficient amount in urine or
excreted in urine but as inactive metabolite.
- Presence of contraindications of the drug e.g pregnancy or children.
‒ Most bacteria causing UTI cause alkaline urine, but strongly alkaline urine is
suggestive of Proteus infection because it is urease +ve and splits urea into
ammonia (NH3) in the urine.

 Plain X-ray & i.v. pyelography: to exclude stone or obstruction.


 Serum creatinine to avoid use of nephrotoxic drugs.

▌Management of UTI

1. Specific treatment: Antimicrobial drugs:

 In lower UTI (usually simple):

‒ The antimicrobial drug should be excreted in large amount in urine.


‒ Drugs used routinely: co-trimoxazole, amoxycillin or nitrofurantoin.
‒ Duration of treatment: 3-5 days.
‒ In some cases, simple UTI could be treated by single large dose of
fosfomycin (3g oral once) but with lower cure rates.

 In upper UTI (usually recurrent):

‒ The antimicrobial drug should be bactericidal of high tissue penetration e.g.


aminoglycosides, fluorinated quinolones or 3rd gen cephalosporins.
‒ Duration of treatment: 10-14 days followed by chemoprophylaxis for 15-30
days to prevent recurrence.
‒ Indications of chemoprophylaxis: recurrent upper UTI and UTI in pregnancy
and children.
‒ Drugs used routinely in chemo-prophylaxis: co-trimoxazole, amoxycillin or
nitrofurantoin.

 
  409
2. Changing the urinary pH: Drug % Renal excretion
of unchanged drug
 Normal urine pH is 5.2-6.5. It is Ampicillin 75-92
possible, by the use of Amoxicillin 60-98
pharmacological agents, to produce Piperacillin 75-90
Ticarcillin 80-98
urinary pH values ranging from ~ 5.0
Aztreonam 65-95
to 8.5. Imipenem 5-40
Meropenem 62-83
 Alkalinization of the urine: Cephalexin 91-100
Ceftriaxone 65-95
Indications: Ciprofloxacin 30-50
 To enhance the activity of Levofloxacin 61-86
sulfonamides & aminoglycosides. Moxifloxacin 20
Gentamycin >90
 To prevent uric acid stones and
Topramycin >90
sulfonamides crystalluria.
Nitrofurantoin 27-56
 To relieve dysuria (burning Co-trimoxazole 50-75
micturition) in some cases of
bladder infection.
 E. coli is inhibited in alkaline medium.

Alkalinizing agents:
 Oral: sodium and potassium citrate salts: citrate is metabolized into
bicarbonate which is excreted in urine.
 Intravenous bicarbonate solution: contains 5% NaHCO3.

 Acidification of the urine:

Indications:
 To enhance the activity of nitrofurantoin, fosfomycin, and hexamine.
 Acidifying agents can lead to dangerous systemic acidosis in cases of renal
or hepatic impairment.

Acidifying agents:
 Oral: ascorbic acid > 2 g/d.
 Intravenous ammonium chloride (NH4Cl) solution.

3. Urinary antiseptics:

 Hexamine (Methenamine)
‒ In acidic urine, hexamine is hydrolyzed into ammonia and formaldehyde.
Formaldehyde is bactericidal and lacks bacterial resistance. Urine must be
acidified (pH below 5.5) to get this effect.
‒ Side effects: Chemical cystitis.

 
410  
 Nitrofurantoin

‒ Nitrofurantoin causes bacterial DNA damage by an unclear mechanism


‒ It is used only as a urinary antiseptic in lower simple UTI against E. coli.
Other urinary tract Gram-negative bacteria are often resistant.
‒ It becomes more active in acidic urine.
‒ It is contraindicated in renal failure.
‒ Adverse effects include hemolytic anemia in patients with G-6-PD
deficiency, peripheral neuritis, and dark brown urine.

 Fosfomycin

‒ Fosfomycin inhibits bacterial cell wall synthesis.


‒ It is a broad spectrum antibiotic against many Gram positive and negative spp.
‒ It is used only as a urinary antiseptic in lower simple UTI.
‒ It becomes more active in acidic urine.

Causes of failure of treatment:

 Improper choice, dose, or duration of drug treatment.


 Bacterial resistance.
 Presence of complications e.g. renal calculi, ureteric stricture, etc.
 Renal failure.
 Immunocompromized patient: e.g. DM

Contraindicated drugs Recommended drugs


UTI with  Nitrofurantoin: absolutely contraindicated  Penicillin
Renal because it is nephrotoxic and needs  Most cephalosporins
Failure acidic urine to act while we cannot acidify
 Co-trimoxazole
urine in renal failure.
 K salts and acidifying agents: absolutely
contraindicated because they can cause
dangerous acidosis or hyperkalemia.
 Aminoglycosides.
UTI in  Sulfonamides → kernicterus.  Penicillin
Pregnancy  Fluorinated quinolones → arthropathy.  Most cephalosporins
and  Aminoglycosides → congenital deafness.
lactation  Tetracyclines → teratogenic and cause
yellow staining of teeth.
 Chloramphenicol → grey baby syndrome.

 
  411
Part 4
4: Che
emotherapy of TB and Leprosy
y

Overview:

 Myccobacteriu culosis, o
um tuberc one of a
nummber of mycobacteria, can lead to o
serious infec ctions of the lungss, genito--
urin
nary tract, skeleton,
s and
a mening ges.
 Worldwide esstimations record 9 m million new
w
cases every year, and approxximately 2
million die of the
t disease each yea ar.
 Treaatment of TB (and other myc cobacteria))
pressents theraapeutic prob blems beca ause:
 Tubbercle bac cilli are either exxtracellularr
(meetabolicallyy activ
ve), in
ntracellularr
(meetabolicallyy inactive) or inact ive inside e
neccrotic case eous material. Howe ever, mostt
of tubercle ba acilli are intracellular with sloww
growwth rate. Resista ant strain ns occurr
natuurally to anny agent given as so le drug.
 Baccterial r
resistance e is common,
partticularly in patients non-adhere
n ent to the treatment
t protocol.
p
 Thee organism m grows slo owly and m may require e 6-24 monnths of treaatment.

Genera
al rules du
uring TB th
herapy:

 Nevver treat TB g: at least two


T by a single drug t of the first line d
drugs, are used to
prevvent emerg gence of reesistant st rains.
 Nevver add a single dru ug to a failling regim
men!
 Treaatment mu ust be conttinued for long period (6-24 moonths) to eeradicate bacilli.
b
 Pooor patient complian nce is the ccommones st cause of
o therapeuutic failure,, so it is
bettter to give drugs in a single daiily dose to
o enhance patient
p commpliance.
 Thee patient must
m be followed up for detection of adve erse effectts of drugs
s and/or
therrapeutic fa
ailure.

█ FIRS
ST LINE ANTI-TUBE
A ERCULOU
US DRUGS
S

 Isonniazide
 Rifa
ampicin (R
Rifampin)
 Ethambutol
 Pyrrazinamide
e
 Streeptomycin
n

 
412  
1. Ison
niazid (IN
NH)

Mecha
anism of action
a

 INHH is an anaalog of pyridoxine (vittamin B6).


 INHH is a proddrug that is
s activated
d inside M. tuberculoosis into
active metab bolite whicch inhibits the synth hesis of mycolic
m
acid, an esse ential compponent of tthe mycob bacterial cell
c wall (b bactericidaal).
 Myycolic acid d is presen
nt only in M
M. tubercullosis, so IN
NH has no o activity on
o other
baccteria or attypical myc
cobacteriaa.
 Ressistance iss due to mutations
m in yme respon
n the enzy nsible for cconversion
n of INH
into
o the activee metabolite.
N.B.
macokinetics
Pharm
IN
NH exerts co ompetitive e
 Abssorption of INH is good d after oral o anntagonism with pyrid doxine
admministrationn. It penettrates mosst body fluuids fo
or enzyme aapotryptoph han
inclluding the CSF. leading to cliinical picturre of
 It a
accumulate es in case eated lesio
ons and can c pyyridoxine deeficiency
(neurotoxicityy). 
atta
ack both extra- and intracellula
i ar bacilli.
 INHH is acetyylated in the liver. The rate of
aceetylation is genetically determin
ned (“rapid d acetylators” and “s low acetylators”).

Therap
peutic use
es
 Treatment off TB: INH is adminisstered in combination with oone or morre other
firstt-line drugss to minimize the devvelopmentt of resistance.
 For prophylax xis (close contacts),
c INH is use
ed alone.

Advers
se effects

I Inh ↓ metabolissm of othe


hibition of CYP450 (↓ er drugs, es
specially p
phenytoin).

N Neu
urotoxicityy:
‒ High levells of INH compete
c wwith pyrido
oxine at the enzyme pyridoxal kinase
leading to peripheraal neuropaathy.
‒ It is more common in slow accetylators.
‒ It could bee minimize
ed by co-ad
dministratiion of pyrid
doxine (vittamin B6).

patotoxiciity with jau


H Hep undice (3%%):
‒ It is due to
o accumulaation of toxxic metabo
olites in the liver.
‒ It is more common in rapid accetylators and in ind dividuals ovver 35 years.

H Hyp
persensitivity reactions with rrash and fe
ever (2%).

H Hem nemia in patients


molytic an p witth G-6PD deficiency
d

 
  413
2. Rifa
ampin (Riifampicin
n)

Mecha
anism of action
a

 Rifa ampin sele ectively inh


hibits bactterial DNA--
dep pendent RNA po olymerase e enzyme e
lead ding to ↓ RNA
R synthhesis (huma an enzyme e
is nnot affected d).
 Rifa ampin is bactericida
b al for both h intra- and
d
extracellular M. M tubercu ulosis, and atypical mycobacte
m ria.
 It iss also effec
ctive againnst many G Gram-positive and Gram-nega
G ative bacte
eria.
 Res sistance develops
d rapidly
r if th
he drug is used alon
ne. It is duue to a change in
the polymerasse enzyme e.

Pharm
macokinetics
 Rifa
ampin is absorbed
a orally
o and can reach
h all body tissue an d fluids in
ncluding
the CSF, pleuural and asscetic fluidss.
 It c
can reach TB cavitie es, sputumm and pen acrophage killing intra- and
netrate ma
extracellular TB
T bacilli.
 It eenters ennterohepattic circullation and d induces s hepatic microsom mes to
deccrease the s of other d
e half-lives drugs.
 d through the bile and urine
It iss excreted e. Its metaabolites caause oran
nge-red
disccoloration of urine, stool,
s tearss, and swea
at; the patient should
d be warned.

Therap
peutic use
es

 Treaatment of TB in com
mbination w
with INH and
a pyraziinamide. R
Rifampin and
a INH
are the most effective
e antitubercu lous drugs
s.
 Alth
hough rifam
mpin has activity
a aga
ainst many
y Gram positive and negative bacteria
b
but should be
e used only TB to prev
y against T vent develo
opment of resistance
e.
 Treaatment of leprosy in combination of dap
psone and clofazemiine.
 Useed prophyylacticallyy for indiividuals exposed
e to meninggitis caus
sed by
Men ci or H. influ
ningococc uenzae.

Advers
se effects

‒ Heppatotoxicity (rare): abnormal liver enzy ymes, jaun


ndice, etc. especially
y in old
patiients and patients
p wiith chronic
c liver disea
ase.
‒ Mic
crosomal enzyme
e nduction le
in eading to decrease
d half-lives
h o
of other dru
ugs.
‒ Flu--like symp
ptoms: witth intermitttent therap
py.
‒ Red
d tears and urine: ha
armless, b
but can sta
ain contact lenses.
‒ T upset (na
GIT ausea and vomiting)
v a
and skin ra
ash.

 
414  
3. Ethambutol

Mechanism of action

 Ethambutol inhibits arabinosyl transferase involved in cell wall biosynthesis.


 Ethambutol is specific bacteriostatic for M. tuberculosis. It is active against
intra- and extracellular TB bacilli.
 Relatively less toxic and resistant strains develop slowly.

Therapeutic uses

 Treatment of TB in combination with INH and rifampin to minimize resistance.


 Treatment of TB during pregnancy because it is the least toxic.

Adverse effects

‒ Peripheral neuropathy and visual disturbances (1-5%):


‒ Optic neuropathy manifested by field defects and red-green color
blindness. These effects are reversible on stopping the drug.
‒ There is no specific treatment other than stopping of the drug.

‒ Hyperuricemia and gout: due to ↓ uric acid excretion.


‒ GIT disturbance & hypersensitivity reactions.

4. Pyrazinamide

Mechanism of action

Pyrazinamide is taken up by the macrophages and is converted into the active


product by the acidic medium of the lysosomes inside the macrophages. It inhibits
mycobacterial cell wall functions.

Therapeutic uses: In combination with INH and rifampin to minimize resistance

Adverse effects

‒ Hepatotoxicity is the major side effect.


‒ Hyperuricemia and gout: due to ↓ uric acid excretion.

5. Streptomycin

 It is one of the aminoglycosides. It is bactericidal for extracellular bacilli but it is


ineffective against intracellular bacilli. Why?

 Therapeutic uses: in combination with INH & rifampin to treat resistant

 
  415
pulmonary TB and renal TB (because 50-90% is excreted unchanged via the
kidney). It is given by IM injections.

 Adverse effects: nephrotoxicity, ototoxicity (8th nerve damage), and NM block.

█ SECOND-LINE ANTI-TUBERCULOUS DRUGS

 Para aminosalicylic acid


 Ethionamide
 Cycloserine
 Fluoroquinolones
 Capreomycin

1. Para aminosalicylic acid (PAS)


PAS is an analog of PABA; it works similar to sulfonamides but only against
mycobacteria.

2. Ethionamide
Ethionamide, like isoniazid, blocks the synthesis of mycolic acids. Resistance
develops rapidly. It commonly produces severe GI disturbances.

3. Other drugs: fluoroquinolones, cycloserine, etc.

█ REGIMEN OF TB THERAPY

 Patients with active TB:


‒ Initial phase (first 2-4 months): 4 drugs are used (RIPE): (Rifampin + INH +
Pyrazinamide + Ethmabutol). N.B. Recent guidelines now recommend
giving a 'fourth drug' such as Ethambutol routinely - previously this was only
added if drug-resistance was suspected.
‒ Continuation phase (next 4-6 months): at least 2 drugs are used (INH +
rifampin).

 Patients with latent TB (i.e. patients with +ve Tuberculin skin test and had
history of contact to a person proved to have TB): INH alone for 6 months or dual
Rifampicin + INH for 3 months.

 Patients with meningeal TB: are treated for a prolonged period (12-18 months)
with the addition of steroids.

 
416  
 TB during pregnancy: the only anti-TB drug which is absolutely contraindicated
is streptomycin because of the high risk of congenital deafness. The other first
line anti-TB drugs are safe for use in pregnancy.

 TB with liver disease: INH, rifampin, and pyrazinamide are hepatotoxic but
because of their effectiveness, they should be used depending on monitoring of
liver function tests. In severe liver damage, only one drug can be used.

 
█ CHEMOTHERAPY OF LEPROSY

1. Dapson

 Structural analogue of PABA and chemically related to sulfonamides.


 Mechanism of action: similar to sulfonamides (bacteriostatic).
 Therapeutic uses: treatment of leprosy with rifampin for 2-5 years.
 Adverse effects:
‒ Nausea, vomiting, skin rash
‒ Hemolysis and methemoglobinemia,
‒ Exacerbation of skin lesion of lepromatous leprosy.

2. Clofazimine

 Mechanism of action: inhibit mycobacteria DNA synthesis (bactericidal).


 Therapeutic uses: in combination with dapson and rifampin to prevent
resistance.
 Adverse effects: abdominal pain, brown urine, atropine-like actions.

3. Rifampin: is the most active antilepromatous drug available.

 
  417
Part 5: Antiviral Drugs

█ ANTI-HERPES VIRUS AGENTS

1. Acyclovir

 Acyclovir is a purine analog that inhibits the activity of viral DNA polymerase.
 It is active against herpes simplex virus (HSV) types I and II, and to a lesser
extent against Epstein–Barr virus, varicella-zoster virus, and cytomegalo
virus (CMV).
 Adverse effects: reversible nephrotoxicity and neurotoxicity

2. Gancyclovir

 Similar to acyclovir but highly active against CMV.


 Adverse effects: reversible neutropenia and thrombocytopenia

█ ANTI-INFLUENZA AGENTS

1. Amantadine and rimantadine

 Amantadine and rimantadine inhibit the uncoating and replication of the viral
RNA in infected cells.
 They are used to treat influenza A infections when administered within the first
48 hours of symptoms, and as prophylaxis during flu season.
 Adverse effects: mild CNS effects (insomnia, nervousness) and some GI
dysfunction. Patients with a history of seizures require close monitoring.

2. Ribavirin

 Ribavirin, a guanosine analog that appears to inhibit viral RNA polymerases; the
mechanism of action is not clear.
 It is used to treat respiratory syncytial virus (RSV) and influenza A and B.
 Adverse effects: Hemolytic anemia and teratogenic in pregnancy.

3. Zanamivir and Oseltamivir (Tamiflu)

 Zanamivir, (administered by inhalation), and oseltamivir, (administered orally), are


neuraminidase inhibitors.
 They are used for the treatment and prophylaxis of acute uncomplicated
influenza infection. The agents are effective against both influenza A and B.
 Abdominal pain and GI dysfunction are common with oseltamivir. Zanamivir
may cause bronchospasm.

 
418  
█ ANT
TI-RETROV
VIRAL DR
RUGS

1. Rev
verse Tra
anscriptas
se Inhibittors (RTIs
s):
■ Nucleosid s (NRTI): zzidovudine
de analogs e, lamivudin
ne, tenofovvir
■ Non-Nucleoside an
nalogs (NN NRTI): efav
virenz, nevirapine

Adv
verse effeccts:
‒ A
All agents:: periphera
al neuropatthy and intteraction with
w CYP4550.
‒ Z
Zidoviodin
ne: myopatthy and an emia.

2. Pro
otease inh
hibitors: ritonavir,
r lo
opinavir, attazanavir

Adv
verse effeccts:
‒ A
All agents:: diabetes, hypertrigl yceridaem
mia and hyp
percholest erolaemia
‒ Ritonavir is
s the mostt potent inh
hibitor of CYP450
C kn
nown.

3. Fus
sion recep
ptor protein inhib
bitors: Enffuvirtide.
‒ It competees with the
e gp41 sub
bunit of the
e HIV-1 virral envelop
pe and prev
vents
ffusion to the cell membrane C D4 receptoors.

4. Inte
egrase inhibitors: Raltegraviir
‒ It is a new
w class of drugs
d that inhibit the
e viral enzyme integra
rase to pre
event
HIV replicaation and viral
v integra
ation into the
t host ce ell.

█ ANT
TI-HEPATIITIS VIRUS
S DRUGS

erferon-alpha and peginterf


1. Inte pha (Pegas
rferon-alp sys)

 Inteerferons arre group of natural ccytokines released


r by
b host cellls in response to
infe
ection. The
ey have se ave antivirral effects and regulate the
everal classses; all ha
immmune functtion.
 Gennetically engineered interferonn-α is used, in commbination w with ribavirin, for
bothh HBV and d HCV. Peginterferonn has long duration (injected on nce-week kly s.c.).

 Anttiviral mec
chanism:
‒ Interferon binds to cell membra ane recepttors to initiate a
sseries of re
eactions le
eading to in
nhibition of
o viral repliication.
‒ T
They prom mote apopttosis of vira
al-infected
d cells.

 Advverse effec
cts:
‒ Flu-like symptoms: muscle
m paiin, fever, and
a fatigue e.
‒ Bone marrrow depres ssion: neuttropenia.
‒ Neuropsyc chiatric effects: deprression, co onvulsions.

 
  419
2. Lamivudine

 It is a nucleoside reverse transcriptase inhibitor (NRTI) used for HBV infections,


providing effective and rapid response in most patients, and slows progression
of liver fibrosis.

3. Sofosbuvir (Sovaldi®)

 It is a nucleotide analog used for treatment of HCV in combination with ribavirin


and interferon.
 It works by inhibition of viral RNA polymerase.

4. Grazoprevir/Elbasvir (Zepatier®)

 It is a recently approved combination for treatment of HCV (genotypes 1, 3, 4).


 Grazoprevir is a NS3/4A protease inhibitor. This protease enzyme enables the C
virus to survive and replicate in host cells. Elbasvir is a NS5A inhibitor. NS5A is a
protein needed by the virus for various stages of infection.
 The effectiveness and adverse effects of sofosbuvir, grazoprevir, and elbasvir are
under current investigation.

█ Management of chronic HCV infection

 Treatment with interferon-α alone gives 10-15% success rate in achieving long
term clearing of plasma hepatitis C RNA. A combination of interferon and
ribavirin gives 50% success rate.
 About 50% of successfully treated patients will relapse despite treatment.
 HCV genotype will give guidance to the length of treatment and response
rate:
‒ Those with genotype 2 and 3 can achieve SVR after 24 weeks as they have
better response to treatment.
‒ In genotype 1 and 4, therapy is continued for 48 weeks due to lower
response rate.
 The following also predict a good long term response to interferon:
‒ Younger age.
‒ Female gender.
‒ Absence of cirrhosis on liver biopsy.
‒ Non-black racial origin.
‒ Low hepatic iron.

 No HCV vaccine is currently available.

 
420  
Part 6
6: Che
emotherapy of Fungal Infectio
ons

Fungal infection iss termed mycosis

▌Type
es of fung
gal infecttions:

 Muc
cocutaneo
ous (supe
erficial)
infe
ections:
‒ Derrmatophytees: cause in
nfection
of skin, hairr, and nails: e.g.
tine
ea capitiss (scalp), tinea
cruris (groinn), tinea pedis
(foo
ot), onycho
omycosis (nails).
‒ Yeaasts: causse infectio
ons of
mooist skin and mucousm
membranes: e.g. Cand ans causing oral, pha
dida albica aryngeal, vvaginal, & bladder
infe
ections.

 Sys ycoses: arre fungal in


stemic my nfections affecting
a internal org
gans. It oc
ccurs in
imm
munocomp
promized patients
p e.g
g. cryptoco
occosis, an
nd aspergiillosis (lung
g).

▌Class
sification
n of antifu
ungal dru
ugs:

A. Druggs for c 
Systemic oles: Fluconazole, Itraaconazole,
Azo
mucoc cutaneous
s drugs Voriconazole.
infectio
ons:  Gris
seofulvin
 Terb
binafine

Topical  Azooles: Ketocconazole, M Miconazole e,


drugs Clottrimazole, Tioconazo
T ole, etc.
 Nystatin
 Terbbinafine.
 Otheer drugs: Tolnaftate,
T Ciclopirox
x,
Nafttifine, Whitfield ointm
ment, Gentiian
viole
et, Castella
ani paint, T
Tincture iod
dine.

B. Druggs for sys


stemic  Azooles: Fluconazole, Itraaconazole,
infectio
ons: Voriconazole.
 Ampphotericin--B
 Fluc
cytosine
 Caspofungin

 
  421
1. Azo
oles
[Ketoc
conazole, Miconazo
ole, Flucon
nazole, Itra
aconazole
e, Voricon azole]

Mecha
anism of ac
ction

 Azo
oles inhibitt fungal cy e P450 ne
ytochrome ecessary fo
or ergoste hesis, a
erol synth
major compon nent of fun
ngal cell m
membrane. This will alter
a memb
brane perm
meability
and
d disrupt itss function.
 The
ey are broaad spectrum fungista atic against many derm
matophytees and can
ndida.

Pharmacokinetic cs
 Abssorption of azoles from
m stomach h if affected
d by food and gastric HCl.
 Flucconazole caan reach th
he CSF with good con ncentration
ns. The othher drugs cannot.
 Flucconazole iss excreted in the urinee mostly unnchanged

Therap
peutic use
es

 Sup
perficial fu
ungal infec
ctions: [ke
etoconazo
ole – itraco
onazole – miconazo
ole]
‒ D
Dermatoph
hytes infecttion of the skin (tinea
a), hair, and
d nails (onnychomyco
osis):
‒ For skin infec
ction: treatmment continued for 2-4
2 weeks..
‒ For hair infecttion: treatm
ment continued for 6-8
6 weeks.
‒ For nail infecttion: treatm
ment contin
nued for 3-6 monthss.

‒ M
Mucocauta
aneous can
ndidiasis: o
oropharyngeal, vulvo
ovaginal, eetc.

 Sys
stemic fun
ngal infecttions: [itra
aconazole – fluconazole – vorriconazole
e]
‒ IItraconazoole (orally or
o IV) is the
e drug of choice
c
ffor systemiic blastommycosis.
‒ FFluconazo ole (orally or
o IV) is the
e drug of choice
c
ffor system
mic candid diasis, andd cryptoco occal
mmeningitiss (because e it the onlyy azole tha
at can
ccross to CS
SF with good concen ntration).
‒ VVoriconazole is th he drug of choicee for
iinvasive as sis of the lu
spergillos ung.
N.B. Itraco
onazole hass
placed ketocon-
largely rep
Advers
se effects azole in m
most uses.

‒ Heppatotoxicity and ↑ of serum tra


ansaminas
ses.
‒ Azo
oles inhibit hepatic CYP450 e enzymes (ffluconazole
e is the leaast among them).
‒ Kettoconazolee causes antianderrogenic efffects: gynnecomastiia and imp potence
e to ↓ gona
due adal steroid
d synthesiss.
‒ Vorriconazole
e causes trransient viisual distu
urbances.

 
422  
2. Amphotericin-B

Mechanism of action
Amphotericin B is polyene macrolide that binds to ergosterol of fungal cell
membranes and forms “pores” that alter membrane stability and allow leakage of
cellular contents.

Pharmacokinetics

 Amphotericin B is polar compound that cannot be absorbed from the GIT or


cross the CSF. It should be administered IV or intrathecal.
 Half-life is 15 days. Dialysis is ineffective in case of toxicity.
 Because of significant toxicity, amphotericin B is available in liposomal form in
which the drug is enclosed in lipid microspheres “liposomes”. These lipid
microspheres bind preferentially to ergosterol in the fungal cell membrane with
lower affinity to mammalian cell membranes.

Therapeutic uses

Amphotericin B has the broadest spectrum of activity. It is used to treat severe


systemic fungal infections, including those caused by Candida albicans,
Histoplasma capsulatum, Cryptococcus neoformans, Coccidioides immitis,
Blastomyces dermatitidis, Aspergillus spp., and Sporothrix schenckii.

Adverse effects

‒ Amphotericin B is highly toxic drug. It causes nephrotoxicity in 80% of patients


which is dose-dependent and may be irreversible.
‒ Chills and fever in 50% of patients.
‒ Seizures and neurotoxicity
‒ Hypokalemia and thrombocytopenia

█ OTHER ANTIFUNGAL DRUGS

Flucytosine

 Flucytosine is actively transported into fungal cells and is converted to the uracil
form 5-fluorouracil (5-FU) which inhibits nucleic acid synthesis. Human cells
lack the ability to convert large amounts of flucytosine into 5-FU.
 It is often used in combination with other antifungal agents (because of rapid
development of resistance) to treat severe systemic fungal infections.
 Adverse effects: Flucytosine is relatively nontoxic; the major adverse effect is
depression of bone marrow at high doses and hair loss.

 
  423
Griseofulvin

 Griseofulvin binds to microtubules and prevents spindle formation and mitosis


in fungi. It is fungistatic and requires long duration of therapy.
 The drug binds to keratin structures and accumulates in skin, hair, and nails.
 Griseofulvin is used orally for long-term therapy of dermatophyte infections of
the hair and nail.
 Adverse effects: hepatotoxicity (liver functions should be checked during
therapy), hypersensitivity reactions (skin rash), and CNS effects.

Terbinafine

 Terbenafine inhibits the fungal enzyme squalene epoxidase. This leads to the
accumulation of the sterol squalene, which is toxic to the organism.
 Like griseofulvin, it accumulates in keratin structures and used orally or topically
for treatment of dermatophyte infections of the hair and nail.

Nystain

 Nystatin is polyene macrolide very similar in kinetics and mechanism to


amphotericin B.
 It is too toxic for parenteral administration and is used only topically.
 It is active mainly against Candida, and is used topically for oralpharyngeal and
vaginal candidiasis.

Caspofungin

 It is large cyclic peptide that disrupts the fungal cell wall resulting in cell death.
 Caspofungin is used by i.v. route for salvage therapy in patients with severe
invasive aspergillosis or esophageal candidiasis who failed to respond to
amphotericin B (second line drug).

Ciclopirox

 Broad-spectrum antifungal effective against dermatophytes and yeasts. The


mechanism is unclear.
 It is used topically for skin and nail infections.

 
424  
Part 7
7: Anttiamoeb
bic Drug
gs

 Thee causativve organis sm for a amoebiasiss is Entamoeba h histolytica. Initial


infe
ection occuurs by ingeestion of th
he cyst form. The infectious cyysts pass into the
coloon, where they deve elop into trrophozoitees. These motile org
ganisms ad dhere to
coloonic epithhelial cells
s. Here, the trophozoites fe eed, multiiply, encyyst and
eveentually pa
ass out in th
he feces “a asymptommatic carrier”.
 The
e motile organism inv
vades the ccolonic muucosa caus
sing “amooebic dyseentery”.
The
ey may also spread to
t the liver causing acute
a “amo
oebic liverr abscess””.

▌Class
sification
n of antia
amoebic d
drugs:

A. Lum
minal amoe ebicidal drugs: thesse agents  Diloxanid
de
destroyy the troph
hozoites off E. histolyttica that  Iodoquinool
eventuaally form in
nto cysts in
n the intesstine.   Paromom mycin

B. Tisssue amoeb bicidal druugs: they ddestroy the e  Emetine aand


invadinng trophoz zoite in the
e tissues (e
e.g. liver) but
b dehydroe emetine
are inefffective ag
gainst the trophozoitees in the  Chloroquuine: active e only
intestin
nal lumen. Their use now is ve ery limited d. against livver infection.

C. Mixe
ed tissue and lumin
nal amoeb
bicides: Meetronidazoole, tinidaz
zole,
orn
nidazole.

 
  425
1. Mixed amebicides: Metronidazole

Structure and mechanism of action

 Metronidazole is 5-nitroimidazole compound. Anaerobic protozoa & bacteria lack


mitochondria; instead, they have ferredoxin oxidoreductase enzyme in the
cytoplasm to generate ATP.
 This enzyme system can transfer electron to the 5-nitro groups of metronidazole
converting it into cytotoxic product. This product causes DNA damage and
inhibits DNA repair.
 Metronidazole is active against anaerobic organisms including anaerobic bacteria:
(e.g. Bacteroids, C. difficile), E. histolytica, G. lamblia, T. vaginalis, B. coli.

Therapeutic uses

 Amebiasis: Metronidazole is the most effective agent available for the treatment
of all forms of amoebiasis except asymptomatic person that excrete cysts.
Metronidazole kills trophozoites but not cysts.
 Urogenital trichomoniasis: 250 mg t.d.s. for 7 days is the treatment of choice.
The other partner should be treated simultaneously.
 Giardiasis: 250 mg t.d.s. for 5 days.
 Balantidiasis: 750 mg t.d.s. for 5 days
 Severe anaerobic infections: e.g. puerperal sepsis, peritonitis, acute ulcerative
gingivitis, etc.

Adverse effects

‒ GIT: nausea, vomiting & metallic taste.


‒ CNS: insomnia, headache, vertigo, parasthesia, ataxia, & seizures.
‒ Blood: bone marrow depression, leukopenia &
thrombocytopenia. N.B. Disulfiram is a drug
used for treatment of chronic
‒ Disulfiram-like reaction: metronidazole
alcoholism. It blocks metab-
causes accumulation of acetaldehyde if alcohol
olism of acetaldehyde by the
is consumed leading to nausea & vomiting. enzyme aldhyde dehydro-
‒ Dark brown urine. genase leading to accumula-
tion of acetaldehyde → nau-
sea, vomiting, flushing, etc.
2. Luminal amebicidal drugs:

 Diloxanide: Is active against both trophozoite and cyst forms in the intestinal
lumen but not in the intestinal wall or extraintestinal tissues. The mechanism is
unknown.

 
426  
 Iodo
oquqinol: it is iod quinolone derivative
dinated q e. Is activve agains
st both
trop
phozoite an nd cyst forrms in the intestinal lumen butt not in thee intestinall wall or
extrraintestinal tissues. The
T mecha anism is un nknown.
Beccause it co ontains iodine, it ca
an cause dermatitis
s and perrsistent diarrhea
(iod
dine intolerance).

 Parromomycin: it is a poorly-abso
p orbable am
minoglycos
side antibi otic. It is used
u as
alte ol and diloxxanide. The antiamoebic mechhanism is unclear
ernative to iodoquino u
but may be dued to alteeration of tthe cell me
embrane permeabilit
p ty and leakage of
cell contents.

Uses o
of luminal drugs
 Theey are used for treatme
ent of asym
mptomatic or
o mild inte
estinal infecction.
 Theey can also
o be added d to metroonidazole in
n acute ammoebic dyssentery as well as
hep
patic absce
ess to erad
dicate cystts in the lum
men which
h may causse relapse.

3. Tiss
sue ameb
bicidal drrugs:
 
 Emetine and dehydroe
emetine: E
Emetine is a
nt alkaloid
plan d. Dehydro oemetine iis a potennt
deriivative. All have significant toxiicity.

 Chloroquine:: it is a synthetic drug (se


ee
trea
atment of malaria).
m

 Both drugs can


c sed as altternative to
be us
mettronidazole
e in the treatment o of amoebic
live
er abscesss or extraintestinal a
amoebiasis
s,
howwever, their use now is very lim
mited.

 
  427
Part 8
8: Anttimalarial Drugs
s

█ Life cycle of malaria and


a sites
s of drug actions

A. Sexual cycle in the fema


ale mosqu
uito.

e in man an
B. Asexual cycle nd consistts of:

 S e stage: sp
Sporozoite porozoites injected by
y the mosquito rapidlyy enter the liver.

 E
Exo-erythrrocytic (liv
ver) stage
es:
PPre-exo-errythrocytic stage (pri mary liver stage):
SSporozoitees in the liv
ver underg
go multipliccation to form tissue
e schizonnt which
eeventually ruptures and
a release es merozo oites to infect RBCs. During this stage
tthe patientt is asympttomatic.

S
Secondaryy exo-erythhrocytic sta age (persisstent liver stage):
s
S
Sporozoite
e of P. vivaax or P. ovvale form dormant
d hypnozoite es in the liv
ver that
c
can persistt and reacttivated afte
er a latent period cau using relap
pse of mala aria.

 EErythrocyttic stage:: merozoittes liberated from ruptured


r l iver schizont will
invade RB BCs to forrm blood schizont. This bloo od schizo nt will eventually
rrupture to liberate merozoites
m and toxic
c products s causing fever and clinical
rd
mmanifestations of maalaria (everyy 3 day in
n tertiary malaria).
m

 G
Gametocyytes stagee: some mmerozoites develop in
nto gamettocytes wh
hich will
b
be sucked by the mo
osquito to c
complete the
t sexual cycle.

 
428  
▌Type
es of trea
atment:

I. Chem
moprophy
ylaxis: (Killling the pa
arasite before multiiplication inside RB
BCs)

 Cau hylaxis: killing the pa


usal proph arasite in th
he liver:
Proguanil
P a
and primaq
quine.

 Clin
nical prophylaxis: killing the pa
arasite as soon as th
hey reach tthe RBCs:
Proguanil
P a
and chloro
oquine.

II. Suppressive or
o clinical cure: (kill ing the pa
arasite in the
t RBCs))

 Chlooroquine: for
f chloroqquine-senssitive malaria.
 Quinine and mefloquine
m e: for chloro
oquine-res
sistant malaria.
 Arte
emisinin an
nd its analo
ogs: for ch
hloroquine--resistant malaria.
m
 Sulffonamidess and pyrim
methamine : Fansidar (sulfadoxine + pyrim methamine)).

dical cure: (clearing the dorman


III. Rad nt hypnozooite from the liver to prevent re
elapse)
Primaquine
P e + chloroq
quine

IV. Pre
evention off transmis
ssion: (kill ing the ga
ametocytees)
Proguanil,
P primaquin
ne, and pyrrimethaminne.

Chloro
oquine

Mecha
anism of action
a
 Inside RBCss, the malarial
m pa
arasite digest
hem
moglobin in n a vacuole inside th
he parasite
e cell
to acquire amino acids e essential for
mulltiplication..
 Chlo
oroquine enters the
e digestio
on vacuole
e by
simple diffussion. The acidic ppH inside the
vacuole make es chloroq
quine ionizzed and, thus,
t
can
nnot diffuuse outsid de the vacuole and
bec
comes trappped inside
e it.
 Inside the vacuole,
v th
he ionized
d chloroq
quine
reaccts with thet digesttive produ
ucts of he
eme
mollecule and d forms a highly to oxic complex
thatt kills the parasite.
p
 Chlo
oroquine is a blood nticide forr Plasmodium vivax,, P. ovale, and P.
d schizon
mallariae but not
n for P. falciparum
f which is usually
u resiistant.

 
  429
Pharmacokinetics
 Absorption from the GIT is good and complete.
 It binds to melanin-rich tissue e.g. skin, eye, etc.
 Chloroquine has very high Vd (100-1000L/Kg) and slow rate of elimination.

Therapeutic uses

 Malaria (except P. falciparum):


‒ For treatment of acute attack: 1g IV followed by 500 mg after 6, 24, 48h.
‒ For chemoprophylaxis: 500 mg once/week orally.
 As a 2nd line for treatment of amoebic liver abscess: 500 mg daily for 14 days.
 As a 2nd line (after metronidazole) for treatment of giardiasis.
 Treatment of rheumatoid arthritis and SLE: 500 mg/day then 250 mg/day.

Adverse effects

‒ Long term administration:


‒ Skin: pruritus (common), dermatitis, bleaching of hair, and alopecia.
‒ Visual disturbances: corneal deposits, optic atrophy, and retinopathy.
‒ Rapid i.v. injection causes hypotension, ECG changes, and cardiac arrest.
Parenteral administration is better avoided or should be infused slowly.
‒ Hemolysis in G-6PD deficient persons.

Quinine (and Quinidine)

Structure and mechanism of action

 Both quinine and quinidine are derivatives from the park of cinchona tree.
Quinidine is the D-isomer of quinine.
 Quinine is highly active blood schizonticide against the four species of human
malaria parasites, but it is primarily used to treat chloroquine-resistant P.
falciparum, often in combination with doxycycline.
 The exact antimalarial mechanism is unknown.

Therapeutic uses: treatment of chloroquine-resistant P. falciparum.

Adverse effects (more common with i.v. administration)

‒ Cinchonism: tinnitus, ringing in ears, blurred vision, headache, and vomiting.


‒ CVS: hypotension, syncope, cardiac arrhythmias and ↑ QT interval.
‒ Hypoglycemia is important and may be fatal side effect with i.v. administration.
It is due to stimulation of insulin release).

 
430  
‒ ackwater fever (rarre): massivve hemoly
Bla ysis with fever, hemmoglobinure ea, and
darrk urine. Th
he pathoge enesis is u nclear.
‒ Hem molysis inn G-6PD de eficient pe rsons. N.B.
N
‒ Hyppersensitivvity reactions. Although
A qu inine cause
es
uterine conttractions but
b the
WHO
W guidel ines consid
der
Artem
misinin and its ana
alogs pregnancy
p iis NOT a
contraindica uinine.
ation of qu
Structu
ure and mechanism
m m of action
n

 Arteemisinin iss the active compoonent of a Chinese


e
herb
b. The compound contains unusual peroxide e
dge. Cleavvage of th
brid his peroxidde bridge inside the
e
dige
estion vac cuole libeerates oxyygen free e radicals
s
which are leth
hal to the parasite.
p
 Arte
emisinin and
a its anaalogs are very rapidly actingg
bloo
od schizo onticides against all huma an malaria
a
para
asites. The
ey have no o effect on hepatic stages.

Pharm
macokinetics

 Arteemisinin is insoluble and can b e used only orally.


 Arteesunate and
a arteme ether are semisynth
hetic analo ogs with im
mproved solubility
to b
be suitable
e for parenteral adm inistration..

Therap
peutic use
es

 Com
mbination of artemisinin and otther antimalarials is now
n the sttandard tre
eatment
of fa m malaria in nearly alll endemic areas.
alciparum
 IV a
artesunatee is now re
ecommend ded by thee WHO in preference
p to IV quin
nine due
to fe
ewer side effects.

Adverse effects: GI
G disturba
ances and,, rarely, allergic reacttions and hhemolysis..

Mefloq
quine

Structu
ure and mechanism
m m of action
n

 Meffloquine is structurally related tto quinine.


 Stroong blood schizonticide again nst P. falcip
parum by unknown
u mechanism.
 It is given onlyy orally be
ecause parrenteral addministratio
on causes severe irritation.

Therap es: treatme


peutic use ent of chlorroquine-re
esistant falc
ciparum m
malaria.

se effects:: neurotoxicity: head


Advers dache, vertigo, psych
hosis, and cconfusion..

 
  431
Primaquine

Mechanism of action

Primaquine is a hepatic schizonticide (for P. vivax and P. ovale) and gametocide


(in all 4 types of malaria), through an unknown mechanism.

Therapeutic uses

 Radical cure: after successful treatment, it is given orally for 15 days to kill the
dormant hypnozoites in the liver and prevent relapse.
 Causal prophylaxis: but prolonged course of primaquine should be avoided.
 Prevention of transmission: by killing the gametocytes in all 4 types of malaria.

Adverse effects

‒ Methemoglobinemia nearly in all patients.


‒ Severe hemolysis in patients with G-6PD deficiency.
‒ GIT upset, visual disturbance, and leucopenia.

Antifolate drugs
(Pyrimethamine, Proguanil, Fansidar)

Mechanism of action: ↓ folic acid synthesis necessary for DNA synthesis.

Therapeutic uses

 Fansidar is a combination of sulfadoxine + pyrimethamine. It produces


sequential block of folic acid synthesis. It is used to treat the acute attack of
chloroquine-resistant malaria in combination with quinine or artemisinin.
 Proguanil is used for prophylaxis and for prevention of transmission.
 Pyrimethamine is used for clinical cure and for prevention of transmission.

 
432  
Part 9: Anthelmintic Drugs

Helminths can be divided into three main groups:

Nematodes (round worms) Trematodes (flukes) Cestodes (tape worms)

Intestinal nematodes: Taenia saginata


Schistosomes
Ascaris lumbricoides Taenia solium
Ankylostoma duodenale Intestinal flukes: Diphyllobothrium latum
Enterobius vermicularis Heterophyes heterophyes Hymenolepis nana.
Trichuris trichiura Metagonimus yokogawia Hydatid tape worm.
Strongyloides stercoralis Fasciolopsis buski
Trichostrongylus
Liver flukes:
Tissue nematodes: Fasciola hepatica
Filaria.
Lung flukes:
Guinea worm.
Paragonimus westermani
Cutaneous and visceral
Larva migrans.

█ DRUGS EFFECTIVE AGAINST NEMATODES

1. Benzimidazoles
[Albendazole – Mebendazole – Thiabendazole]

Mechanism of action

These agents inhibit microtubule synthesis and glucose uptake by the worms.
The two effects lead to ↓ ATP production and paralysis of the worm.

Therapeutic uses

Benzimidazoles are broad-spectrum anthelmintic drugs.


 For intestinal nematodes: (400 mg once daily on an empty stomach for 2-3 days).
 For tissue nematodes: (400 mg twice daily with a fatty meal for 10 days).
 For hydatid disease: (400 mg twice daily for one month).
 Neurocysticercosis: albendazole is the drug of choice (400 mg twice daily with a
fatty meal for 21 days). It is given with corticosteroids to minimize the immune
reaction caused by the dying organisms.

Adverse effects: usually mild and mainly GIT upsets.

 
  433
2. Pyrantel pamoate

 It causes depolarizing neuromuscular blockade and inhibits ChE enzyme in the


parasite → paralysis of the worm.
 It is used for intraluminal round worms.
 Adverse effects: very few.

3. Diethylcarbamazine (Hetrazan)

 It causes paralysis of microfilaria and alters their surface structure, making them
more susceptible to destruction by host defense mechanisms. The mechanism is
unknown.
 Therapeutic uses: treatment of filariasis (Wucheraria bancrofti and Loa loa).
 Adverse effects: sudden death of the microfilaria can produce severe allergic
reactions e.g. fever, leukocytosis, eosinophilia, edema, rashes, tachycardia and
headache (corticosteroids may be needed to suppress these allergic reactions).

█ DRUGS EFFECTIVE AGAINST TREMATODES AND CESTODES

1. Praziquantel

Mechanism of action
It increases cell membrane permeability of Ca2+. This leads to rapid and prolonged
muscle contraction and paralysis of the worm.

Therapeutic uses
 Treatment of schistosomiasis: it is active against adult worm and all immature
forms in all species (40 mg/kg single dose orally).
 Other flukes: e.g. H. heterophyes, Fasciolopsis buski, Paragonimus westermani.
 Cestodes: T. saginata, T. solium, D. latum, and H. nana.

Adverse effects
 Nausea, vomiting, drowsiness, arthralgia, myalgia and low grade fever.
 Mild elevation of liver enzymes.

Contraindications and cautions:


Praziquantel is contraindicated in ocular cysticercosis, because parasite
destruction in the eye may cause immune reaction and irreversible damage. Some
workers also caution against use of the drug in neurocysticercosis for the same
reason.

 
434  
2. Bith
hinol

 It in
nhibits the parasite re
espiration.
 It iss an altern
native to triclabenda
t azole for Fasciola
F hepatica
h ((sheep live
er fluke
infe
ection) and as an alte
ernative to praziquantel for pulm
monary pa
aragonimiiasis.

3. Niclosamide
e

 It is a second line drug for


f most ta
ape worm (cestodes)) infectionss.
 It aappears too act by inhibition of oxida
ative phos
sphorylatio on and ↓ energy
prod duction.
 It is given as 2g
2 once on n an emptyy stomach.
 Advverse effeccts are minimal becauuse the dru
ug is not absorbed.
a

 
  435
 

436
Review Questions
1. Mention the antibacterial mechanism of action and the major adverse effects of
each of the following drugs:
 Penicillin
 Clarithromycin
 Aminoglycosides
 Tetracyclines
 Vancomycin
 Isoniazid
 Ethambutol

2. Mention the antiviral mechanism of action and the major adverse effects of each
of the following drugs:
 Interferon-alpha
 Acyclovir
 Amantadine

3. Mention the antifungal mechanism of action and the major adverse effects of
each of the following drugs:
 Itraconazole
 Amphotricin B

4. Mention the antiprorozoal mechanism of action and the major adverse effects of
each of the following drugs:
 Metronidazole
 Chloroquine
 Artemisinin

5. Give a short account on the drug management and drug(s) of best choice in the
following infections:
 Bacterial meningitis
 Community-acquired pneumonia
 Bites (human or animal)
 Typhoid fever
 Pseudomembranous colitis
 Gonorrhea

6. Discuss drug treatment of hepatitis C virus. Mention the mechanism of action


and major adverse effects of interferon-alpha.
7. Discuss drug treatment of amebic liver abscess. Mention the mechanism of each
drug.
8. Discuss drug treatment of chloroquine-resistant P. falciparum malaria. Mention
the mechanism and major adverse effects of each drug.

437
Of each of the following questions,
select ONE BEST answer: 6. Amoxicillin is inferior to ampicillin
for the treatment of the following
1. The penicillin G preparation with infection
the longest duration of action is A. Typhoid
A. Benzathine penicillin B. Tonsilitis
B. Sodium penicillin C. Shigella enteritis
C. Potassium penicillin D. Subacute bacterial endocarditis
D. Procaine penicillin E. Gonorrhoea
E. Pipracillin
7. Which one of the following
2. If a patient gives history of statements about ampicillin is false?
urticaria, itching and swelling of lips A. Its activity is enhanced by sulbactam
following injection of penicillin G, then B. It causes maculopapular rashes
A. He will develop milder reaction C. It is the drug of choice for Listeria
whenever penicillin is injected monocytogenes infection
B. He can be given ampicillin safely D. It eradicates most strains of MRSA
C. He can be given cephalosporins safely E. Pseudomembranous colitis may occur
D. He can be given oral phenoxymethyl with its use
penicillin safely
E. All natural and semisynthetic 8. The mechanism of antibacterial
penicillins are contraindicated for him action of azithromycin involves
A. Binding to a component of the 50S
3. The most important reason for ribosomal subunit
highly restricted use of penicillin G B. Inhibition of translocase activity
injections in present day therapeutics C. Blockade of binding of aminoacyl –
is its tRNA to bacterial ribosomes
A. Narrow spectrum of activity D. Selective inhibition of ribosomal
B. Potential to cause hypersensitivity peptidyl transferases
reaction E. Inhibition of DNA–dependent RNA
C. Short duration of action polymerase
D. Neurotoxicity
E. Nephrotoxicity 9. In the empiric treatment of severe
bacterial infections of unidentified
4. Cefotaxime act by the following etiology, this drug, often used in
mechanism: combination with an aminoglycoside,
A. Inhibition of bacterial protein synthesis provides coverage against many
B. Inhibition of bacterial cell wall staphylococci
synthesis A. Amoxicillin
C. Inhibition of bacterial nucleic acid B. Clavulanic acid
synthesis C. Erythromycin
D. Inhibition of bacterial folic acid D. Nafcillin
synthesis E. Tetracycline
E. Inhibition of bacterial cell division
10. C. difficile colitis is more common
5. Benzathine penicillin injected once complication with the use of:
every 4 weeks for 8 years or more is A. Cephalosporins
the drug of choice for B. Vancomycin
A. Agranulocytosis patients C. Co-trimoxazole
B. Prophylaxis of bacterial endocarditis in D. Meropenem
patients with valvular defects E. Flucoloxacillin
C. Prophylaxis of rheumatic fever
D. Treatment of anthrax 11. Beta – lactamase production by
E. Treatment of sinusitis strains of Haemophilus influenzae,

438
Moraxella catarrhalis, and Neissera 16. Methicillin resistant staphylococci
gonorrhoeae confers resistance do not respond to β-lactam antibiotics
against penicillin G. which one of the because
following antibiotics is most likely to be A. They produce a β-lactamase which
effective against all strains of each of destroys methicillin and related drugs
the above organisms? B. They elaborate an amidase which
A. Ampicillin destroys methicillin and related drugs
B. Ceftriaxone C. They have acquired a penicillin binding
C. Clindamycin protein which has low affinity for β-
D. Erythromycin lactam antibiotics
E. Piperacillin D. They are less permeable to β-lactam
antibiotics
12. A 19-year-old woman with
recurrent sinusitis has been treated 17. Indicate the sulfonamide whose
with different antibiotics on several sodium salt yields a nearly neutral
occasions. During the course of one solution which is suitable for topical
such treatment she developed a severe use in the eye
diarrhea and was hospitalized. A. Sulfadiazine
Sigmoidoscopy revealed colitis, and B. Sulfacetamide
pseudomembranes, were confirmed C. Sulfamerazine
histologically. Which of the following D. Sulfamethizole
drugs, administered orally, is most
likely to be effective in the treatment of 18. Which of the following is not true of
colitis due to C difficile? sulfonamides?
A. Ampicillin A. They are primarily metabolized by
B. Azithromycin acetylation
C. Clindamycin B. They are more likely to produce
D. Metonidazole crystalluria in alkaline urine in which
E. Tetracycline they are less soluble
C. They may exert bactericidal action in
13. The drug of choice for Lyme the urinary tract
disease is: D. Used alone, they have become
A. Doxycycline therapeutically unreliable for serious
B. Sulfonamides infections
C. Penicillin
D. Erythromycin 19. Clavulanic acid is combined with
E. Topramycin amoxicillin because
A. It kills bacteria that are not killed by
14. The drug of choice for anaerobic amoxicillin
infections is: B. It reduces renal clearance of
A. Tetracycline amoxicillin
B. Sulfonamides C. It counteracts the adverse effects of
C. Penicillin amoxicillin
D. Erythromycin D. It inhibits beta lactamases that destroy
E. Metronidazole amoxicillin

15. The drug of choice for typhoid 20. Which toxic effect of
fever is: aminoglycoside antibiotics is most
A. Tetracycline irreversible in nature?
B. Sulfonamides A. Optic neuropathy
C. Penicillin G B. Ototoxicity
D. Ciprofloxacin C. Hepatotoxicity
E. Metronidazole D. Muscle weakness
E. Kidney damage

439
21. Highest incidence of antibiotic C. Oseltamivir
associated pseudo membranous D. Ribavirin
enterocolitis has been noted with the E. Ritonavir
use of
A. Ampicillin 27. The primary mechanism of
B. Chloramphenicol antibacterial action of penicillin
C. Vancomycin involves inhibition of:
D. Clindamycin A. Beta-lactamases
E. Gentamycin B. Cell membrane synthesis
C. N-acetylmuramic acid synthesis
22. Which antibiotic class can cause D. Peptidoglycan cross-linking
teeth staining and dental enamel E. Transglycosylation
hypoplasia if given to small children?
A. Fluoroquinolones 28. Fluid expressed from the penile
B. Cephalosporins chancre of a patient revealed to be
C. Tetracyclines infected with Treponema pallidum, the
D. Aminoglycosides best course of action would be to:
E. Macrolides A. Administer a single oral dose of
fosfomycin
23. Antiviral agents that are active B. Administer a single oral dose of
against cytomegalovirus (CMV) include gentamycin
which of the following? C. Inject intramuscular benzathine
A. Ganciclovir penicillin G
B. Acyclovir D. Treat with oral tetracycline for 7 d
C. Amantadine E. Treat with vancomycin
D. Oseltamivir
E. Ribavirin 29. Which of the following statements
about beta-lactam antibiotics is false?
24. Which one of the following drugs is A. Cephalexin and other first-generation
least likely to be effective in the cephalosporins do not cross the
treatment of esophageal candidiasis, it blood-brain barrier
is used by the oral route? B. Ceftriaxone and nafcillin are both
A. Amphotericin B eliminated mainly via biliary secretion
B. Clotrimazole C. Instability of penicillins in gastric acid
C. Fluconazole can limit their oral absorption
D. Griseofulvin D. Renal tubular reabsorption of
E. Ketoconazole amoxicillin is inhibited by probenecid
E. Ticarcillin has activity against several
25. Which one of the following drugs is gram negative rods
most appropriate for oral use in vaginal
candidiasis? 30. A patient needs antibiotic
A. Clotrimazole treatment for native valve, culture
B. Griseofluvin positive infective enterococcal
C. Fluconazole endocarditis. His medical history
D. Flucytosine includes a severe anaphylactic reaction
E. Nystatin to penicillin G during the last year. The
best approach would be treatment with
26. The antiviral actions of this drug A. Amoxicillin-clavulanate
include inhibition of both RNA and DNA B. Aztreonam
synthesis. The drug is used for the C. Ceftriaxone
treatment of severe respiratory D. Ticarcillin
syncytial virus infections in neonates. E. Vancomycin
A. Amantadine
B. Amprenavir

440
31. If ampicillin and piperacillin are B. Intravenous third-generation
used in combination in the treatment of cephalosporin
infections resulting from Pseudomonas C. Oral amoxicillin
aeruginosa, antagonism may occur. D. Oral ciprofloxacin
The most likely explanation is that E. Oral neomycin
A. Ampicillin is bacteriostatic
B. Ampicillin induces beta-lactamase 36. Clarithromycin and erythromycin
production have very similar spectra of
C. Autolytic enzymes are inhibited by antimicrobial activity. The major
piperacillin advantage of clarithromycin is that it
D. Piperacillin blocks the attachment of A. Does not inhibit hepatic drug-
ampicillin to penicillin-binding proteins metabolizing enzymes
E. The 2 drugs form an insoluble B. Eradicates mycoplasmal infections in
complex a single dose
C. Has greater activity against H pylori
32. Which statement about D. Is active against methicillin-resistant
vancomycin is accurate? strains of staphylococci
A. Active against methicillin-resistant E. Is active against strains of
staphylococci streptococci that are resistant to
B. Bacteriostatic erythromycin
C. Binds to PBPs
D. Hepatic metabolism 37. The primary mechanism of
E. Oral bioavailability resistance of gram-positive organisms
to macrolide antibiotics including
33. A 52 years old patient with signs erythromycin is
and symptoms suggestive of typical A. Changes in the 30S ribosomal subunit
bacterial meningitis. Treatment of this B. Decreased drug permeability of the
patient should be initiated immediately cytoplasmic membrane
with intravenous administration of C. Formation of drug-inactivating
A. Amoxicillin acetyltransferases
B. Cephalexin D. Formation of esterases that hydrolyze
C. Benzylpenicillin G the lactone ring
D. Nafcillin E. Methylation of binding sites on the
E. Piperacillin 50S ribosomal subunit

34. The mechanism of antibacterial 38. A suitable treatment for


action of azithromycin involves: community-acquired pneumonia with
A. Antagonism of bacterial translocase little risk of drug interactions can be
activity achieved with
B. Binding to a component of the 50S A. Azithromycin
ribosomal subunit B. Clindamycin
C. Inhibition of DNA-dependent RNA C. Doxycycline
polymerase D. Erythromycin
D. Interference with binding of E. Vancomycin
aminoacyl-tRNA to bacterial
ribosomes 39. Regarding the toxicity of
E. Selective inhibition of ribosomal aminoglycosides which statement is
peptidyl transferases accurate?
A. Gentamicin and tobramycin rarely
35. If a patient had been scheduled for cause renal damage
elective colonic surgery, optimal B. Ototoxicity includes vestibular
prophylaxis against infection would be dysfunction, which may be irreversible
achieved by the use of C. Ototoxicity is reduced if loop diuretics
A. Intravenous cefoxitin are used to facilitate the renal

441
excretion of aminoglycoside 44. Silver sulfadiazine is clinically used
antibiotics for:
D. Reduced blood creatinine is an early A. Treatment of Rocky Mountain spotted
sign of aminoglycoside nephrotoxicity fever
E. Skin reactions are very rare following B. To prevent infections of skin burns
topical use of neomycin C. Treatment of typhoid fever
D. Treatment of Legionella pneumonia
40. Your 23-year-old female patient is E. Treatment of lead toxicity
pregnant and has gonorrhea. The
medical history includes anaphylaxis 45. Which statement about the
following exposure to amoxicillin. The fluoroquinolones is accurate?
most appropriate drug to use is A. Antacids increase their oral
A. Azithromycin bioavailability
B. Cefixime B. Contraindicated in patients with
C. Ceftriaxone hepatic dysfunction
D. Ciprofloxacin C. Fluoroquinolones are drugs of choice
E. Doxycycline in a 6-year-old child with a urinary
tract
41. In a patient suffering from D. Gonococcal resistance to
pseudomembranous colitis due to C fluoroquinolones may involve changes
difficile with established in DNA gyrase
hypersensitivity to metronidazole the E. Modification of dosage is required in
most likely drug to be of clinical value patients renal impairment.
is
A. Amoxicillin 46. Which adverse effect is most
B. Chloramphenicol common with sulfonamides?
C. Doxycycline A. Stevens Johnson syndrome
D. Levofloxacin B. Hematuria
E. Vancomycin C. Kernicterus in the newborn
D. Neurologic dysfunction
42. Trimethoprim-sulfamethoxazole is E. Skin rash
established to be effective against
which of the following opportunistic 47. Which statement about
infections in the AIDS patient? ciprofloxacin is accurate?
A. Cryptococcal meningitis A. Antagonism occurs if used with
B. Herpes simplex dihydrofolate reductase inhibitors
C. Oral candidiasis B. Ciprofloxacin is active against MRSA
D. Toxoplasmosis strains of staphylococci
E. Tuberculosis C. Most “first-time” urinary tract
infections are resistant to ciprofloxacin
43. A 65-year-old woman has returned D. Organisms that commonly cause ear
from a vacation abroad suffering from infections are highly resistant
traveler’s diarrhea, and her problem E. Tendinitis may occur during treatment
has not responded to antidiarrheal
drugs. A pathogenic gram-negative 48. Supplementary folinic acid may
bacillus is suspected. Which drug is prevent anemia in folate-deficient
most likely to be effective in the persons who use this drug
treatment of this patient? A. Ciprofloxacin
A. Ampicillin B. Levofloxacin
B. Ciprofloxacin C. Linezolid
C. Sulfadiazine D. Clarithromycin
D. Trimethoprim E. Trimethoprim
E. Vancomycin

442
49. Supplementary pyridoxin may prophylaxis against CMV retinitis in
prevent neurotoxicity in persons who AIDS patient is
use this drug A. Fluconazole
A. Ciprofloxacin B. Gancyclovir
B. Isoniazid C. Indinavir
C. Linezolid D. Rifabutin
D. Clarithromycin E. Trimethoprim-sulfamethoxazole
E. Trimethoprim
55. Which of the following statements
50. Which statement regarding about interferon-alpha is false?
ethambutol is correct? A. At the start of treatment, most patients
A. It is contraindicated in pregnancy experience flu-like symptoms
B. Visual adverse effects are very rare B. Therapeutic outcome is low when
C. It inhibits arabinosyl transferase used alone for HCV
involved in cell wall biosynthesis C. It is used in the management of
D. Ocular toxicity of ethambutol is hepatitis B and C
prevented by thiamine D. Lamivudine interferes with its activity
E. Resistance is common and rapid. against hepatitis B
E. Toxicity includes bone marrow
51. Interactions between this drug and suppression
cell membrane components can result
in the formation of pores lined by 56. A 22-year-old man with gonorrhea
hydrophilic groups present in the drug is to be treated with cefixime and will
molecule. need another drug to provide coverage
A. Caspofungin for possible urethritis caused by
B. Flucytosine Clmydia trachomatis. Which of the
C. Griseofulvin following drugs is least likely to be
D. Nystatin effective in non-gonococcal urethritis?
E. Terbinafine A. Azithromycin
B. Ciprofloxacin
52. Which statement about fluconazole C. Co-trimoxazole
is accurate? D. Nitrofurantoin
A. Does not penetrate the blood-brain E. Tetracycline
barrier
B. Drug of choice in treatment of 57. The drug regimen most likely to be
aspergillosis effective in treating severe
C. Induces hepatic drug-metabolizing extraintestinal amebiasis is
enzymes A. Chloroquine
D. Has the least effect of all azoles on B. Diloxanide furoate plus iodoquinol
drug metabolism C. Emetine plus diloxanide furoate plus
E. Oral bioavailability is less than that of chloroquine
ketoconazole D. Chloroquine followed by primaquine
E. Tinidazole plus diloxanide furoate
53. The following is the drug of choice
for invasive aspergillosis 58. Metronidazole is not effective in the
A. Voriconazole treatment of
B. Ketoconazole A. Amebiasis
C. Miconazole B. Infections due to Bacteroides fragilis
D. Fluconazole C. Infections due to Pneumocystis carinii
E. Itraconazole D. Pseudomembranous colitis
E. Trichomoniasis
54. The drug most likely to suppress
herpetic infections and provide 59. Which statement about
antiprotozoal drugs is accurate?

443
A. Chloroquine is an inhibitor of 62. After successful treatment of
plasmodial dihydrofolate reductase malaria, which drug should be given
B. Mefloquine destroys secondary later to eradicate schizonts and latent
exoerythrocytic schizonts hypnozoites in the patient’s liver?
C. Primaquine is a blood schizonticide A. Artesunate
and does not affect secondary tissue B. Fansidar (Pyrimethamine-sulfadoxine)
schizonts C. Chloroquine
D. Artemisinin is not useful for P. D. Primaquine
falciparum malaria E. Quinine
E. Intravenous quinine can cause serious
arrhythmia
Answers
60. Plasmodial resistance to
chloroquine is due to: 1A 14 E 27 D 40 A 53 A
A. Change in receptor structure 2E 15 D 28 C 41 E 54 B
B. Decreased accumulation of the drug in 3A 16 A 29 D 42 D 55 D
the food vacuole 4B 17 B 30 E 43 B 56 D
C. Increased activity of DNA repair 5C 18 B 31 E 44 B 57 E
mechanisms 6C 19 D 32 A 45 D 58 C
D. Increased synthesis of dihydrofolate 7D 20 B 33 C 46 E 59 E
reductase 8A 21 D 34 B 47 E 60 B
E. Induction of drug-inactivating 9D 22 C 35 E 48 E 61 A
enzymes
10 A 23 A 36 C 49 B 62 D
11 B 24 D 37 E 50 C
61. Which drug should be used for
12 D 25 C 38 A 51 D
treatment of the acute attack of P vivax
13 A 26 D 39 B 52 D
malaria but does not eradicate
exoerythrocytic forms of the parasite?
A. Chloroquine
B. Mefloquine
C. Primaquine
D. Pyrimethamine-sulfadoxine
E. Quinidine

444

You might also like